Sie sind auf Seite 1von 133

Aleksandra mega Horvat: MEDICAL ENGLISH WORKBOOK

BASICS
GENERAL HEALTH VOCABULARY
I

Complete the sentences with appropriate words or phrases. In most


cases there are several possibilities:

1. There were a lot of people at the doctors __________, so I had to wait for
hours.
2. Its just a small hospital, so all the beds in the surgical __________ are
occupied.
3. Theres been an accident, someone call a(n) __________!
4. You wont get this antibiotic without a __________.
5. They did a biopsy and took a __________ of the suspicious tissue.
6. The doctor gave her a painkiller to __________ the pain.
7. The flu is a highly ___________ disease.
8. I hurt my wrist yesterday playing tennis and today its ______________.
9. The cut was bleeding a lot, so he had to put on a __________.
10. The arm was broken and had to be put in a __________.
11. The usual symptom of an allergy is a __________ on your skin, or, in the
case of hay fever, a __________ nose.
12. I feel sick, I think Im going to __________.
13. She slipped and fell, and now shes got a nasty blue __________ on her
forehead and a(n) __________ on her cheek.

II

DISEASE, DISORDER, ILLNESS, or SICKNESS?

disease/disorder = any deviation from the normal structure or function of a


part, organ or system of the body, or the mind,
manifested by characteristic symptoms
illness

= 1. state of bad health, experience of not being healthy


for a period of time; 2. disease

sickness

= 1. feeling ill in the stomach and vomiting; 2. disease

Use them appropriately in the following sentences:


1. He died at the age of 83 after a short ___________.
2. Cirrhosis is a serious liver __________.
3. Many women suffer from morning ___________ in the first trimester of
pregnancy.
4. Heart ___________ is one of the leading health problems in developed
countries.
5. It is inconvenient if frequent flyers suffer from air ___________.
6. Her prolonged ___________ was hard for the family.

Which other words for DISEASE can you think of?

III Complete the sentences with the following words:


BENIGN
CURE
DEFICIENCY
HEAL
HEREDITARY

MALIGNANT
RECOVER
RELAPSE
RELIEF
REMISSION

1.

After surgery and chemo, shes in __________ and feeling fine. We all
Hope the cancers gone and there wont be a __________.

2.

__________ tumors are normally not life-threatening.

3. Due to his young age, it didnt take him long to __________ after the
operation.
4. In diabetics, wounds tend to __________ with difficulty.
5. Unfortunately, the biopsy proved that the tumor was __________.
6. Diseases which run in families are called ___________.
7. An efficient antibiotic will __________ the infection in a few days.
8. The treatment is going to take some time, but Ill give you an analgesic for
pain __________.
9. A vitamin __________ can lead to serious health problems.

PARTS OF THE BODY


Go through the alphabetical list of some parts of the body and sort them
out under the following headings (2 words belong to two categories):
ABDOMEN
ANKLE
ARCH
ARMPIT
BALL
BUTTOCK
CALF
CHEEK
CHEST
CHIN
ELBOW
head (10)

EYEBROW
EYELASH
EYELID
FINGER
FLANK
FOREARM
FOREHEAD
GROIN
GUM
HEEL

torso (11)

HIP
KNEE
KNUCKLE
LOBE
LOIN
NAIL
NAVEL
NIPPLE
NOSTRIL
PALM

PELVIS
SHIN
SHOULDER
SOLE
TEMPLE
THIGH
THUMB
TOE
WAIST
WRIST

arm + hand (11)

leg + foot (11)

Now look at each category and list the structures from top to bottom
according to their location on the body.

ORGANS AND ORGAN SYSTEMS

I Match the names and functions of the organ systems:


A
B
C
D
E
F

INTEGUMENTARY SYSTEM
FEMALE REPRODUCTIVE S.
CARDIOVASCULAR SYSTEM
DIGESTIVE SYSTEM
RESPIRATORY SYSTEM
MALE REPRODUCTIVE S.

G
H
I
J
K

MUSCULOSKELETAL SYSTEM
LYMPHATIC SYSTEM
URINARY SYSTEM
NERVOUS S. + SENSE ORGANS
ENDOCRINE SYSTEM

a immunity; drainage of tissue fluid; absorption of lipids


b external protection; thermoregulation; sensory reception
c exchange of gases between lungs and external environment
d coordination of all voluntary and involuntary body activities
e transport of nutrients to all body cells; elimination of waste
materials from the cells
f internal support of the body and protection of inner organs;
body movement

g production of hormones responsible for female secondary sex


characteristics; production of ova; place for growth of embryo
and fetus
h production of hormones that regulate many functions of the organism
i production of testosterone and sperm cells
j filtration of blood; maintaining balance of body fluids; endocrine function
k breakdown of foods; absorption of nutrients; elimination of solid wastes

II Decide which systems the listed groups of organs belong to:

muscles, bones, joints, tendons, ligaments

mouth, pharynx, esophagus /gullet, stomach,


intestines (small & large) + liver, gallbladder, pancreas ______________

brain, spinal cord, nerves (motor & sensory)


+ eye & ear

______________

heart, blood vessels (arteries, veins, capillaries)

______________

nose, pharynx, larynx /voice box, trachea /windpipe,


bronchi, lungs, bronchioles, alveoli

______________

______________

ovaries, fallopian tubes /uterine tubes /salpinges /oviducts,


uterus /womb, vagina, vulva + mammary glands
______________

testes & associated tubes, urethra, penis


+ prostate & other glands

______________

skin + hair, nails, glands (oil /sebaceous & sweat /sudoriferous) ________

hypophysis /pituitary, epiphysis /pineal, thyroid,


parathyroid, thymus, pancreas (islets of Langerhans),
adrenal, gonads

______________

lymph vessels & nodes + spleen, thymus gland

______________

kidneys, ureters, urinary bladder, urethra

______________

WHAT IS IT IN ENGLISH?

There are many medical words that are also frequently used in everyday
speech. What are their equivalents in general English?

ANALGESIC (ANALGETIC)

____________________________

CEREBRAL INSULT

____________________________

DILATATION (DILATION)

____________________________

EDEMA

____________________________

FETUS

____________________________

FRACTURE

____________________________

GRAVIDITY

____________________________

HEMORRHAGE

____________________________

HYPERTENSION

____________________________

INSOMNIA

____________________________

LARYNGITIS

____________________________

MENSTRUATION

____________________________

MUCUS

____________________________

MYOCARDIAL INFARCTION

____________________________

NAUSEA

____________________________

NECROSIS

____________________________

SALIVA

____________________________

SPASM

____________________________

COLLOCATIONS

Collocations are typical, common or frequent combinations of certain


words (e.g. adjectives + nouns, or verbs + nouns, etc.) in a specific
language. But what may function in one language does not necessarily
work in another. That is why you have to make sure and check in a
dictionary before you mechanically translate Croatian collocations into
English.
For example: teko disanje translates as hard (heavy, labored) breathing,
teko krvarenje as heavy bleeding, teak porod as difficult birth, teka
bolest (ozljeda) as serious (grave) disease (injury), etc.
I

See how many of the following common collocations for body


structures and functions or conditions you can get right:

KLJUNA kost
SLOBODNA rebra
TANKO crijevo
DEBELO crijevo
arterijska STIJENKA
KUCANJE srca
SPUTENA stopala
CUREI nos
MASNA koa
MASNA kosa
MASNE naslage

_______________ bone
_______________ ribs
_______________ intestine
_______________ intestine
arterial _________
heart _________
_______________ feet
_______________ nose
_______________ skin
_______________ hair
_______________ plaques

Now match the adjectives on the left with the nouns on the right to get
some more common medical collocations:
1
2
3
4
5
6
7

allergic
bedside
bone
brain
clinical
general
general

A
B
C
D
E
F
G

death
trial
anesthetic
marrow
reaction
manner
practitioner

II From the jumble below, make as many collocations around the words
DISEASE and TREATMENT as you can. Then use some of your
collocations in sentences of your own:

DISEASE

TREATMENT

HOME INFECTIOUS FATAL VIRULENT PALLIATIVE DRUG


INCURABLE DENTAL HOSPITAL CANCER CONTAGIOUS
PROGRESSIVE HEART HEREDITARY INTENSIVE CHRONIC
SURGICAL DEGENERATIVE ACUTE CONGENITAL
SEXUALLY TRANSMITTED MANAGEMENT

GIVE PROVIDE DIAGNOSE CAUSE RECEIVE CARRY


CURE SUFFER FROM TRANSMIT CATCH FIGHT
CONTROL TREAT PASS ON CONTRACT

III

Spot the medical collocations in the following text and list them
under the headings below:

IN-FLIGHT EMERGENCIES
A recent study about in-flight medical emergencies estimated that there are
an average of 30 emergencies on U.S. flights every day. Most of them are
not serious; fainting, dizziness and hyperventilation are the most frequent
complaints. But 13% of them roughly four a day are serious enough to
require a pilot to change course. The most common of the serious
emergencies include heart trouble (46%), strokes and other neurological
problems (18%), and breathing difficulties (6%).
Most people agree that plane rides are stressful. First, cabin pressures at high
altitudes can be compared to living at 5,000 to 8,000 feet above sea level.
Usually people can tolerate these pressures easily, but passengers with heart
disease may experience chest pains as a result of the reduced amount of
oxygen flowing through their blood. Low pressure can also cause the air in
body cavities to expand up to 30%. Again, most people won't notice anything
beyond mild stomach cramping. But if someone has recently had an
operation, their wound could open. And if a medical device has been
implanted in your body a splint, a tracheostomy tube or a catheter it could
expand and cause damage.
Another common in-flight problem is deep venous thrombosis the so-called
economy-class syndrome. When you sit too long in a cramped position, the
blood in your legs tends to clot. Most people just feel a pain in their calves.
But blood clots could travel to the lungs, causing breathing difficulties and
even death. Such clots are easily prevented by keeping blood flowing, so you
should walk and stretch your legs when possible.
But theres no reason for panic. Emergency medical kits with automated
defibrillators in case of heart attacks have become standard equipment on
most flights since 2004.

adjective + noun

verb + noun

Now use some of the collocations from your lists in the following
sentences:
1. One of the symptoms of food poisoning is ________ ________.
2. If there is an obstruction of the upper airways, a ________ ________
may be implanted.
3. Heart attack patients usually _________ _______ ________.
4. Deep venous thrombosis is caused by a ________ ________, and if
untreated may even ________ ________.
5. Ambulances should be equipped with ________ ________.
6. Smoking may ________ serious ________ to your respiratory and
cardiovascular systems.
.
8. In case of a ________ _________, call 911.

REGISTER
By register we mean the style or level of language (vocabulary and
grammar) we use in particular situations, from highly formal to informal,
colloquial, or even vulgar.
Additionally, each profession has its own language (jargon), which is
especially true of medicine, with its huge corpus of professional
terminology.
Professional words and idioms are usually classified as formal.
I Words for some body parts and basic body functions are frequently
used in different registers, depending on the occasion. Fill in the
tables:

MEDICAL
FORMAL

NEUTRAL

INFORMAL
COLLOQUIAL

trbuh
pupak
bol u elucu
garavica
podrigivanje
nadutost

vjetrovi
munina
povraati
imati stolicu
mokriti
SPECIALIZED

NON-SPECIALIZED
painkiller

auscultation
swelling
expectoration
hidrosis
slime
feeling sick
(examination by) touching
paroxysm
percussion
pyrexia
rhinorrhea
chest
windpipe

II Copy the passage below using neutral instead of medical terms from
the table. Make any other necessary changes:
The patient presented with intense gastralgia, eructation and constant nausea
accompanied by flatulence. He reported repeated emesis and defecation in
the past 12 hours. His abdomen was tender to palpation in the umbilical area.
Now do the opposite with the passage below:
The patient reports severe coughing fits, with coughing out slime,
accompanied by fever, sweating and a runny nose lasting for 5 days.
Examination by listening to and tapping his chest revealed no swelling of the
windpipe and upper airways. A drug to bring down the fever, painkillers and
agents to make coughing out easier were prescribed.
STATISTICS
I Look at the list of 10 TOP CAUSES OF DEATH AND DISABILITY in the
US in the last decade of the 20th century (left) and the Harvard School
of Public Health projection for the year 2020 (right):
1. Respiratory infections
2. Diarrheal diseases

1. Heart disease
2. Severe depression

3.
4.
5.
6.
7.
8.
9.
10.

Complications of birth
Severe depression
Heart disease
Stroke
Tuberculosis
Measles
Traffic accidents
Congenital anomalies

3.
4.
5.
6.
7.
8.
9.
10.

Traffic accidents
Stroke
Chronic pulmonary disease
Respiratory infections
Tuberculosis
War injuries
Diarrheal diseases
HIV/AIDS

Comment on the changes predicted to occur and their possible causes.


How might a 10 top causes list for Croatia be different at present?

II Translate the following sentence, and then add 2 more sentences


based on the lists above:
Devedesetih su godina prve na listi 10 najeih uzroka smrti bile infekcije
dinih puteva, dok se za 2020. godinu kao glavni uzrok predviaju bolesti
srca.

A LOOK INTO THE FUTURE


I Read the text about tailored medicines of the future and underline the
passages which answer the following questions:
1. What does the patient give the doctor?
2. How does the doctor reach her diagnosis and what does she prescribe?
3. What will be the main practical consequences of the decoded human
genome?
4. What are customized drugs? Which other term for them is used in the text?
5. How and why will pharmacology of the future affect human reproduction?
6. What will be the drawbacks of those wonderful new drugs?
TAILORED MEDICINES

Its the year 2025, and you are at your doctors. You're feeling terrible. It's not
just a hangover after an all-night party. You're sweating. You're listless.
You're aching all over. The doctor examines you. Then she asks for your
gene card. The computer processes the rectangle of plastic you hand her,
confirming her diagnosis of flu. She sends you to the pharmacy, where they
will create a drug specially for you.
This is a very likely scenario of what will be happening when you visit your
physician 20 years from now, given molecular biology's current pace of
progress. By then scientists will have found exactly where common diseasecausing errors lie along the genome's long, interlocked chains of DNA.

10

That will have numerous practical consequences. Your genetic profile,


recorded on a chip, will let doctors and their computerized diagnostic tools
determine your exact level or risk for a particular disease. So you'll get
customized prescriptions, created to fit on the very first try without producing
adverse reactions.
And that's not all. The genetically based pharmacology of the future will offer
you not only the usual well-known drugs like tranquilizers, antihistamines,
analgesics and antibiotics, but also all kinds of new medicines for virtually
every ailment and condition. These will range from mood and pleasure
enhancers (legal or not) to new medications for diseases likely to be much
more common in an aging population, like Alzheimer's, cardiovascular
problems and cancer.
Since in developed countries at least 20% of the population will be over 60,
drug companies will turn from contraception to conception with the purpose to
help older women have babies. As for aging men, they'll have at their disposal
libido and sex-performance boosters compared to which Viagra will seem like
baby aspirin.
But when you take your gene card to the pharmacy in 2025, bring a credit
card too. Made-to-fit drugs won't be cheap, so many of us will still have to
keep to the old aspirin!
Vocabulary check - match the underlined words from the text with their
meanings. Which two are synonyms?

1
2
3

hangover
listless
current pace

4
5

adverse
ailment

6
7
8

enhancer
conception
booster

A present tempo
B illness
C something that improves / increases the
quality, amount or strength of something
D fusion of egg and sperm cell
E feeling of illness after drinking too much
alcohol
F lacking energy and enthusiasm
G bad, negative

In paragraph 4, different kinds of drugs are mentioned. Do you know


what they are used for?
tranquilizers

antihistamines

analgesics

antibiotics

In pairs, take turns to define the following to each other:


ANTIARTHRITICS
ANTICONVULSANTS
ANTIPYRETICS
ANTISPASMODICS

ANTIASTHMATICS
ANTIDEPRESSANTS
ANTIRHEUMATICS

ANTICOAGULANTS
ANTIEDEMICS
ANTISEPTICS

11

II Put the following paragraphs about how people could be computerenhanced in the future into sequence:
GETTING CHIPPED
A
Implant technology is progressing at high speed. In England, cybernetics
experts are working on the next step. Implants that wirelessly connect the
nerves of a body part to a PC are being tested on volunteers. In such cases
the computer records the activity of the nervous system and stimulates the
nerves to produce small movements and sensations; such an implant could
eventually help a person suffering from paralysis to move parts of the body
the brain cant reach.
B
The operation to insert the chip is simple. An antiseptic swab, a local
anesthetic, an injection and a Band-Aid thats all it takes. Once the skin
heals, the chip is completely invisible, and you will hardly know its there.
C
An American family are the first volunteer test subjects for a new, implantable
computer device called VeriChip. In a few months, if they get approval by the
U.S. Food and Drug Administration, doctors will load a needle with a
microchip containing a few kilobytes of silicon memory and a tiny radio
transmitter and inject it under the skin of their left arms, where it will serve as
a medical identification device. It sounds like science fiction.
D
In the next few years, it will be possible to add sensors to the chip that will
read your vital signs pulse, temperature, blood sugar and so on and a
satellite receiver that can track where you are. There is already great interest
for such a device in Brazil, where kidnapping has become common, especially
among the rich and powerful.
VeriChip seems to be looking at a great future!
E
But VeriChip is real, and this family could be the first in a new generation of
computer-enhanced human beings. They are particularly well suited to test
VeriChip for use in medicine, because one of them suffers from allergies to
antibiotics and another has undergone chemotherapy. If a patient with
VeriChip is injured, an ER doctor can quickly access the victims medical
background by scanning the chip, which could be a lifesaver.
Now choose the correct answer:
1. The family have not had the VeriChip implanted yet because
a) they are still having second thoughts about it.
b) they are waiting for the FDA to approve it.
c) the device has some imperfections which need to be corrected
before implantation.

12

2. The actual procedure of implantation


a) is very simple.
b) is done under general anesthesia.
c) will leave a scar on your arm.
3. The future development of implant technology will make it possible to
a) restore damaged nerve tissue.
b) connect human beings with PCs.
c) cure paralysis.
4. People who will probably be most interested in having VeriChip
implanted are
a) people with serious medical conditions.
b) those whose personal security is at a greater than average risk.
c) both categories of people.

13

Aleksandra mega Horvat: MEDICAL ENGLISH WORKBOOK

MUSCULOSKELETAL SYSTEM
WHAT IS IT IN ENGLISH?
I Provide the adjective forms of the listed bones and give their English
names:
CRANIUM
ZYGOMATIC B.
MANDIBLE / MANDIBULA
SCAPULA
CLAVICLE / CLAVICULA
STERNUM
ILIUM + ISCHIUM + PUBIS
VERTEBRAL COLUMN
COCCYX
OLECRANON
CARPALS
FEMUR
PATELLA
TIBIA
FIBULA
TARSALS
CALCANEUS

cranial / skull bones


/ cheek bones

II Use the adjective forms of the nouns and verbs in brackets in the
following sentences:
1. The muscles which are attached to bones are called ________________
(SKELETON) muscles.
2. The bones of a fetus are composed of __________________ (CARTILAGE)
tissue.
3. Sphincters are _______________ _________________ (CIRCLE,
MUSCLE) structures.
4. The trachea and esophagus are some of the many ________________
(TUBE) structures in the body.
5. Bone, blood, fat and cartilage are examples of __________________
(CONNECTION) tissue.
6. Muscles and joints make the body _______________ (FLEX).

7. Some ligaments are quite ________________ (POWER).


8. The bones of the thorax and pelvis have a _________________ (PROTECT)
function.
STRENGTH TRAINING
I Work in pairs. As you read the following text, one of you should make a
list of the benefits of strength training, while the other should list the
advice for workouts. Then look at your list and tell your partner about it.
Aerobic exercise - running, swimming, biking has been in for years. But the
more researchers learn about the vast health benefits of keeping fit, the more
they stress the importance of adding weight-lifting workouts to cardiovascular
routines, with the goal to prevent injuries, osteoporosis and even heart disease.
Muscles are the engines that power every single movement we make. They are
elegant structures, consisting of strands of protein that slide over each other
when you move. As that contraction happens, your muscles give you the strength
to perform an action.
However, just using your muscles isn't the same as actively building them.
Scientists still havent fully explained the molecular process, but a leading theory
is that workouts create microscopic tears in the muscle tissue. Your body
responds by repairing the gaps with proteins, which form new filaments,
increasing your muscle mass and making you stronger.
Humans lose about 1 percent of muscle mass every year, beginning in our late
20s.
But you can fight back and regain about half the loss. The ideal workout plan will
vary depending on your age, occupation and goals. Still, some general
recommendations apply. First, you dont have to spend that much time at the
gym. The trend now is toward free weights, since they more closely mimic the
activities of life. Many of these exercises push-ups, pull-ups and squats can
be done at home, using your own body weight.
Resistance training doesn't have to be repetitive, either. Work your chest, back
and arms on one day, your thighs and calves on another. Experts recommend a
minimum of 20 minutes of resistance training three times a week, but you should
alternate those workouts with aerobic exercise as well. Workouts can differ by
the number of repetitions and intensity, light ones alternating with heavy ones. In
any case, make sure to give each muscle group at least 24 hours of rest between
sessions, and spend about 10 minutes stretching your muscles before and after
you work out. The symptoms of overtraining are clear: an increase in injuries and
pain so bad that even the smallest movement hurts.
Strong muscles don't just look good. Workouts improve glucose and cholesterol
levels, reduce blood pressure and boost metabolism. Weak muscle strength is a
major cause of frailty later in life, which can lead to falls and broken hips. And as
far as chronic illness is concerned, a Harvard study published recently found that

men who trained with weights for 30 minutes or more per week, lowered their risk
of coronary heart disease by 23 percent.
II Match the verbs on the left with the nouns and adjectives on the right to
get collocations from the text (there are also alternative combinations)
and use them in sentences of your own:
1
2
3
4
5
6
7

prevent
perform
keep
make
improve
reduce
lower

A
B
C
D
E
F
G

the risk
fit
glucose levels
an action
blood pressure
sure
injuries

III Give the opposites of the underlined words and change the sentences
accordingly:
1. Smoking raises the risk of lung cancer.
e.g. Smoking lowers/reduces/decreases/diminishes/lessens the chances to stay
healthy.
2. Cholesterol levels improve with an appropriate diet.
3. Exercise reduces the chances of osteoporosis.
4. There has been a rise in cardiovascular incidents.
5. A decrease in underage pregnancies has been noted.
Now complete the table of adjectives and adverbs. Which of them could
you use in the above sentences? Where would you put them?
ADJECTIVES
minimal

ADVERBS
slightly

slow
gradually
sudden
fast

fast
rapidly

sharp
steeply
considerable
greatly
maximal

1. Cholesterol levels gradually /rapidly /greatly /considerably improve with an


appropriate diet.
2. _____________________________________________________________
3. _____________________________________________________________
4. _____________________________________________________________
5. _____________________________________________________________

IV Make nouns from the following verbs:


suffer
diagnose
degenerate
refer
replace
recover

_____________________
_____________________
_____________________
_____________________
_____________________
_____________________

Now rephrase the sentences below using nouns insted of verbs. Be sure to
make all the necessary changes:
1. She suffered intense pain in her hip for a long time.
Her intense
2. Her GP diagnosed severe osteoarthritis.
Her GPs
3. He said that the hip joint had degenerated badly.
He said that there was a bad
4. He referred her to an orthopedic surgeon.
He gave her a
5. After doing all the necessary tests, the surgeon replaced the hip joint.
After all the necessary tests, she got a
6. It took her some time to recover fully.
After some time she made a

BLOOD, LYMPH, & IMMUNITY


COMPOSITION OF BLOOD
Produce a scheme depicting the composition of blood using the words
from the alphabetical list:
ALBUMIN
ALPHA
BASOPHILS
BETA
BLOOD
CELLS
EOSINOPHILS
ERYTHROCYTES
FIBRINOGEN

GAMMA
GLOBULINS
HORMONES
IMMUNOGLOBULINS
LEUKOCYTES
LYMPHOCYTES
MONOCYTES
NEUTROPHILS

NUTRIENTS
PLASMA
PROTEINS
PROTHROMBIN
SALTS
THROMBOCYTES
VITAMINS
WATER

ANEMIA
I In this short passage about anemia, supply the missing medical terms
with the help of the definitions below:
Anemia is any condition in which the capacity of blood to carry oxygen is
reduced. It is not a disease, but a symptom of various diseases. It can be caused
either by (1) _______________ or by a decrease in the amount of
(2) _______________ within the red blood cells.
The symptoms accompanying most anemias include (3) ______________,
weakness, (4) _________________, paleness, (5) _________________, and a
slight fever.
1
2
3
4
5

deficiency in the number of red blood cells


red oxygen-carrying pigment of erythrocytes
difficulty breathing
rapid heartbeat
low blood pressure

II Which anemia is it?


Study the notes about 7 types of anemia and decide which of the
following types they talk about. Underline the words which helped you
guess:
1 APLASTIC anemia
2 HEMOLYTIC anemia
3 HEMORRHAGE

4
5
6
7

IRON-DEFICIENCY anemia
FOLIC ACID DEFICIENCY anemia
PERNICIOUS anemia
SICKLE CELL anemia

A _________________________
-

lack of mature erythrocytes caused by low levels of vitamin B12 in the body
vitamin B12 cannot be absorbed because of the lack of intrinsic factor in
gastric juice
treatment: vitamin B12 injections

B _________________________
-

occurs due to acute (trauma, childbirth) or chronic blood loss (e.g. bleeding
ulcers)
hemoglobin levels normalize when the cause is eliminated

C _________________________
-

most common type of anemia


caused by the lack of iron, important for hemoglobin production (iron is either
deficient in the diet, or cannot be absorbed properly)

D _________________________
-

sometimes associated with the exposure to certain medications and other


agents, but mostly idiopathic
due to aplasia of bone marrow cells, which cannot produce blood cells, so
pancytopenia occurs
treatment: blood transfusions, antibiotics, bone marrow transplants

_________________________

hereditary, mostly found in black people


caused by a genetic defect in the gene responsible for hemoglobin synthesis,
so RBCs form crescents and other irregular shapes
because of their altered shape, erythrocytes cannot enter capillaries and may
cause thrombosis and tissue death
symptoms: arthralgia, acute abdominal pain, ulcerations of extremities
appears with different degrees of severity

_________________________

result of insufficient intake of folic acid (due to poor diet, drug therapy,
pregnancy etc.)

treatment: supplementing diet with folic acid or elimination of underlying


causes

G _________________________
-

caused by early destruction of RBCs due to their spheroidal shape which


makes them fragile
can be congenital (due to cell defects) or acquired (due to extrinsic factors)
treatment: splenectomy (because RBCs are destroyed in the spleen)

LYMPH CIRCULATION
I As you read about lymph circulation, provide the correct form of the
words in brackets:
Lymph (1) _______________ (ORIGIN) from blood plasma. As blood
(2) _______________ (CIRCULATION) through capillaries, small quantities of
plasma leak out into the (3) _______________ (SURROUND) tissue and
become (4) _______________ (INTERSTICE) fluid, whose function is to cleanse
and (5) _______________ (NOURISHMENT) body tissues. As that fluid enters
lymph capillaries, it becomes lymph. Lymph (6) _______________ (PASSAGE)
to larger vessels and (7) _______________ (FINAL) to lymph nodes, where it
gets (8) _______________ (FILTRATION). The large lymph vessels from the
right chest and arm join the right lymphatic duct, which (9) _______________
(DRAINAGE) into the right subclavian vein. Lymph from all other parts of the
body enters the (10) _______________ (THORAX) duct and flows into the left
subclavian vein. From these veins, lymph is redeposited into the blood and
becomes plasma, thus starting the cycle again.

IMMUNODEFICIENCY DISEASES - AIDS


I What does AIDS stand for? ___________ ___________ ___________
Read the passage about HIV (___________ ___________ ___________)
and decide if the statements below are true or false, marking the words
and expressions that helped you decide.
HIV destroys T-cell helper lymphocytes (CD4+ cells), thereby incapacitating the
cell-mediated immune response. Thus an organism whose resistance has been
lowered can easily become infected by bacteria and parasites that a healthy
immune system can normally fight. Such infections are called opportunistic, and
they range from candidiasis and herpes simplex to PCP, which causes
pneumonia, and tuberculosis.
HIV can mutate very fast, and it can lie dormant in body cells, where antibodies
cannot attack it, without revealing its antigen. It can rapidly destroy helper T-cells
and cripple the immune system even before it can counterattack. That means
that an effective vaccine should be able to stimulate the immune system to
incapacitate the virus immediately after exposure, before it can penetrate the
helper T-cells.
1.
2.
3.
4.

An HIV infection interferes with humoral immunity.


Among opportunistic infections, viral, fungal and bacterial ones can be found.
A person cannot be infected by HIV without showing any symptoms.
A vaccine which would prevent HIV from entering T-cells would be effective.

II Read about scientists predictions on the global future of AIDS.


A CURE FOR AIDS?
Although scientists as a rule dont like to make predictions about the future, they
seem to agree that the global AIDS epidemic will get much worse before it gets
any better. It looks as if this modern plague will be with us for several
generations, despite major scientific advances.
Having already claimed more than 15 million lives and left more than 40 million
people living with a viral infection that slowly erodes their immune systems, the
AIDS virus has become the deadliest microbe in the world.
Although we now know more about HIV than about any other virus, the epidemic
continues to rage in South America, Eastern Europe and sub-Saharan Africa. By
the 2020s AIDS will be by far the major killer of young Africans, decreasing life
expectancy to as low as 40 years in some countries. Reports have shown that
half of the new infections among African children are caused by breastfeeding.

It is Asia, with its huge population at risk, that will have the biggest impact on the
global spread of AIDS. The magnitude of the pandemic could range from 100
million to 1 billion, depending largely on what happens in India and China. Four
million people have already become HIV-positive in India and half a million
Chinese are now infected.
An explosive AIDS epidemic in the US is unlikely. Instead, HIV infection will still
continue in about 0.5% of the population. But the face of the epidemic will
change. New HIV infections will occur predominantly among the deprived, with
rates 10 times as high in minority groups. Nevertheless, American patients will
live quality lives for decades, thanks to advances in medical research. Dozens of
powerful and well-tolerated AIDS drugs will be developed to restore the immune
system.
A cure for AIDS in the next decade is not unthinkable. But due to economic
reality, this therapeutic progress will have only limited benefit outside the US and
Western Europe.
A vaccine is our only real hope to avert a disaster unparalleled in medical history,
and there have been promising results from experiments on animals. But even
when an AIDS vaccine gets developed, it will require an extraordinary effort of
political will to get it to the people who need it most.
Vocabulary check: Match the underlined words from the text with their
meanings:
1
2
3
4
5
6

plague
erode
life expectancy
impact
the deprived
avert

A
B
C
D
E
F

number of years sb is statistically expected to live


avoid, prevent
those who dont have essential things in life
weaken, damage
effect
disease that spreads fast and kills many people

Now try to sum up the text in 3 sentences and write them down.

AUTOIMMUNE DISEASES SYSTEMIC LUPUS ERYTHEMATOSUS


Read about this autoimmune disease and explain the underlined medical
terms using general English.
The name lupus (wolf) has been used for this disease for centuries, because the
characteristic (1) erythematous rash over the cheeks and nose was thought to
resemble a wolf bite.
Lupus is believed to be an autoimmune disese, which means that the body
produces (2) antibodies against its own cells and tissues, causing inflammation
and injury. It can be diagnosed by the presence of certain antibodies in the blood
as well as characteristic (3) leukocytes called LE cells. (4) Leukopenia or
(5) thrombocytopenia are also sometimes present.
The patients are primarily women. The condition affects connective tissue; more
specifically, the collagen (a type of flexible protein) in bones, cartilage, tendons
and ligaments all over the body. Symptoms include (6) polyarthralgia, (7) pyrexia,
(8) nephritis, and (9) malaise. Periods of (10) remission and (11) relapse are
typical.
Treatment: (12) corticosteroids against inflammation and
(13) immunosuppressive drugs.

HYPERSENSITIVITY ALLERGIES
I Read about allergies, which are becoming more and more common.
As you are reading, mark the passages which talk about:
-

the definition of allergy and allergen


the definition of anaphylactic shock and its treatment
possible causes of allergies
treatments and prevention

ALLERGY - A MODERN EPIDEMIC?


An allergy is an overreaction of the immune system to a foreign substance in
which histamine, a chemical found in all body tissues, plays a key role. Almost
any food can trigger an allergy, although about 90% of allergic reactions are
caused by milk, eggs, peanuts, nuts, fish, shellfish, soy and wheat. It still hasnt
been fully explained why in some people these otherwise harmless substances
provoke the same reactions by which the body tries to get rid of dangerous
pathogens. The reactions range from sneezing, vomiting and inflammation to
asthma attacks. In extreme cases, the reaction involves virtually all organ
systems and proceeds to anaphylactic shock, a dramatic drop in blood pressure

accompanied by extreme respiratory distress that may be fatal if not treated


immediately.
Although allergies tend to run in families, i.e. can definitely have genetic causes,
genetics cannot explain the rapid rise in allergies over the past few decades.
So something must have changed in the environment. Air pollution is the first to
come to mind. Some studies have found a strong correlation between pollutants
exhaust fumes and cigarette smoke and the development of allergies.
Also, industrialization has brought about declines in infectious diseases and
close exposure to farm animals. The hygiene hypothesis states that these
trends may have contributed to the rise in allergies. The human immune system,
which evolved in a natural environment full of hostile bacteria and parasites, finds
itself with nothing to do in the antiseptic modern world and terefore sometimes
attacks harmless peanuts and shrimps.
Although numerous studies support this theory, the hygiene hypothesis hasnt
provided any concrete prescriptions yet. Meanwhile, researchers are developing
new drug therapies that go beyond epinephrine (for emergency treatment of
anaphylaxis) and the growing number of antihistamines.
Pediatricians have also begun taking allergies more seriously. They keep
stressing the importance of breast-feeding exclusively for at least six months.
According to the American Academy of Pediatrics, the later infants are exposed
to new foods, the better it is for food-allergy prevention.
II Now read the passage about the CORRELATION BETWEEN
ANTIBIOTICS AND ALLERGIES IN CHILDREN
and fill the gaps with these prepositions:
FOR

FROM

IN / WITHIN (2x)

OF (5x)

TO (5x)

According _____ recent research, children who receive antibiotics _____ the first
6 months ______ life have an increased risk ______ developing asthma and
allergies ____ pets, grass, and dust mites. Researchers investigated 448
children ____ birth ___ age 7 years, and found that those who were given at
least one antibiotic ____ the first 6 months were 1.5 times more likely to develop
allergies than were those who didnt receive antibiotics. However, infants were
less susceptible ____ these effects if they lived with at least two dogs or cats
during the first year ____ life. Maternal history ____ allergy also added ____ the
risk _____ allergy ____ children taking antibiotics.
On the basis of this article, what would be your advice regarding antibiotics
in the first 6 months of life?

III CASE REPORT: ANAPHYLACTIC SHOCK


Provide the missing medical words and expressions from their definitions
below:
History and symptoms: A 6-year-old boy went near a wasp nest and was stung
several times in the face, arms and legs. Within minutes he became very pale
and weak, complained of chest tightness and (1) _______________. He was
immediately rushed to the ER of the local hospital.
Physical examination: At presentation, the boys breathing was shallow and
extremely labored and he was (2) _______________. Physical examination
revealed marked (3) _______________ (BP = 90/70) with a rapid, weak pulse
that was difficult to (4) _______________. The boys skin was clammy and cool
and he appeared dazed and confused. (5) _______________ of the chest
revealed (6) _______________ (120 beats/ min) and harsh bronchial sounds
including squeaks and wheezes.
Diagnostic tests: The acute presentation of allergy is easily diagnosed by the
presence of classic symptoms and signs. No specific diagnostic tests were
performed.
Diagnosis: Type I hypersensitivity reaction with (7) _______________
_______________ and (8) _______________ _______________.
Treatment: The boy was immediately given a (9) _______________ injection of
epinephrine, which was repeated after 20 minutes. Ventolin was
(10) _______________ as an inhalant to relieve his difficult breathing.
1
2
3
4
5
6
7
8
9
10

difficult breathing
with bluish skin and mucous membranes, due to hypoxia
low blood pressure
feel by touching
exam by listening
rapid heartbeat
failure of circulation
sudden contraction of the smooth muscle of the bronchi
pertaining to under the skin
given

NERVOUS SYSTEM
HOW DIFFERENT ARE MALE AND FEMALE BRAINS?
For decades, neurophysiologists have been trying to identify the physiological
basis of the obvious behavior differences in men and women. The key questions:
Is the anatomy of the male brain different from that of the female brain, and how?
Do structural differences cause differences in the way the brains work?
Although preliminary evidence suggests some answers, there are no final
conclusions as yet. And even if there were, they still could not take into account
how big a role in the development of typically male or female behavior patterns is
played by environmental factors (family and society).
I Sort out the following statistically based differences in behavior and
general brain operations under male or female:
-

superior verbal fluency


do better in maths
babies more passive
better at categorizing objects of different shapes, sizes and colors
babies more aggressive
better at verbal tasks (e.g. guessing words from context)
better at spatial orientation (e.g. reading maps, negotiating mazes)
learn to read earlier
better at visualizing an object and then imagining it in rotation

male
______________________________
______________________________
______________________________
______________________________

female
______________________________
______________________________
______________________________
______________________________
______________________________

II Now look at the empirically established structural differences between


the male and female brains and provide the names of the missing
structures from their definitions:

sexually dimorphic nucleus, a region in the ______________ (structure that


regulates the release of hormones from the pituitary), has distinctive synaptic
patterns in each sex
women have more neurons in ____________ ____________ (lateral regions
of the cerebrum)

____________ ____________ (band of nerve fibers connecting the two


halves of the brain) wider in women, particularly in posterior regions
better connection between the ______________ (halves of the brain) in
women
clusters of ______________ (conducting cells of the nervous system) in the
____________ ____________ (part of the CNS running along the back)
which serve the external genitalia are larger in men

DID YOU KNOW THAT...?


Work in pairs. Read the following items and take turns to tell each other
what you have read:

at birth the brain is already of its final size, while the rest of the body is
only 1/20?

the brains of the Neanderthals were actually larger than ours in total, but that
the parts of the brain responsible for abstract thinking are larger in modern
humans?

much of what is known about cerebral function comes from observing


dysfunctions that occur when specific regions of the cerebrum are
traumatized?

the brain of a newborn is as sensitive to excessive oxygen as it is to oxygen


deprivation?
(Before doctors realized the dangers of oxygen toxicity, premature babies
were sometimes treated with oxygen, which resulted in retinal damage and
blindness.)

Aleksandra mega Horvat: MEDICAL ENGLISH WORKBOOK

MUSCULOSKELETAL SYSTEM
OSTEOPOROSIS
Work in pairs. One of you should look at version A of the text on
osteoporosis on this page, the other at version B on the next page. To get
the missing information, you will have to put questions to your partner, and
answer his/ her questions with the information from your text.
Write your questions below the text (the first one has been done for you):
Version A
Osteoporosis is a condition in which bones lose their normal strength and
become (1) ____________. It is caused by (2) ____________. During this
process, there is a continuous cycle of breakdown and repair which takes place
in (3) ____________. Special cells called (4) ____________ eat away or resorb
areas of old, damaged bone. Other cells called osteoblasts then fill in the empty
spaces with new bone. Osteoporosis can be caused either by (5) ____________
or ____________. The result is a bone that is weaker than normal and may
break with a minor injury, or even spontaneously. Two of the most important risk
factors for osteoporosis are (6) ____________ and ____________.
The most dramatic bone loss (about 3 5% per year) occurs at the time of
menopause. The most common bone fractures that occur in women with
osteoporosis are fractures of the hip, vertebrae and wrist. Prevention is the best
approach to osteoporosis. When bone loss has occurred, even the best
treatments available are not enough to restore normal bone density. Some
medications can improve it by 10% and possibly more, but will not make the
bones as strong as they would be if preventive steps had been taken from the
beginning. The key features of osteoporosis prevention are getting enough
calcium and vitamin D, regular exercise, and, for some women, hormone
replacement therapy after menopause. It is also very important to stop smoking,
since it increases the risk of osteoporosis significantly.
1.
2.
3.
4.
5.
6.

What do bones become like in osteoporosis?


____________________________________________________________
____________________________________________________________
____________________________________________________________
____________________________________________________________
____________________________________________________________

Version B
Osteoporosis is a condition in which bones lose their normal strength and
become porous and weak. It is caused by an imbalance in the process called
remodelling. During this process, there is a continuous cycle of breakdown and
repair which takes place in all bones throughout life. Special cells called
osteoclasts eat away or resorb areas of old, damaged bone. Other cells called
osteoblasts then fill in the empty spaces with new bone. Osteoporosis can be
caused either by too much resorption of old bone, or too little formation of new
bone.The result is a bone that is weaker than normal and may break with a minor
injury, or even spontaneously. Two of the most important risk factors for
osteoporosis are female sex and increasing age.
The most dramatic bone loss (about 3 5% per year) occurs at the time of
(1) ____________. The most common bone fractures that occur in women with
osteoporosis are fractures of the (2) ____________, ___________ and
____________. (3) ____________ is the best approach to osteoporosis. When
bone loss has occurred, even the best treatments available are not enough to
restore normal bone density. Some medications can (4) ____________, but will
not make the bones as strong as they would be if preventive steps had been
taken from the beginning. The key features of osteoporosis prevention are
(5) ____________, ____________ and, for some women, ____________. It is
also very important to (6) ____________, since it increases the risk of
osteoporosis significantly.
1.
2.
3.
4.
5.
6.

When does the most dramatic bone loss occur?


____________________________________________________________
____________________________________________________________
____________________________________________________________
____________________________________________________________
____________________________________________________________

ACETABULAR RECONSTRUCTION
I Read about the most complicated kind of orthopedic surgery and mark
the passages which answer the following questions:
1.
2.
3.
4.
5.

What does the procedure of acetabular reconstruction involve?


Why does it have to be done within 2 millimeters of perfection?
Why do some patients develop arthritis after this operation?
What about the timing and the duration of the operation?
In planning the operation, as well as in assessing the postoperative outcome,
what is the relation of X-rays and CT scans?

6. How long does the postoperative period usually take and what does it
involve?
Acetabular reconstruction in patients whose hips have been shattered in an
accident are considered the most complicated, technically demanding kind of
orthopedic trauma surgery. To access the patient's shattered hip socket, the
medical team composed of surgeons, anesthesiologists, skilled nurses and Xray technicians has to avoid critical arteries, veins and nerves. Any mistake
could cause serious bleeding or nerve injury that might mean permanent
weakness or paralysis. When they reach the socket, surgeons must move in from
behind and piece it together like a jigsaw puzzle. Then they fix it in place,
screwing in plates up to eight inches long that remain in the body.
It is critically important to get this reconstruction just right, ideally within two
millimeters of perfection. Any displacement in the articular surface the place
where the ball and socket meet or articulate will expose the injured cartilage to
stresses that will eventually lead to its deterioration. And there are other potential
complications as well. The surgeons must make sure the screws dont enter the
joint space, because for the patient that would feel like walking with a pebble in
his shoe.
Still, some patients will nevertheless develop arthritis later just because of the
trauma to their cartilage.
Performing this type of surgery requires enormous stamina and concentration,
since operations may be up to 12 hours long. One advantage is the timing. Unlike
some surgical procedures that must be done immediately, acetabular fractures
can often wait for several days, or even a few weeks.
In a procedure like this, with so many potential complications, planning is crucial.
But X-rays alone may not be enough since bones block good views of the
acetabulum; it is also hard to roll an injured, hurting patient into position for the
necessary images. So CT scans are usually added to the planning mix. CT
scans have also proved to be more accurate in assessing patients postoperative
outcome.
During the postoperative recovery period which often lasts nine months or more
the team carefully monitor each patient's progress. They use video cameras
and other tools to measure stride length, walking speed and body angles, and
compare that with data on people who have not been injured.
They also assess muscle strength in various ways. In studying the results, they
have found that a good outcome seems to correlate with strength in particular
muscles around the hip.
What is the English term for acetabulum (paragraph 1)?
__________________

Put the various stages of the procedure in sequence and then tell your
partner about it:
A
B
C
D
E
F

fixing the acetabulum in place with plates


avoiding critical arteries, veins and nerves
monitoring the patients postoperative progress
planning the procedure with the help of X-ray and CT-imaging
putting the acetabulum together like a jigsaw
accessing the patients shattered acetabulum

II Complete the table with other forms of the words from the text:
NOUN

VERB
access
add
assess

ADJECTIVE
_

displacement
injury
length
perform
strength
weakness

necessary
_

ARTHRITIS
Work in pairs. Take turns to read about the four forms of arthritis to your
partner, who should simultaneously fill in the table on the next page:
Ankylosing spondylitis
Ankylosing spondylitis is a chronic inflammation and stiffening of the vertebrae. It
is characterized by bilateral sclerosos of the sacroiliac joints. The changes that
occur in the joints are similar to those found in rheumatoid arthritis. The disease
is treated with corticosteroids and anti-inflammatory drugs.

Gouty arthritis
Gouty arthritis is a metabolic disease caused by accumulation of uric acid
crystals in the blood, joints and soft tissues near them, which damage the
articular cartilage and synovial membrane. It can involve any joint, although it
typically affects the big toe, which is usually very painful. Treatment consists of
drugs which lower the production of uric acid, as well as anti-inflammatory drugs.
Besides, a special diet to avoid foods rich in uric acid is recommended.
Osteoarthritis (OA)
Osteoarthritis, also called degenerative joint disease (DJD), is the most common
type of connective tissue disease. The typical signs are destruction of articular
cartilage and new bone formation (hypertrophy of bone, or formation of bone
spurs) at the edges of joints. It mainly occurs in the hips and knees of older
patients. Aspirin and other analgesics and anti-inflammatory drugs are used in
treatment, as well as physical therapy to exercise the stiff joints. In the case of
complete immobilization of a joint in end-stage osteoarthritis, joint replacement
surgery is performed.
Rheumatoid arthritis (RA)
Rheumatoid arthritis is a chronic inflammatory disease of joints and their related
structures characterized by painful and crippling deformities. Usually the small
joints of the hands and feet are affected first, while the larger joints follow later. It
is believed to be caused by an autoimmune reaction of joint tissue. It is most
common in women. The painful and swollen joints, often accompanied by fever,
can become completely immobile. As there is no cure for RA, it is treated by heat
applications, drugs to reduce inflammation and pain, as well as antirheumatic
drugs.

Ankylosing
spondylitis

Gouty
arthritis

Osteoarthritis

Rheumatoid
arthritis

Which
structures
does it
affect?
Whom does
it affect?
Symptoms

Causes

Treatment

Translate:
Primjenom Exogena 2000, ultrazvune sprave koja koristi neinvazivni pulzirajui
ultrazvuk niskog intenziteta, uspjeno je zacijeljeno 86% nezglobnih prijeloma
kod kojih su prethodna lijeenja bila bezuspjena.
Posebice se istie vanost lijeenja prijeloma u starijih osoba. U SAD se,
primjerice, vie od polovine od ukupno 900.000 prijeloma godinje zabiljei kod
starijih od 65 godina.

CASE REPORT: MUSCULAR DYSTROPHY


Explain the underlined medical expressions from the first part of the case
study and supply the appropriate medical terms in the second part from the
definitions below:
Physical examination: On examination, the patient appeared fatigued. Vital
signs: HR = 104, RR = 28, temperature (oral) = 102.4 degrees F, BP = 138 / 74.
Pupils were normal and reactive to light, with (1) ocular movements intact. No (2)
dysphasia or facial muscle weakness was noted. (3) Percussion of the thorax
suggested (4) pulmonary infiltrates bilaterally in the lower lung fields. The patient
complained of occasional (5) dyspnea. Heart sounds were normal, with no
murmur. Bowel sounds were normal.
Musculoskeletal exam revealed a scoliotic deformity of the spine when the
patient sat up. (6) ______________, (7) _______________ and
(8)_______________ muscles were significantly (9) _______________, but the
(10) ________________ muscles appeared enlarged. Bilateral elbow
contractures limited elbow extension to 80 degrees. Bilateral (11)
_______________ contractures were also present. Muscle strength was reduced
in the (12) ____________ muscles bilaterally, and he was not able to move either
thigh into (13) ___________ or ____________.
1
2
3
4
5
6
7
8
9
10
11
12
13

_____________________________________________________
_____________________________________________________
_____________________________________________________
_____________________________________________________
_____________________________________________________
chest muscles
shoulder muscles
thigh muscles
diminished in size, wasted away
calf muscles
heel tendon
upper arm muscles
bent or stretched out position

INTEGUMENTARY SYSTEM
I Read about the integumentary system and provide the correct form of
the words in brackets:
The skin is a dynamic interface between the body and the external environment.
It is considered an organ because it consists of various tissues (1) ___________
(STRUCTURE) arranged to function together. Together with its (2) ___________
(ACCESS) organs (hair, nails, sudoriferous and sebaceous glands) it constitutes
an independent body system with a variety of functions, ranging from mechanical
and chemical protection against external influences and pathogens to protection
against (3) ____________ (LOSE) of fluids, thermoregulation and receiving
sensations.
Covering a surface of 2 square meters on average, the skin is the largest organ
in surface area. Its (4) ____________ (THICK) varies from 6 mm on the soles
and palms to only 0.5 mm on the tympanic membrane in the ear.
The skin is (5) ____________ (CLINIC) important because on the basis of its
(6) ____________ (APPEAR) conclusions can be made about certain conditions
and dysfunctions. For instance, pale skin may indicate shock, whereas erythema
and warm skin may indicate fever or infection. Cyanosis occurs when there is a
lack of oxygen, which is (7) ____________ (SYMPTOM) of some cardiovascular
or respiratory diseases; if there is (8) ____________ (EXCESS) bile pigment in
the blood (hyperbilirubinemia), the skin turns yellowish (jaundice). An exanthem
may suggest allergies or local infections. Abnormal skin texture may be the result
of (9) ____________ (NUTRITION) problems.
Even the state of a persons nails can be a source of information for a physician.
Abnormalities in texture and coloring usually indicate dietary or (10) __________
(GLAND) dysfunctions, while chewed nails may suggest emotional problems.
II Spot the medical words for the following structures and conditions in
the text:
1.
2.
3.
4.
5.

sweat glands
oil glands
disease-producing organism
eardrum
abnormality of function

6.
7.
8.
9.
10.

red, flushed skin


bluish shade of the skin
pertaining to the heart and vessels
pertaining to breathing
rash

WHAT IS IT IN ENGLISH?
Match the medical terms for skin conditions and symptoms with their
general English names:
1
2
3
4
5
6
7
8
9
10
11
12
13
14
15
16
17

ALOPECIA
CALLUS
COMEDO
CONTUSION
DECUBITUS ULCER
ERYTHEMA
EXANTHEM
KELOID
LESION
MACULE
NEVUS
PAPULE
PRURITUS
SEBORRHEIC DERMATITIS
URTICARIA
VERRUCA
VESICLE

A damage
B corn
C hives
D pimple
E redness of the skin
F blister
G thickened scar
H wart
I baldness
J mole
K bedsore
L blackhead
M bruise
N rash
O freckle
P itching
R dandruff

MALIGNANT MELANOMA
How does melanoma differ from an ordinary mole? Check for the ABCDs of
melanoma:
ASYMMETRY - one half is unlike the other
BORDER

- irregular

COLOR

- varied (combinations of brown, black, white, red, blue)

DIAMETER

- larger than 6 mm

Work in pairs. One of you should look at version A of the text on malignant
melanoma, the other at version B on the next page. To get the missing
information, you will have to put questions to your partner, and answer his
/ her questions with the information from your text.
Write your questions below the text (the first one has been done for you):

Version A
Melanoma develops when pigment-producing skin cells called (1) ___________
turn cancerous, a process that can be triggered by (2) ____________, especially
severe blistering sunburns during childhood. There is also evidence that
ultraviolet radiation used in indoor tanning equipment may cause melanoma.
If (3) ____________, the chance of recovery is over 95 percent. However, if the
tumor is allowed time to grow vertically down into the skin, reaching blood and
lymphatic vessels, it will easily (4) ____________, most typically the lungs or
brain. In that case, the five-year survival rates are (5) ____________.
Treatment: The first step is to surgically remove the cancerous lesion
(melanoma in situ) and a small patch of healthy skin surrounding it (a procedure
called taking margins). To determine if the melanoma has spread to the adjacent
lymph nodes, a technique called sentinel node biopsy is usually performed. In
metastatic cases, the affected nodes are excised. But if melanoma cells have
spread beyond the lymphatic system, prognosis is poor.
Risk factors: Light-colored skin and hair, freckles, moles. People with more than
50 moles (especially atypical/dysplastic moles) are at an increased risk (for the
appearance of the moles, see the ABCDs of melanoma above). Melanoma also
runs in families, so if you have a blood relative who has had it, you are at a
greater risk yourself.
Prevention: Early detection is key. Therefore, perform a monthly self-exam,
looking for irregular lesions that are growing and changing, and use the ABCD
rules. Melanoma can occur anywhere on the skin, even in places not directly
exposed to the sun, as well as in the oral cavity or the eye, but it is most common
on the backs of men and the legs of women. If you have an atypical mole, see a
dermatologist immediately and insist on a biopsy. In any case, visit a
dermatologist at least once a year for a complete skin exam.
1.
2.
3.
4.
5.

What are pigment-producing skin cells called?


__________________________________________________________
__________________________________________________________
__________________________________________________________
__________________________________________________________

Version B
Melanoma develops when pigment-producing skin cells called melanocytes turn
cancerous, a process that can be triggered by excessive sun exposure,

especially severe blistering sunburns during childhood. There is also evidence


that ultraviolet radiation used in indoor tanning equipment may cause melanoma.
If the cancer is removed before it penetrates a full millimeter into the skin, the
chance of recovery is over 95 percent. However, if the tumor is allowed time to
grow vertically down into the skin, reaching blood and lymphatic vessels, it will
easily metastasize into tumors in other body tissues, most typically the lungs or
brain. In that case, the five-year survival rates are much smaller and drop
dramatically to below 50 percent.
Treatment: The first step is to (1) ____________ (melanoma in situ) and a small
patch of healthy skin surrounding it (a procedure called taking margins). To
determine if the melanoma has spread to the adjacent lymph nodes, a
(2) ____________ is usually performed. In metastatic cases, the affected nodes
are excised. But if (3) ____________, prognosis is poor.
Risk factors: Light-colored skin and hair, freckles, moles. People with more than
50 moles (especially atypical/dysplastic moles) are at an increased risk (for the
appearance of the moles, see the ABCDs of melanoma above). Melanoma also
runs in families, so if you have a blood relative who has had
it,(4) ____________.
Prevention: Early detection is key. Therefore, perform a monthly self-exam,
looking for irregular lesions that are growing and changing, and use the ABCD
rules. Melanoma can occur anywhere on the skin , even in places not directly
exposed to the sun, as well as in the oral cavity or the eye, but it is most
common on (5) ____________. If you have an atypical mole, see a
dermatologist immediately and insist on a biopsy. In any case, visit a
dermatologist at least once a year for a complete skin exam.
1.
2.
3.
4.
5.

What is the first step in treating melanoma?


__________________________________________________________
__________________________________________________________
__________________________________________________________
__________________________________________________________

BURNS
Burns are heat injuries caused by thermal, chemical, electrical or radioactive
agents. They occur on the skin, but can also involve the respiratory or digestive
tract lining. They can have a local, or, much more seriously, systemic effect.
Burns are classified into 3 categories according to how deeply the tissue has
been damaged.

Sort out the jumbled notes about the various degrees of burns under the
following headings:
-

symptoms: whitish appearance, also deep red, brown, black, charred


usually no blisters
also called full-thickness burn injury
affects epidermal and dermal layer
also called superficial burns
skin grafts (dermoplasty) used to assist recovery
example: sunburn
symptoms: erythema, hyperesthesia, blistering, edema; moist
affect the epidermal layer
destroy epidermis and dermis, damage subcutaneous layer, sometimes even
underlying muscular and bone structures
also called partial thickness burn injury
initially nerve sensation lost, but later severe pain
symptoms: erythema, hyperesthesia; dry

1st degree burns

2nd degree burns

3rd degree burns

CASE REPORT: ACNE


Explain the underlined medical expressions by using general English
terms:
The patient presented with (1) multiple acneform lesions in the (2) buccal and
periorbital areas. Both cystic and (3) comedonal lesions were present, most
ranged between 3 and 6 mm in diameter, especially along the (4) nasolabial
folds. Some were (5) edematous and (6) erythematous. There were also
developing areas of (7) folliculitis on the patients chest and forearms.
A (8) topical antibiotic was prescribed, and a serum biochemical and hematologic
profile was ordered.
1 ________________________________________________
2 ________________________________________________
3 ________________________________________________
4 _________________________________________________
5 ________________________________________________
6 ________________________________________________
7 _______________________________________________
8 ________________________________________________

SKIN NEWS
Work in groups of 3. Read the texts and suggest headlines for them.
Compare the suggested headlines and take turns to tell your group about
one item each. Then use the underlined collocations in sentences of your
own and write them down.
_________________________
According to the British Journal of Dermatology, a new light treatment has shown
promising results in the treatment of mild to moderate acne. In clinical trials, a
combination of red and blue light treatment led to an average improvement of
76% in the number of spots after 12 weeks. Dermatologists believe that the blue
light kills off bacteria, while the red light helps the skin to heal.
_________________________
To refresh the appearance of skin, chemical peels have been common for some
time now, but laser resurfacing is becoming more and more popular. In this
procedure, outer layers of skin are vaporized, diminishing wrinkles, spots and
scars and sometimes tightening skin as well.
However, too aggressive treatments can result in burns, scarring and permanent
changes in skin color. It may also trigger dormant herpes infections and allergies,
so caution is advised.
_________________________
Dermatologists know that inactive hair follicles arent dead, but just catatonic; a
search for an agent that would activate them again seems logical.
Experiments on mice appear promising. Scientists know that a gene named SHH
controls brain development in embryos, but in mature animals, as well as in
humans, it governs natural on-off cycles of hair growth. When researchers
inserted SHH into mouse hair follicles, the dozing follicles woke up and started
growing again.
Can SHH wake human cells in the off position? Dermatologists wont know until
they try. However, theyll have to consider side effects it is known that SHH in
big doses can trigger skin cancer. Still, the research suggests that the new hope
genetics is bringing to victims of cancer and other devastating diseases may also
cover the bald.

RESPIRATORY SYSTEM
I Match the sentence halves to get a description of the process of
RESPIRATION:
1 Respiration, the process of gas exchange, is conducted
2 The first part of the process is called external respiration (breathing or
ventilation)
3 During external respiration oxygen passes from the inhaled air into the lungs
4 At the same time, carbon dioxide passes
5 During internal respiration the cells within all body structures take oxygen from
the blood
A from the capillaries back to the lungs to be exhaled.
B while the second part is referred to as internal respiration (cellular or tissue
breathing).
C and give back carbon dioxide, which is a waste product from the energyliberating process that takes place within each cell.
D at various levels between the environment and the body, as well as within the
body.
E and then to the blood through capillaries by a process called diffusion.
II Match some of the structures of the respiratory system with their
functions:
1
2
3
4
5
6
7
8
9

NOSE
PHARYNX
LARYNX
EPIGLOTTIS
TRACHEA
BRONCHI
ALVEOLI
DIAPHRAGM
VOCAL CORDS

A
B
C
D
E
F
G
H
I

place of gas exchange between air and blood


cartilaginous flap that prevents food to enter the trachea
conduct air from the trachea to the lungs
acts as a filter to warm and moisten the inhaled air
air passageway and place of voice production
vibrate to produce sound
separates the thoracic from the abdominal cavity
food and air passageway
air passageway

DID YOU KNOW THAT...?


Work in pairs. Read the following items and take turns to tell each other
what you have read.

the internal surface of each lung is more than 40 times greater than the
external surface of the entire body?

compared to a toy baloon, a lung is about 100 times more distensible?

it is impossible for a person to commit suicide by holding their breath,


because the rising CO2 levels in the blood are such a powerful respiratory
stimulant that, when they are high enough, breathing will start automatically,
whether we want it or not?

it was reported that over 300 prescription drugs, ranging from antidepressants
and beta-blockers to headache pills, could lead to breathing difficulties and
permanent lung damage?

in Eastern Europe and central Asia tuberculosis is still a major public health
issue, not only something associated with drug addicts, prisoners, or the
homeless, and that people in those areas are 10 times more likely to be
infected with multidrug-rasistant TB than in the rest of the world?

Translate:
Tuberkuloza je uzrok 2 milijuna smrti godinje irom svijeta. Lijeenje zahtijeva
striktni estomjeseni reim koji ukljuuje 4 razliita antibiotika. Bolesnici esto
prekidaju lijeenje nakon 2 ili 3 mjeseca, nakon to im se stanje popravi, time
zapravo ubrzavajui razvoj sojeva otpornih na lijekove.
FIGHT THE COLD
I Read about the chances of finding a vaccine against the common cold
and then decide if the statements below are true or false, marking the
lines which helped you decide:
Colds are considered incurable today because it would take months to come up
with a vaccine for every new strain. Colds viruses mutate even while they are
infecting you, and new strains appear so often that by the time drugmakers
create a vaccine against one variation, the serum is already outdated.
However, research on flu viruses, which only mutate every few years, may point
the way toward a cold cure. Researchers have isolated a protein called M2 that
seems to be present in virtually every flu strain known to man. Using that
knowledge, they have made a vaccine that they think could protect against all
flus - old, new and those not yet in existence.
If a similar protein is found in cold viruses a protein present no matter what
strain is involved then it is possible that in the next decade children could be
routinely vaccinated against the common cold.

1.
2.
3.
4.

Flu viruses mutate faster than cold viruses.


It is impossible to produce a vaccine against any strain of the common cold.
A protein present in all known flu strains has been found.
Researchers assume that the new vaccine involving M2 will not be effective
against future flu strains.
5. Scientists have isolated a protein present in all strains of the common cold.
II Complete the text with appropriate words:
WORKING OUT WITH A COLD?
When you get your first cold of the year, should you exercise extra hard to
_____________ your immunity, shift your workout to a lower intensity, or rest
until you feel better? Take this advice from fitness professionals:
Check your symptoms. Moderate exercise is safe if you have above-the-neck
signs, such as a _____________ nose, sneezing or a _____________ throat. If
you have below-the-neck signs, such as chills, diarrhea, or muscle ___________,
its time to rest.
Dont exercise with a temperature. This is evidence you are _____________
with a virus, and exercising would _____________ your risk of dehydration.
Modify exercise intensity. If you exercise hard, youll reduce your bodys ability
to _____________ infection. During the workout, make sure your ____________
rate and body _____________ do not increase excessively.
Dont let illness stop you altogether. Use your downtime to focus on flexibility
and stress management. It could be a good time to try meditation, yoga or
Pilates.
RESPIRATORY SYMPTOMS AND PROCEDURES
Match the symptoms and procedures on the left with their definitions:
1
2
3
4
5
6
7

AUSCULTATION
CREPITATION
EPISTAXIS
EXPECTORATION
HEMOPTYSIS
HEMOTHORAX
HYDROTHORAX

8 PAROXYSM

A
B
C
D
E
F
G

pleural effusion containing serous fluid


expectoration of blood or blood-stained sputum
accumulation of air or gas in the pleural space
presence of fluid in the pleural space
spasm / seizure
harsh, high-pitched breath sound
coughing up and spitting out matter from lungs,
bronchi or larynx
H inhalatory sound like that made by throwing salt into
fire

9 PERCUSSION
10 PLEURAL EFFUSION
11 PNEUMOTHORAX
12 PYOTHORAX
13 SPUTUM
14 STRIDOR
15 WHEEZE

I pleural effusion containing pus


J listening for sounds within the body
K high-pitched whistling breath sound
L mucus ejected from lungs, bronchi or larynx
M nosebleed
N short, sharp taps on body parts for diagnostic
purposes
O pleural effusion containing blood

ABDOMINAL THRUST MANEUVER


Work in pairs. Each of you should read one description and then describe
the procedure to the other:
If a person is choking on food that got lodged in their trachea, a simple technique
called the abdominal thrust or Heimlich maneuver can save their life. Just do
the following:
If the victim is standing or sitting:
-

stand behind the victim and put your arms around their waist
make a fist with one hand, grasp it with the other, and place the fist on the
victims abdomen above the navel and below the rib cage
press your fist into the victims abdomen with a quick upward thrust
if necessary, repeat several times

If the victim is lying:


-

put the victim on their back


sit astride their hips
place your hands one on top of the other on the victims abdomen above the
navel and below the rib cage
press into the victims abdomen with a quick upward thrust
repeat several times if necessary

If you are alone and choking, do whatever you can to apply force just below
your diaphragm. Press into a table, or use your own fist.

SMOKING
I Get into groups of 4-8, half smoker, half nonsmoker.
Smokers discuss: How much do you smoke? On which occasions? When and
how did you start? Would you like to stop? Why? Have you ever tried quitting?
Do your parents smoke? How do they feel about your smoking? What would you
do if your boy/girlfriend was a non-smoker and asked you to stop?
Nonsmokers discuss: Have you ever smoked? If yes, why and how did you
stop? How do you feel about so many of your smoker friends and colleagues?
Have you ever tried to persuade any of them to stop? Do your parents smoke? If
yes, does it bother you? Can you imagine having a boy/girlfriend who smokes?
Both discuss: How do you feel about the strong anti-smoking trend that
started in the US and is now spreading in EU countries, some of whom have put
a ban on all smoking in public places? What about attempts like the anti-smoking
campaign started by the Andrija tampar School of Public Health in Zagreb a few
years ago?
Should students of medicine and doctors display a greater readiness to be an
example in this matter?
II The debate on decriminalizing the possession and use of marijuana has
repeatedly been raised in various European parliaments, including the
Croatian. What is your standpoint in this matter? Discuss in small
groups.
How many colloquial expressions for marijuana can you think of?
Read the short interview with a neuroscientist and say whether you
agree.
Explain the play on words in the title:
A DOPEY IDEA
Q. How can marijuana blow the mind?
A. Drugs interfere with the careful interplay of chemical and electrical impulses
between cells, throwing out the balance of the brain either by causing too many
chemicals to be released or by stopping the cells' chemical transmitters reaching
the vital receptors. The reason marijuana is so potent is that is has its own
receptor in the brain. The more you smoke, the less sensitive the receptor
becomes.

Q. How is a person's behavior affected?


A. Research shows that the drug leads to impaired memory and coordination.
These effects may be long-term and irreversible. There is strong evidence that
marijuana can trigger schizophrenia. Then there's demotivational syndrome, an
inability to focus on anything beyond the next fix.
Q. Is it addictive?
A. That depends how we define addiction. Cannabis users have to take everlarger quantities to achieve the desired effect. Studies show that about 10% of
users can't stop their habit, despite wanting to do so.
PLUS...
Translate one of the following items:

Smoking not only causes bad breath, but also discolors teeth and fillings,
diminishes taste and smell, increases the risk of gum disease, and
consequently of tooth loss and failure of dental implants, as well as the risk of
mouth cancer.

Recent research suggests that the reason why some people are hopelessly
addicted to cigarettes may be the fact that nicotine triggers a pattern of
activity in their brains that makes quitting impossible for them. Also, this
strong neurobiological reaction to nicotine seems to be associated with hostile
personalities marked by anger, aggression and anxiety.

Scientists believe to have found a physiological reason why nicotine and


alcohol so often go together- it seems that even a small amount of alcohol
significantly boosts the pleasurable effects of nicotine. That could explain the
statistics according to which more than 80% of alcoholics smoke, and
alcoholism is 10 times as prevalent among smokers as among nonsmokers.

CASE REPORT: ASTHMA


Explain the underlined medical terms using general English:
The patient is a 12-year-old girl who has had a nocturnal nonproductive cough
two to three times per month for the past 3 months associated with increasing
episodes of shortness of breath that resolve spontaneously.
Past medical history: The patient has had recurrent upper respiratory
infections,

bronchitis 2 years ago, and intermittent runny nose. A review of systems reveals
that she has had numerous episodes of sneezing, itchy eyes, and clear
discharge from the nose. The patient states that she does not smoke cigarettes
or any other inhaled substances such as marijuana.
Physical examination: Vital signs as follows: T = 98.6 degrees F, RR = 17, HR
= 82, BP = 118 / 75 mmHg. No (1) dyspnea or (2) stridor is evident. Her color is
normal, without (3) cyanosis. The (4) tympanic membranes are mobile and
without (5) erythema or air / fluid levels. Inspection of the chest does not show
accessory muscle use or (6) intercostal, (7) suprasternal, or (8) supraclavicular
retractions. (9) Pulmonary auscultation reveals (10) inspiratory and expiratory
wheezing scattered throughout both lung fields.
1 _____________________________________________________________
2 _____________________________________________________________
3 _____________________________________________________________
4 _____________________________________________________________
5 _____________________________________________________________
6 _____________________________________________________________
7 _____________________________________________________________
8 _____________________________________________________________
9 _____________________________________________________________
10 ____________________________________________________________

When does it happen?


nocturnal
recurrent
intermittent

__________________________
__________________________
__________________________

CASE REPORT: AVIAN INFLUENZA


Provide the missing medical terms from the definitions below:
Day 1: 13-year-old symptomatic female admitted to pediatric ward from
emergency department.
PMH: Previously healthy. Sore throat, (1) ____________, dry cough 1 week prior
to admission. Fever 4 days prior to admission.
Examination: Alert, (2) ____________, no respiratory distress. Lung auscultation:
decreased breath sounds and (3) ____________ in right lung base.
CBC: WCC 4700/microL, platelets 62000/microL
Blood culture: negative
Sputum culture: nil of note

Viral (4) ____________: nil of note


Diagnosis: atypical pneumonia
Day 2: (5) ____________. Increasing respiratory rate and distress; (6)
____________ despite oxygen therapy.
Admitted to ICU. Mechanical ventilation 6h after admission. Auscultation: audible
rub in middle lobe.
Days 3 5: Rapid deterioration with (7) ____________ and multiple organ
dysfunction.
Day 6: Upper gastrointestinal bleeding. Worsening respiration requires prone
position ventilation for 7 days.
Day 26: Died. Cause of death: (8) ____________ ____________ ___________.
1
2
3
4
5
6
7
8

discharge of nasal mucus


with raised body temperature
inhalatory sound like that made by throwing salt into fire
quantity of a substance required to react with a given amount of another
substance
expectoration of blood or blood-stained sputum
low oxygen supply to tissues
suddenly occurring signs of breathing difficulties (4-letter acronym)
incurable breakdown of respiration

NERVOUS SYSTEM
NEUROLOGICAL SYMPTOMS AND DISORDERS
I Sort out the following symptoms and disorders into the categories below
and say what you know about some of them:
ALZHEIMER
COMA
BRAIN TUMORS
HERPES ZOSTER
CEREBRAL CONCUSSION
HYDROCEPHALUS
CEREBRAL CONTUSION
MENINGITIS
CEREBROVASCULAR ACCIDENT

MULTIPLE SCLEROSIS
PARKINSON
SPINA BIFIDA
SYNCOPE

Symptoms
____________________
____________________
Disorders
congenital: _______________
degenerative: _______________
infectious:
_______________
neoplastic: _______________
traumatic:
_______________
vascular:
_______________

________________
________________ ________________
________________
________________
________________

II Work in pairs. Take turns to read about the 3 types of CVA to your
partner, who should simultaneously fill in the table on the next page:
1. Thrombotic CVA: Due to a thrombus (blood clot) in the cerebral arteries,
which causes an occlusion (blocking) of the vessel. The blood vessels
become narrower over time due to atherosclerosis (accumulation of fatty
plaques on the inner arterial walls). Before total occlusion, the patient
sometimes experiences short episodes of neurological dysfunction known as
TIAs (transient ischemic attacks).
Treatment: anticoagulant drug therapy, or surgical (endarterectomy removal
of plaque from the inner arterial lining).
2. Embolic CVA: Occurs when a blood clot breaks off from somewhere in the
body (embolus) and travels to the cerebral arteries occluding a small vessel.
Embolic CVAs are usually very sudden.
3. Hemorrhagic CVA: Caused by bleeding from a cerebral artery. The bleeding
results from advanced age, atherosclerosis or high BP, all of which cause
degeneration of blood vessels. Trauma to the head or the bursting of an

aneurysm can also be the cause of cerebral bleeding. If the bleeding is large,
the stroke is often fatal; if it is small, good recovery with or without
consequent disability is possible.
Risk factors for all CVAs: hypertension, diabetes, heart disease, smoking,
obesity, high cholesterol levels.
CEREBROVASCULAR ACCIDENTS
THROMBOTIC
EMBOLIC
HEMORRHAGIC
immediate
cause
underlying
cause
symptoms
prognosis
treatment
risk factors

III Translate:
Prema nedavno objavljenoj studiji danskih znanstvenika, sniavanje tjelesne
temperature za oko 1C u prvih nekoliko sati nakon modanog udara moe
smanjiti oteenja mozga i rizik od smrti.
ALZHEIMER DISEASE
II As you read about current research on Alzheimer disease, mark the
Lines which answer the following questions:
1. Which two aspects of dealing with Alzheimer can give us hope of a cure for
this disease in the future?
2. What happens in the brains of Alzheimer sufferers?
3. According to many scientists, what is the underlying cause of Alzheimer?
4. List the 5 pharmaceutical approaches to Alzheimer that are currently

investigated:

____________________________________________________________
____________________________________________________________
____________________________________________________________
____________________________________________________________
____________________________________________________________

NEW HOPE FOR LOST MEMORIES


Alzheimer disease has traditionally been diagnosed only post mortem. Now
technology is transforming our understanding of this neurological disease. Armed
with state-of-the-art PET scanners and MRI machines, specialists are learning to
spot and follow the disease in people who havent even developed the distinctive
symptoms yet.
The diagnostic revolution began during the 1990s, as researchers learned to
monitor neurons with an imaging technique called PET, or positron-emission
tomography. Unlike an X-ray or CT imaging, PET records brain activity by
focusing on glucose, and it can spot significant pathology in people who are still
functioning normally.
In its long run, Alzheimer replaces the brain's sophisticated tissues with sticky
plaque and dead, twisted neurons. No drug can repair that kind of damage as
yet, but fortunately the possibilities for therapy are changing almost as fast as the
diagnostic arts. And if the new treatments work as anticipated, they will be able to
stop or slow the destruction while the mind is still intact.
Though experts still cant agree about the ultimate cause of Alzheimer, many
believe that the trouble starts with a bit of junk protein called amyloid beta (A-beta
for short). Each of us produces it, and small amounts are harmless. But as Abeta builds up in the brain, it replaces healthy tissue with the plaques seen in
Alzheimer sufferers. In 1987 researchers discovered A-beta was part of a larger
molecule they called the amyloid-precursor protein (APP).
The ideal Alzheimer remedy would simply slow the production of A-beta by
disabling the enzymes (secretases) that produce it. Secretase inhibitors may be
our best hope of preventing Alzheimer, but they're not the only hope. Compounds
are now being tested that are supposed to keep A-beta from forming fibrils, and
others to keep fibrils from aggregating to create plaque. All of the experimental
drugs have helped reduce amyloid buildup in mice, suggesting they might help
people as well. But human studies are just now getting underway.
Scientists are also researching several strategies for clearing deposits from the
brain. One idea is to mobilize the immune system. It was shown that animals
given an antiamyloid vaccine started attacks on their plaques. Unfortunately, the
vaccine proved toxic in people, triggering attacks on normal tissue as well as

plaque. But the dream isn't dead. Some pharmaceutical companies are now
developing ready-made antibodies that, if successful, will target amyloid for
removal from the brain without triggering broader attacks by the immune system.
If even half of these treatments fulfill their promise, in the future old age may
prove more pleasant than today's projections suggest.
II Alzheimer, Parkinson and multiple sclerosis are examples of
DEGENERATIVE neurological diseases. Unfortunately, many other
negative adjectives apply in cases of such diseases. Make negatives of
the listed adjectives by adding the following prefixes:
AB
able
abled
aware
balanced
coherent
conscious
continent
controlled
curable
emotional
interested
known
mobile
normal
rational
treated
visible
voluntary

DIS

IM

IN

IR

MIS

UN

_____________________
_____________________
_____________________
_____________________
_____________________
_____________________
_____________________
_____________________
_____________________
_____________________
_____________________
_____________________
_____________________
_____________________
_____________________
_____________________
_____________________
_____________________

Now use some of the negative adjectives in sentences of your own.


CASE REPORT: MULTIPLE SCLEROSIS
Supply the missing medical terms from their definitions below:
A 38-year-old woman presented with tingling, numbness and clumsiness of both
hands for 1 week, with a band of numbness from the (1) ____________ to the
(2) ____________. Six months earlier, following an upper respiratory tract
infection, she had experienced (3) ____________ in the feet, numbness from the
waist downwards and burning pains behind the right ear. She was anxious
because her maternal grandmother had suffered from multiple sclerosis.

On neurological examination, she had absent (4) ___________ ___________


with brisk tendon jerks and (5) ____________ ____________ ____________
responses. Blood investigations were normal, including (6) ____________, whitecell count and differential, (7) ____________ ____________ ____________,
vitamin B12 and folate levels, and syphilis serology. A (8) _________ ________
was carried out. The (9) ____________ ____________ investigation results
allowed a likely clinical diagnosis of multiple sclerosis.
1 navel
2 armpits
3 abnormal touch sensation (burning, prickling) in the absence of an external
stimulant
4 contractions of the muscles of the area between the thorax and the pelvis
upon scratching this area
5 pertaining to the extension of the sole of the foot on both sides
6 oxygen-carrying pigment of erythrocytes
7 the rate at which red blood cells precipitate out from a specimen of venous
blood
8 withdrawal of fluid from the subarachnoid space in the region between L3-L4
9 watery fluid flowing throughout the brain and around
DYSLEXIA
Scan the text on some new findings concerning the neurological basis of
dyslexia and mark the parts which tell you:
-

what dyslexia is
how the brain reads
where the problem lies for dyslexics
the relation between dyslexia and intelligence
why reading certain languages (e.g. Italian) takes less effort than reading
others (e.g. English)

COMPLEX WRITING
English is notoriously illogical. For example, if tongue is pronounced tung, why
isn't argue pronounced arg? And if enough is enuff why isn't bough
pronounced buff? The arbitrary rules of English spelling and pronunciation have
always been a major problem for students of the language. But such frustrations
are small in comparison with those suffered by dyslexics, who live with a learning
disability that can make reading and writing almost impossible.
In the past two decades, scientists have learned a great deal about the
neurological causes of dyslexia. But what they hadn't yet explained is why its
incidence varies so from country to country. Now Italian, French and British

researchers suggest an answer. They offered the first convincing evidence that
the disorder has a common neurological basis across linguistic and cultural
borders.
The brain does not have an innate reading ability as it does for speech so it
deals with the written word by converting it into a familiar phonetic language.
According to prevailing theory, the reading centers of the brain break words down
into sound units known as phonemes and recognize them as the elements of a
phonetic code. Then the centers assemble that code to derive meaning from the
symbols on the page. Most of us learn to do this by the time we're seven years
old.
Dyslexics, however, often can't get past the first step breaking written words
down into phonemes. This in no way reflects on their intelligence there are
countless famous and successful dyslexics. Historical figures who may have had
the disorder include the poet W.B.Yeats and Leonardo da Vinci. Nevertheless, it
can be a lifelong challenge.
In their study, the scientists compared the reading ability of dyslexics from Britain,
France and Italy and found that Italian dyslexics read far better than their French
and English counterparts. Brain scans conducted during reading exercises
confirmed that the boundary between language and visual processing areas was
inactive in dyslexics, no matter what language they spoke. So why do Italian
dyslexics read better? The difference seems to be not in the languages
themselves, but in their writing systems, which vary in complexity.
English has 1,120 different ways of spelling its 40 phonemes. By contrast, Italian
needs only 33 combinations of letters to spell out its 25 phonemes. As a result,
reading Italian takes a lot less effort, and that's probably why the reported rate of
dyslexia in Italy is less than half of that in the US, where about 15% of the
population is affected to some degree. By some estimates, Americans spend
more than $1 billion a year to help their children cope with dyslexia.
Explaining this discrepancy isn't all that the study has accomplished. By
establishing a universal neurological basis for dyslexia, the scientists make it
clear that teachers should more often consider that children with reading
problems may be suffering from dyslexia, and treat them accordingly.
Based on this article, do you suppose that the percentage of dyslexic
children in Croatia is bigger or smaller than in the mentioned countries?
Why?

Vocabulary check match the words from the text with their meanings:

1
2
3
4
5
6
7

arbitrary
incidence
innate
converting
prevailing
assemble
discrepancy

A
B
C
D
E
F
G

current, present
difference
put together
random, not systematic
inborn
occurrence
changing

BIONICS NEW HOPE FOR THE PARALYZED?


bionics - science that studies the functions, characteristics and phenomena of
the living world and applies that knowledge to new devices and techniques in the
world of machines (biomechanics)
COMPUTER CHIPS FIGHT SENSORY LOSS
Researchers in Europe, America and Japan are implanting electrodes into the
bodies of patients to restore vision, treat brain disorders and help victims of
paralysis get back motor function, while engineers are creating hybrid prosthetic
body parts such as ankles, legs and knees in which silicon ships are melded with
living tissue, thus making our bodies bionic.
For one young man at least, technology has already become a part of his body.
Eight years ago he broke his neck in a motor accident, which made him a
tetraplegic. Except for some minor movement in his shoulders, right arm and
hand, he was paralyzed below the neck. He underwent an experimental surgical
procedure to implant a neural prosthesis an interface between an electronic
device and the human nervous system to bypass the damaged stretches of his
spinal cord and restore some movement of his limbs.
From the following description of how the device works, draw a scheme
using the listed words and phrases DOWNWARDS, HAND CLOSES,
MUSCLES CONTRACT, UPWARDS, POSITION SENSOR, MUSCLES RELAX,
HAND OPENS, MUSCLES IN RIGHT ARM AND HAND, MOVING LEFT
SHOULDER, CHEST STIMULATOR
To get back basic function to his right arm, the patient uses a device that restores
the ability to grasp, hold and release objects. During a seven-hour operation,
surgeons made incisions in his upper arm, forearm and chest. Eight flexible
electrodes, each about the size of a small coin, were attached to the muscles in
his arm and hand that control grasping. These electrodes were then connected
by ultrathin wires to a stimulator a kind of pacemaker for the nervous system

implanted in his chest. The stimulator was in turn linked to a position-sensing unit
attached to his left shoulder, over which he retains some motor control.
When he wants to pick up a glass, he moves his left shoulder upward. This
movement sends an electrical signal from the position sensor, which is worn
under his clothing, to the stimulator in his chest, which amplifies it and passes it
along to the appropriate muscles in his arm and hand. In response, the muscles
contract and his right hand closes. When he wants to release the glass, he
moves his left shoulder downward, and by the same process his right hand
opens.
This device is not for everyone, though. To benefit from it, patients must have
use of a shoulder and upper arm and partial use of their hands. Also, patients
must be constantly on guard against infection around the implanted electronics.
Another drawback is that the system provides no tactile feedback for things like
temperature, so users also have to be careful when handling hot objects such as
cigarettes or coffee.
To get around this problem, researchers are developing neural prosthetics that
can actually feel the texture of objects and transmit this information back to the
user.
This kind of sensitive prosthetic would stimulate afferent nerves to send tactile
information from paralyzed limbs to other parts of the body, where the sensations
could be perceived.
At present, all of this may seem like science fiction, but the numerous scientists
enthusiastically working all over the world will surely make the fantasies real,
maybe even sooner than we think.
II List the drawbacks and limitations of the described device.

Aleksandra mega Horvat: MEDICAL ENGLISH WORKBOOK

DIGESTIVE SYSTEM
I There are several processes involved in what we usually call DIGESTION.
Match their medical names with their description:
1 INGESTION
2 MASTICATION
3
4
5
6
7

DEGLUTITION
PERISTALSIS
DIGESTION
ABSORPTION
DEFECATION

A rhythmic contractions of digestive structures that


help the passage of food
B passage of nutrients from small intestine into
blood or lymph
C intake of food into mouth
D elimination of solid wastes from the body
E chewing of food
F chemical and mechanical breakdown of food
G swallowing of food

II WHAT IS IT IN ENGLISH?
Give the general English names for the digestive structures below:
ORAL CAVITY
HARD AND SOFT PALATE
GINGIVA
SALIVA
PHARYNX
ESOPHAGUS
GASTER
ILEUM
COLON
DUODENUM+JEJUNUM+ILEUM+
COLON+RECTUM

___________________________
___________________________
___________________________
___________________________
___________________________
___________________________
___________________________
___________________________
___________________________
___________________________

III Assign each of the following functions to one of the structures from
exercise II. Also, try to define the terms in italics:

chemically and mechanically breaks down chyme from stomach with help of
hepatic secretions and pancreatic juices; absorbs nutrients; transports
wastes to large intestine: ____________
transports bolus from pharynx to stomach: ____________
mechanically breaks down food mixing it with saliva; starts digestion of

carbohydrates: ____________
mixes bolus with gastric juice; starts digestion of proteins; moves chyme into
duodenum; when necessary, regurgitates food: ____________
absorbs water and electrolytes from undigested wastes; forms and expels
feces by defecation reflex: ____________
transports bolus from mouth to esophagus: ____________

DID YOU KNOW THAT...?


Work in groups of 4. Each of you should read one of the following items,
suggest a headline for it, and try to sum it up in one sentence. Then tell
your group about what you have read.
______________________________
According to various European and American studies, the worldwide trend
towards obesity is causing serious health concerns in the industrialized world.
Between 10% and 25% of people in the EU have been classified as obese, and
all Americans will be overweight by 2059 at the current rate of increase.
The medical consequences of this trend are well known: being overweight
significantly increases the risk of coronary artery disease, congestive heart
failure, hypertension, diabetes, depression, deep-vein thrombosis, osteoarthritis,
hip- and knee-replacement surgery to mention only the top of the list.
One more thing: it has been established that the most common cause of obesity
is not overeating, but underactivity.
______________________________
Orlistat, a weight loss drug that works in the intestine, decreases the users fat
absorption by about 30%, and, according to long-term studies, enabled more
than 50% of them to lose at least 5% of their body weight. But Orlistat users have
to take vitamins A, D, E, K and beta-carotene supplements, and tolerate sideeffects such as frequent bowel movements. Of course, the drug is not meant to
be a substitute for a healthy lifestyle a balanced diet and exercise still remain a
must!
____________________________________

Bad breath (halitosis), unpleasant as it may be, isn't an illness, but just a
symptom. In some cases, the underlying cause may be diabetes, some kidney
disorder or an infecion of the sinuses, bronchial tubes or gums.
Another cause is foods like onions or garlic, because essential oils from such
foods get into the blood, lungs and then out with each exhaled breath.

But the most common reason for bad breath are food molecules rotting in the
mouth. Mouthwash masks the smell, brushing and flossing remove larger
particles, but dentists suggest brushing the back of the tongue as well, where
food residues and bacteria congregate. The microscopic bits that remain must be
flushed down by drink or saliva (morning breath occurs because salivation shuts
down at night).
______________________________
Searching for cancers and other diseases by inserting endoscopes up the rectum
or down the throat can be painful, and some areas remain inaccessible. But the
wireless video pill, a new diagnostic device recently introduced in our country
too, allows patients to live their normal lives while it passes painlessly through
their digestive systems. For 24 hours it takes two pictures a second, sending
images to a small recorder worn at the waist; the data are downloaded to hospital
computers for analysis. However, the capsules won't make the endoscope
superflous, because endoscopes can perform biopsies and other procedures as
well. Besides, this new diagnostic method has its price: ca 600 euro per capsule.
GASTROINTESTINAL SYMPTOMS
I Gastrointestinal symptoms are important because they can signal a large
number of disorders, not only of the digestive system. Match the
symptoms with their definitions:
1 ACHLORHYDRIA
A abnormal frequency and liquidity of stools
2 ANOREXIA
B syndrome characterized by hyperbilirubinemia and deposition of bile pigment in the skin, mucosa and sclera, resulting in
yellow appearance of the patient
3 ASCITES
C bad breath
4 COLIC
D absence of hydrochloric acid from gastric
secretions
5 CONSTIPATION
E air / gas from the stomach passed through
the mouth
6 DIARRHEA
F effusion and accumulation of serous fluid
in the abdominal cavity
7 DYSPHAGIA
G unpleasant abdominal sensation often
culminating in vomiting
8 ERUCTATION
H difficulty in swallowing
9 FLATUS
I lack / loss of appetite
10 GASTROESOPHAGEAL
J heartburn
REFLUX

11 HALITOSIS
12 JAUNDICE
(ICTERUS)
13 MELENA
14 NAUSEA
15 PYROSIS
16 STEATORRHEA

K excessive amounts of fats in the feces


L gas / air in the GI tract or expelled
through the anus
M reflux of stomach and dudenal contents
into the esophagus
N infrequent / difficult evacuation of feces
O acute abdominal pain
P dark stools stained with blood pigments

II If chronic, GASTROESOPHAGEAL REFLUX can be a serious, sometimes


life-threatening disease. Scan the text about a new, less invasive
technique of diagnosing the disorder and mark the lines that give you
the following information:
-

definition of gastroesophageal reflux


symptoms
3 ways to diagnose it
the surgical option to cure it
how the Bravo system works and what are its advantages

Gastroesophageal reflux occurs when the valve between the esophagus and
stomach (the cardiac sphincter) weakens, and gastric fluids flow back up into the
esophagus and throat. The condition can cause symptoms ranging from
heartburn to chest pain that can be mistaken for heart disease. Some people
become hoarse and have difficulty speaking, and chronic reflux might also
increase the risk of esophageal cancer.
The most common way to diagnose reflux is to put a patient on treatment with
antacids or other drugs, and then follow the patient to see if symptoms improve
or disappear. It is also possible to diagnose reflux by examining a patient with an
endoscope, a rubber tube with a camera at the end that is slipped into the
esophagus of a sedated patient. Yet another option is 24-hour pH testing to
monitor acid levels in the esophagus, which involves running a catheter up the
patient's nose and down the back of the throat into the esophagus.
To resolve the condition, in chronic cases many patients decide on minimally
invasive surgery to strengthen the cardiac sphincter.
The Bravo diagnosing system does without the catheter entirely and is thus more
comfortable than traditional pH testing methods. It uses a small capsule that
doctors attach to the wall of the esophagus near the junction to the stomach. The
capsule transmits electrical signals to a pager-like device. After about a week,
the capsule falls away from the wall of the esophagus and passes naturally out of
the system.
The Bravo system causes less nasal and throat irritation than the trans-nasal
catheter. Patients can maintain their normal diet and activities, which enables
more representative test results to aid diagnosis and plan treatment.

Now choose 3 of the collocations in italics and put them into sentences of
your own.
GASTROINTESTINAL DISORDERS
Match the names of some common GI disorders with their simple
explanations:
1 CHOLECYSTITIS
2 CHOLELITHIASIS
3 CIRRHOSIS
4 DIVERTICULOSIS
5 ENTERITIS
6 ESOPHAGEAL
VARICES
7 GASTRITIS
8 HEMORRHOIDS
9 HIATAL HERNIA
10 IRRITABLE BOWEL
SYNDROME (IBS)
11 INGUINAL HERNIA
12 PEPTIC ULCER
13 PERITONITIS
14 ULCERATIVE COLITIS

A inflammation of the small intestine


B various GI symptoms in the otherwise normal
intestine
C swollen veins in the rectal and anal lining
D protrusion of part of the stomach through the
diaphragm
E gallstones in the gallbladder
F inflammation of the peritoneum
G protrusion of part of the intestine in the groin
area
H chronic inflammation of the colon with the
presence of ulcers
I side pockets (pouches) in the intestinal wall
J inflammation of the gallbladder
K swollen veins in the distal part of the esophagus
or the upper part of the stomach
L inflammation of the stomach lining
M chronic liver disease often associated with
alcoholism
N lesion of the stomach or duodenal lining

CASE REPORT: IRRITABLE BOWEL SYNDROME (IBS)


Provide the missing medical terms from their definitions below:
Condition description
The (1) ____________ is characterized by some combination of:
(2) ____________ pain
altered bowel function, (3) ____________ or (4) ____________
hypersecretion of (5) ____________ ____________
(6) ____________ symptoms (7) ____________, (8) ____________,
(9) ____________
small (10) ____________

a feeling of incomplete evacuation after (11) ____________


A most definitive sign is (12) ____________ ____________, particularly in the
descending colon.
The symptoms are probably due to (13) ____________ of the colon with no
demonstrable organic pathology.
The condition is also known as nervous indigestion, spastic colitis, intestinal
neurosis.
It is important to eliminate all other possible pathologies before diagnosing IBS.
1
2
3
4
5
6
7
8
9
10
11
12
13

set of symptoms occurring together


pertaining to the part of the body between the thorax and the pelvis
infrequent/difficult evacuation of bowel
abnormal frequency/liquidity of stools
slime from the large intestine
pertaining to indigestion
excessive amounts of air/gas in the stomach or intestine
unpleasant abdominal sensation often culminating in vomiting
lack of appetite
stools
bowel evacuation
involuntary contraction of the bowel
bad/poor function

A NEW LIVER
I DID YOU KNOW THAT...?

the ancient Greeks and Romans regarded the liver as the seat of life, and
used to predict the outcome of war by looking at the livers of sacrificed
animals before going into battle?
the liver, beside the brain, is the only organ so complex that its function
cannot be artificially supported or replaced in the event of damage or failure?
living donor liver transplantation is possible because the liver unlike any
other organ in the body has the ability to regenerate, or grow, in only a
period of weeks?

II Read the text and decide whether the statements below are true or false.
Mark the places which support your decision.
LIVER TRANSPLANTS
Several years ago, the unexpected death of a healthy donor following a liver
transplant operation in New York raised questions about how justified living
donor transplants are, even if the risks involved are minimal an estimated 0.5%

to 1%. Voices were heard asking for a national moratorium on the procedure and
recommending cadaver liver transplants instead.
Currently, about 20,000 people in the US are on the waiting list for liver
transplants, according to the United Network for Organ Sharing, which controls
the allocation of cadaver organs; only about 5,000 can expect to get a transplant
each year many will die waiting.
In living donor transplantation, a piece of liver is surgically removed from a living
person and transplanted into a recipient, immediately after the recipients
diseased liver has been entirely removed.
After the donor liver is removed, preserved and packed for transport, it must be
transplanted into the recipient within 12 to 18 hours. The surgery begins by
removing the diseased liver from the four main blood vessels and other
structures that hold it in place in the abdomen. After the recipient's liver is
removed, the new healthy donor liver is then connected and bloodflow is
restored. The final connection is made to the bile duct, a small tube that carries
bile made in the liver to the intestines.
After this demanding surgery, the postoperative course must be closely
monitored for possible complications typically involved in live liver transplants:
infection, pneumonia, blood clotting, allergic reaction to anesthesia and injury to
the bile ducts or other organs.
In comparison with the risks for liver donors, for kidney donors the risk of death
ranges from 1 in 2,500 to 1 in 4,000. That explains why nearly 40% of kidney
transplants in the US come from living donors.
The operation to transplant a liver, however, is a lot trickier than the one to
transplant a kidney. A living donor transplant works best when an adult donates a
small portion of the liver to a child. Usually only the left lobe of the organ is
required, leading to a mortality rate for living donors of 1 in 500 to 1 in 1,000. But
when the recipient is another adult, as much as 60% of the donor's liver has to be
removed, which makes the odds less favorable, both for the donor as well as for
the recipient.
Transplant centers have developed screening programs to ensure that living
donors fully understand all the risks involved in their decision. But there will
always be family members prepared, without much thought, to accept almost any
risk for a loved one. Thus the ethical committees within the medical profession
should, together with society, provide guidelines to help those involved to make
the right decision in each individual case.
1. There were requests for living donor transplants to stop because of the risks
involved for the recipient.
2. About one fourth of the patients on the US waiting list for liver transplants can
expect to be transplanted every year.
3. A donor liver must be implanted within a day of removal.
4. Kidney donors are at greater risk of death than liver donors.
5. Kidney transplants are less complicated than liver transplants.
6. The donor mortality rate is higher in adult-child transplants.

III Now restore the original word order in the sentences describing the liver
transplant procedure and then put the sentences into sequence:
1. the healthy donor liver / bloodflow / is restored / then / is connected / and
__________________________________________________________
2. be transplanted / the recipient / must / within 18 hours / into / it
__________________________________________________________
3. must / for possible complications / closely monitored / be / the postoperative
course
__________________________________________________________
4. surgically / from / a portion / a healthy person / removed / is / of the liver
___________________________________________________________
5. connected / the bile duct / finally / gets
___________________________________________________________
6. is preserved / for transport / and / then / it / packed
___________________________________________________________
7. that hold it in place / diseased liver / from the structures / is removed / the
recipients
___________________________________________________________
Translate:
Prema studiji objavljenoj u New England Journal of Medicine, a na temelju
istraivanja provedenog u 1500 amerikih bolnica, smrtnost nakon razliitih
kirurkih zahvata to je manja to vie takvih zahvata odreeni kirurki tim obavlja.
Tako je npr. kod sranoilnih operacija i zahvata na prostati smrtnost 25% do
40% nia u bolnicama gdje se takve operacije obavljaju 200 ili vie puta godinje.
Za neke rjee operacije, ukljuujui potpunu zamjenu kuka i odstranjenje
debelog crijeva, smrtnost je znaajno pala u bolnicama gdje se godinje obavlja
najmanje 10 do 50 takvih zahvata.
Preporuka je studije da se pojedine bolnice specijaliziraju za specifine
visokorizine zahvate, to bi rezultiralo smanjenjem rizika od smrtnog ishoda
odreenih operacija.
CASE REPORT: ACUTE PANCREATITIS
Explain the underlined medical terms using general English:
A 42-year-old female presented with an acute onset of abdominal pain and
vomiting. She had a recent 3-week history of upper respiratory infection and a 5day history of diarrhea.
PMH: Allergic (1) rhinitis, endometriosis, (2) hysterectomy, (3) appendectomy,

(4) cholecystectomy, (5) hypertension.


Social history: The patient denied alcohol or tobacco use.
Physical examination: Within normal limits with the exception of the abdomen,
which was very tender to palpation, mostly around the (6) epigastric
area. There were significantly decreased bowel sounds. No
(7) hepatosplenomegaly.
Diagnostic exams: An ultrasound of the abdomen demonstrated abundant
upper abdominal bowel gas that moderately obscured the liver and totally
obscured the pancreas. The gallbladder was not visualized, presumably
surgically removed. No
(8) biliary tract dilatation could be identified and there was mild diffuse
(9) adipose infiltration of the liver.
Hospital course: The patient was admitted to the medical floor with a diagnosis
of (10) acute pancreatitis. She was made (11) NPO and given intravenous fluids.
For pain control, she was started on intravenous meperidine via a patientcontrolled (12) analgesia pump, but since this did not lessen her pain, the
medication was changed to morphine with better effect. After 3 days, she
required no pain medications and was started on a clear liquid diet. She tolerated
this well with no recurrence of abdominal pain.
1
2
3
4
5
6
7
8
9
10
11
12

___________________________________________________________
___________________________________________________________
___________________________________________________________
___________________________________________________________
___________________________________________________________
___________________________________________________________
___________________________________________________________
___________________________________________________________
___________________________________________________________
___________________________________________________________
___________________________________________________________
___________________________________________________________

CARDIOVASCULAR SYSTEM
CIRCULATION OF BLOOD
Put the following alphabetically listed structures where they belong in the
description of pulmonary and systemic circulation:
AORTA
AORTIC VALVE
LEFT ATRIUM
LEFT VENTRICLE
LUNGS

MITRAL VALVE
PULMONARY ARTERY
PULMONARY VALVE
PULMONARY VEINS

RIGHT ATRIUM
RIGHT VENTRICLE
TRICUSPID VALVE
VENAE CAVAE

Deoxygenated blood is brought to the heart by the two largest veins in the body,
the ____________ ____________. It first enters the ____________
____________, and when that contracts, the blood is forced through the
____________ ____________ into the ____________ ____________. Its
contraction pumps blood through the ____________ ____________ into the
____________ ____________, whose branches take it to the ____________.
This is the place where gaseous exchange takes place and blood gets freshly
oxygenated, returning to the heart by way of the ____________ ____________.
Blood enters the ____________ ____________, from which it is forced, through
the ____________ ____________, into the ____________ ____________. The
contractions of its thick walls propel the blood through the ____________
____________ into the ____________, whose branches carry it all over the
body.

IS YOUR LIFESTYLE DEAR TO YOUR HEART?


I WHAT CAN YOU DO?
By adopting a healthy lifestyle you can reduce the danger of developing
and dying from cardiovascular disease. Get the titles right for the
following bits of advice (in some cases, there are alternatives), and then
match the titles with the appropriate passages (the key words have been
left out):
1
2
3
4
5
6

STOP
LOWER
LOSE
DONT COUNT ON
REDUCE
CONTROL

A
B
C
D
E
F

BLOOD PRESSURE
WEIGHT
CHOLESTEROL
STRESS
HORMONES
SMOKING

a) Chronic ____________ can increase the risk of heart disease and lead to
unhealthy habits like smoking, abusing alcohol and eating junk food. Yoga
and meditation can reduce ____________; so can getting enough sleep every
night. If you are depressed, get support or treatment as necessary.
b) ____________ more than doubles your chances of having a heart attack. If
you quit, that risk is cut in half within two years; after 10 years, the odds return
to nearly normal.
c) ____________ protect younger women against heart disease, but we know
now that ____________ replacements do not protect postmenopausal women
- and can actually increase the risk for women who have already had a heart
attack.
d) Elevated lipids - ____________ and triglycerides are important risk factors.
Although doctors have traditionally focused on levels of LDL, HDL may be a
better predictor of heart-disease risk in women. Women should maintain HDL
at levels as high as possible.
e) ____________ makes the heart work harder to move blood through the body,
so it puts you at higher risk of both heart disease and stroke. If you get short
of breath when you are physically active, dont ignore it. And if you have
____________, treat it with proper diet, exercise and medication if needed.
f) Carrying excess ____________, especially around the middle, increases your
risk of a heart attack or stroke. ____________ can also lead to diabetes, a
major risk factor. Doctors recommend a reduced-calorie diet with lots of
vegetables and whole grains, plus 30 minutes of moderate aerobic exercise a
day.
II

DO YOU KNOW YOUR BMI?


Figure out your BMI to see if you are at risk of heart disease:

To figure out your body mass index (BMI), first you have to divide your weight (in
kilograms) by your height (in meters), and then divide the result by your height
once again.
e.g. 60 (kg) : 1.67 (m) = 35.92 : 1.67 = 21.5
The body mass index, a measure of the relation of weight to height, is a common
way to estimate excess weight. Most health advisers say that a BMI of under 25
is healthy, 25-30 is overweight and over 30 is obese. But according to some
studies, women should aim for an even lower BMI, as it was found that the risk of
heart disease actually increases above a BMI of 22, not 25.
Interestingly, not only the amount of weight is important, but also how that weight
is distributed. Though female fat is distributed more evenly than that of men,
overweight women fall into two categories: apples, whose fat is concentrated in

the waist, and pears, who have fatter hips and thighs. It's now widely
recognized that obese people have a greater risk of heart disease, but there is
new evidence to suggest that the pearstyle fat may actually protect against
attacks. A recent Danish study found that women with fat deposits in the legs,
hips and buttocks were less likely to have atherosclerosis.
III Work in pairs. Each of you should read one of the following items
talking about the RELATION BETWEEN WEIGHT AND CARDIAC
PROBLEMS problems and then tell your partner about it:

Although it is well known that regular exercise and weight loss are good for
your health, the results from a recent study of women with heart problems
suggest that sometimes its better to be fit than thin. Active women, no matter
how thin or fat, were much less likely to to have a heart attack and other
cardiac problems than women who didnt exercise.
Nevertheless, weight matters: another study proved that overweight women,
regardless of how much they exercised, were nine times more likely to
develop diabetes than women of normal weight.

Losing weight, especially around the waist, lowers your risk of heart disease,
high blood pressure and diabetes. But this only works if you get rid of your fat
the right way, by diet and exercise. Liposuction, a shortcut to a slim figure,
wont help. A study has shown that a group of women who each had about 10
kg of abdominal fat surgically removed, didnt show any improvement in
insulin sensitivity, cholesterol level, blood pressure or other risk factors for
heart disease three months after the operation.

IV Test yourself: HOW MUCH DO YOU KNOW ABOUT CHOLESTEROL?


Work in pairs. Cover the answers, read through the 8 statements and
decide whether they are true or false, giving arguments for your opinions.
Then go through the answers in turns, explaining to your partner where
and why he / she went wrong.
1. The body needs cholesterol.
2. The best way to reduce cholesterol is to limit cholesterolrich foods, such as
eggs.
3. To lower your cholesterol, you should stop eating all meat.
4. Any total cholesterol level below 6.5 is fine.
5. All vegetable oils are good for the heart.
6. Lowering cholesterol can help people who have already had a heart attack.
7. Exercise can raise good cholesterol.
8. Women don't need to worry about high levels of cholesterol and heart disease.

Question 1: TRUE. Cholesterol is a soft, waxy substance that the body uses to
build cell membranes and make steroid hormones, such as estrogen, testosterone
and cortisone. The problem is excess cholesterol particularly the bad LDL
cholesterol, which contributes to plaque buildup in the arteries.
Question 2: FALSE. Although so-called dietary cholesterol does raise blood
cholesterol in most people, saturated fat is mostly responsible. The most effective
way to control blood cholesterol is to reduce foods high in saturated fat, such as
full-fat cheese, cream, butter and meat.
Question 3: FALSE. Red meat is high in saturated fat, which can raise
cholesterol. But lean meat can be part of a healthy diet. Reduce your daily intake
of lean meat to 150 g.
Question 4: FALSE. For most people, total cholesterol should be under 5.5. But
cholesterol comes in two basic forms. In general, bad LDL cholesterol should be
below 3.6 unless you have heart disease or diabetes, in which case you should
aim even lower. However, a low level of good HDL cholesterol - under 1.1 raises the risk for heart disease.
Question 5: FALSE. The vegetable oils found in hard margarine and many
baked products are high in saturated fat and raise blood cholesterol. Focus
instead on the other vegetable oils, preferably olive, which contain mostly
unsaturated fats.
Question 6: TRUE. People who have had one heart attack are at higher risk for a
second, but lowering cholesterol can greatly reduce that risk. If you have heart
disease, your LDL level should be less than 2.7.
Question 7: TRUE. Doctors recommend at least half an hour a day. Other
measures that can help raise HDL include losing weight and stopping smoking.
Question 8: FALSE. Before menopause, women tend to have lower cholesterol
levels than men. But afterward, their levels go up, along with their risk for heart
disease.
FEMALE HEARTS
I In groups of 3, read about how the possible causes, manifestations and
consequences of three major vascular disorders may differ in men and
women.
Take turns to sum up the information on one disorder for the group.

ANGINA
Women vs. men: More women than men complain of chest pain, but in relatively
few women angina is accurately diagnosed. Doctors are just beginning to realize
that any chest pain in women, even younger women, could be a sign of heart
trouble.
HEART ATTACK

Women vs. men: Women tend to experience heart attacks 10 to 20 years later
than men, but a woman's first heart attack is more likely to be fatal. Many women
don't experience the classic symptoms that men feel pain in the chest and arm
and shortness of breath. Some doctors believe this is because heart attacks in
women may have more to do with spasms in the heart vessels than with blocked
arteries.
CONGESTIVE HEART FAILURE

Women vs. men: One in five women with congestive heart failure will die within
one year of their diagnosis, probably because women tend to be older and their
disease more advanced when they receive the diagnosis. For the same reasons,
twice as many women as men who have a heart attack will be disabled by heart
failure.
II Read the text about HEART ATTACKS IN MEN AND WOMEN and
mark the passages which contain the following information:
-

the main risk factors for heart attacks in both sexes


the physiology of a heart attack
how an angiogram is performed
patterns of vessel blockage in women
the difference in plaque buildup in men and women
the difference in the symptoms of a heart attack
how angioplasty is performed
the risks of angioplasty for women in the past
drug dosage concerns in the treatment of women

The more scientists learn about female hearts, the more they realize that females
aren't just smaller versions of males. There are subtle but important differences in
how women's cardiovascular systems respond to various factors which may
cause heart disease.
It has been established that female hormones protect women against heart
disease before menopause, although doctors arent sure how. But all the benefits
of female hormones disappear with menopause and women end up with more or
less the same risk of heart disease as men.
Smoking is the most dangerous risk factor for both sexes, followed by diabetes,
high blood pressure, high cholesterol levels, excess weight and physical inactivity.
Some factors, however, seem to affect women more severely than men. In fact,

smoking and diabetes completely counteract the protective benefits a woman


normally enjoys before menopause.
Cardiologists are confident that they understand how heart attacks occur in men.
Plaque, built up over decades on the inside of a coronary artery, becomes
unstable and bursts, triggering a clot that blocks a blood vessel. Doctors can see
these plaques by means of an angiogram, in which a catheter is threaded through
an artery in the groin or leg up to the arteries of the heart and a dye is then
released to make blockages easier to spot.
Bursting plaques, the immediate triggers of a heart attack, may not be as
significant for women as for men. Doctors have noticed that some of their female
heart-attack patients show few signs of artery-clogging plaques on their
angiograms. It seems their blockages don't occur in the major arteries of the
heart, where angiograms are performed and bypasses are most effective. It may
be that blood flow is restricted in the smaller vessels that branch off the coronary
arteries. And perhaps the problem isn't plaques at all but the fact that these
smaller blood vessels are more prone to spasm, closing suddenly at the slightest
trigger, cutting off the flow of blood to parts of the heart.
It is also possible that plaques, whether in the main coronary arteries or the
smaller vessels, behave differently in women. Unlike men, women tend to
distribute all the garbage associated with atherosclerosis, such as saturated fat
and oxidized waste products, more evenly throughout the arteries. So although
women generally avoid the big plaques that kill so many men in early middle age,
the continuing buildup in women's arteries may make itself felt in their 60s or later.
Also, the classical symptoms of a heart attck chest pain that radiates into the left
arm - are more diffuse in women and can include severe breathlessness,
sweating, nausea, vomiting and pain in the neck, arms and shoulders. Women
often don't interpret the symptoms correctly, which puts them at greater risk.
Finally, American studies suggest that women may at last be benefiting as much
as men from angioplasty, a procedure in which doctors use catheters and
balloons to open up dangerously narrowed arteries and insert stents to keep the
arteries open. In the past, catheters and stents were all made in one standard size
to fit men's larger arteries. As a result, women suffered more complications and
a much higher risk of death from angioplasty. Also, until about three years ago
doctors prescribed the same doses of the anticoagulant heparin for men and
women, leading to greater internal bleeding in women. Smaller catheters and
stents and lower doses should give women better results.
HEART ATTACK
I CASE REPORT: MYOCARDIAL INFARCTION
Provide the missing medical terms from their definition below:
A 33 year-old-man was admitted to our emergency department complaining of 5
hours of severe crushing chest pain.

On physical examination, paleness and cold sweating were noted. His


(1) ____________ and (2) ____________ blood pressures were 110 and 80
mmHg respectively, and his (3) ____________ ____________ was 90 /minute.
The (4) ____________ showed (5) ____________ ____________, and was
consistent with an (6) ____________ ____________ ____________
____________.
.
.
.
We decided to perform primary (7) ____________ coronary intervention (PCI).
(8) ____________ ____________ and left (9) ____________ were performed.
The left coronary system was imaged at left and right oblique, right cranial and
caudal and anteroposterior cranial positions. Significant coronary artery (10)
____________ ____________ narrowing was observed in the mid segment of
the left anterior
descending coronary artery at left anterior oblique cranial position on coronary
(11) ____________. The right coronary artery was normal. The left ventricular
angiography showed (12) ____________ and (13) ____________
____________. PCI was cancelled and medical follow up was decided.
The patient was transferred to coronary care unit for medical treatment.
.
.
.
A normal hemoglobin level was achieved after ten hours. Three hours after the
patient was admitted to the emergency department, his chest pain had
completely disappeared, ST elevations had come to an (14) ____________ line
and frequent ventricular (15) ____________ were observed on the monitor.
Myocardial enzyme values taken after twenty-four hours were elevated to a level
three times greater than normal.
.
.
.
At the one year follow-up visit, our patient was healthy and had no cardiac
complaints.
1
2
3
4
5
6
7
8
9
10
11
12
13
14
15

occurring during the contraction of the heart


occurring during the dilatation of the heart
number of ventricular contractions per min
procedure of recording the el. activity of the heart muscle
normal heart rhythm originating in the SA node
sudden-onset, situated in the AV septum, heart attack
performed through the skin
radiographic visualization of the blood vessels of the heart
radiography of a ventricle of the heart
pertaining to the lumen during systole
radiographic record of blood vessels
located at the apex
at the front and side, abnormally decreased mobility
showing no variation in electric potential
premature contractions of the heart independent of the normal rhythm

II Read about CARDIOPULMONARY RESUSCITATION (CPR) in cases of


cardiac arrest and sum up the text in 1 sentence.
The high number of deaths after cardiac arrests is often blamed on the time
delay between collapse and treatment. In the US, in out-of-hospital cases this
delay averages 8 minutes.
In situations where the patient can be treated within a minute or two with
automated external defibrillators (AEDs), an immediately delivered shock is the
best option. However, in out-of-hospital cases CPR seems to be of benefit.
According to recent reports, the provision of approximately 90 seconds of CPR
prior to use of AED is associated with increased survival when response intervals
are 4 minutes or longer. This conclusion is the result of a study which
investigated the effect of providing 90 seconds of CPR before defibrillation in
patients who had had out-of-hospital cardiac arrests. The survival of 478 patients
who received CPR before defibrillation during a 36-month period was compared
with the survival of 639 similar patients treated in the 42 months before CPR was
introduced.
The researchers reported that provision of CPR was associated with an
improvement in survival from 24% to 30%. Also, the percentage of survivors with
good neurological function on discharge increased from 71% to 79%.
Although these results are encouraging, researchers think that further clinical
trials are necessary before they are accepted as all-conclusive.

III Read the text about INFLAMMATION as a possible contributing factor in


heart attacks and decide whether the statements below are true or false.
Mark the places which support your decision.
Although cardiologists always urge their patients to reduce the fat in their diet,
still half of all heart attacks in the US occur in people with normal cholesterol
levels.
Now research has shown that inflammation might be another significant
contributing factor. It seems that a similar process like the one initiated by the
body's immune system against the joints in arthritis may also be occurring in the
blood vessels of the heart. No one knows exactly what triggers the immune
system in heart patients - it could be fatty deposits or bacteria or the toxins in
cigarette smoke or the physical strain caused by high blood pressure. As the
immune system becomes active, white blood cells, clotting factors and a number
of other factors of the body's defense system gather and begin to build up inside
a vessel wall, forming plaques. The most dangerous plaques are those prone to
rupture, as the explosive release of clotting factors and other cells into the blood
can cause a heart attack.
Thus noticing, tracking and measuring inflammation is of great importance in the
process of prevention. This can be done by testing for C-reactive protein (CRP),
produced wherever there is inflammation. It has been proved that those with low

levels of bad cholesterol (LDL) but high levels of CRP, as measured by a new
supersensitive test, suffered the same rate of heart attacks as those with high
levels of LDL and low CRP. That doesnt mean that CRP testing should replace
cholesterol testing. Lipids tell us how much plaque has built up in an artery, and
CRP tells us how likely that plaque is to rupture and cause a heart attack.
Fortunately, there are some efficient medications to keep inflammation under
control. Aspirin may protect against heart disease not only by keeping clots from
forming but also by controlling inflammation. And the newest studies show that
even some of the statin drugs, traditionally used to lower cholesterol, are good for
decreasing CRP levels as well.
1. The majority of heart attacks in the US are caused by elevated cholesterol
levels.
2. The immune system appears to have an important role in triggering attacks in
heart patients.
3. High CRP levels are in direct proportion to inflammatory processes and
therefore also to plaques that are likely to rupture and cause a heart attack.
4. Statins are drugs primarily used to lower CRP.
IV Read about a new technique of HEART IMAGING which might be
Relevant in the process of prevention. As you are reading, supply the
correct forms of the words in brackets.
Mark the lines that answer the following questions:
1. Why did cardiologists find images of the heart unreliable until recently?
2. How is EBCT different from traditional CT scanning of the heart?
3. How do calcium deposits in the heart relate to the possibility of a heart attack?
Until recently, most cardiologists put little value on pictures of the heart. They
assumed that it was impossible to get a truly accurate image of an organ in
constant motion. But with (1) ____________ (IMPROVE) in the technology for
taking freeze-frames of the beating heart, physicians have started to (2)
____________ (RELIABLE) more and more on images of the heart not just for
looking at problem areas but also to (3) ____________ (PREDICTION) the (4)
____________ (LIKELY) of future heart trouble.

The newest device, an ultrafast form of computer scanning called electronbeam computer tomography (EBCT), notices the (5) ____________
(PRESENT) of tiny deposits of calcium in the heart. One study based on
the scan showed that patients who build up 20% or more calcium each
year have an 18-fold greater chance of suffering a heart attack than those
with less calcium in their hearts.
Although it is not yet clear whether calcium deposits spotted by EBCT can

(6) ____________ (ACCURATE) predict who will go on to have a heart attack,


at least it is one more test that cardiologists can use to decide how (7)
____________ (AGGRESSION) to treat a patient who is at risk.

ARRHYTHMIA

I Arrhythmia (dysrhythmia) is any variation from the normal rhythm of the


heartbeat. Match the types of arrhythmia and associated occurrences
with their definitions:
1 CARDIAC ARREST
2 CARDIOVERSION
3 FIBRILLATION
4 FLUTTER
5 HEART BLOCK
6 PALPITATION

A failure of normal conduction of impulses from


atria to ventricles; transient or permanent
B rapid, regular contractions of atria or ventricles
C subjective sensation of rapid or irregular
heartbeat
D establishing a normal heart rhythm by
electroshock (defibrillation)
E rapid, irrregular contractions of atria or ventricles
F sudden stop in heart movement caused by heart
block or ventricular fibrillation

II Read about the traditional surgical way of treating certain kinds of


arrhythmia, and new research which promises a minimally invasive
procedure.
LESS INVASIVE TREATMENT FOR ARRHYTHMIA
Atrial fibrillation, causing painful symptoms and accounting for about 15% of all
heart attacks, affects more than 2 million Americans. Medication can alleviate
symptoms in some patients, but it cannot cure the problem.
Normally, electric signals trigger the synchronized contraction of muscles in the
heart's two upper chambers, the atria. The electrical impulse begins at the sinus
node (SA), then spreads across the upper chambers, the atria. Next, the impulses
reach the atrioventricular node (AV), signaling the ventricles to contract from the
bottom up.
In atrial fibrillation, the atria convulse with erratic electrical charges. The AV node
gets mixed signals about when to fire, and the result is a dangerously rapid,
irregular heartbeat. Blood pools in the atria; clotting is possible, leading to a
stroke.
Twenty years ago, surgeons developed a cure called the Maze procedure to
control these erratic impulses. In the procedure, surgeons make small,
strategically placed incisions in the atria. The incisions generate scar tissue that

serves as a barrier, trapping abnormal electric signals in a maze. Only one path
remains intact, guiding impulses to their correct destination.
With a success rate of more than 90%, the Maze procedure revolutionized the
treatment of atrial fibrillation. However, it is technically difficult and therefore not
performed frequently. As surgeons must temporarily stop the heart in order to
make the incisions, not all patients are healthy enough to endure the operation.
Now researchers have developed an alternative: bipolar radiofrequency. The
method uses two electrodes that pass a current through a section of heart tissue.
Like Maze procedure incisions, radiofrequency energy causes scar tissue that
blocks the abnormal impulses responsible for atrial fibrillation.
So far researchers have tested this procedure on animals, at four of the standard
Maze incision locations. Each lesion took nine seconds to complete, as
compared with Maze incisions, which take 5 to 10 minutes to create.
Preliminary findings suggest that the procedure is safe to do in human patients.
Now rephrase the following sentences as suggested:
1. 20 years ago, surgeons developed the Maze procedure to control erratic
impulses.
The Maze procedure _________________________________________
2. In this procedure, surgeons make small incisions in the atria.
Small incisions ______________________________________________
3. To do that, they must temporarily stop the heart.
The heart ___________________________________________________
4. The incisions generate scar tissue.
Scar tissue __________________________________________________
5. The scar tissue traps abnormal electric signals in a maze.
Abnormal electric signals _______________________________________
6. One intact path guides impulses to their correct destination.
Impulses ____________________________________________________
7. Now researchers have developed the bipolar radiofrequency procedure.
The bipolar radiofrequency procedure ______________________________
8. In this method, they create scar tissue by means of an electric current.
Scar tissue ____________________________________________________
9. They havent tested this procedure on humans yet.
This procedure _________________________________________________

PSYCHIATRY
CLINICAL SYMPTOMS
I Below are some of the most frequently mentioned psychiatric clinical
symptoms. Match them with their definitions:
1 AMNESIA
2
3
4
5
6

ANXIETY
APATHY
COMPULSION
DELUSION
DYSPHORIA

7 EUPHORIA
8 MANIA
9 OBSESSION
10 PARANOIA

A fixed false belief in spite of logical evidence to the


contrary
B delusions of persecution or grandeur
C state of hyperexcitability and hyperactivity
D lack / loss of memory
E exaggerated feeling of physical / mental well-being
F emotional state ranging from uneasiness to dread,
often accompanied by palpitations and chest tightness
G uncontrollable impulse to perform an action
repeatedly and unnecessarily
H involuntary, persistent and recurring idea or image
that is distressing
I restlessness, fatigue, hopelessness
J lack of emotion, indifference

II Now make adjectives from the nouns above. Some of them are often
used in everyday speech. Give examples:
1. _________________
2. _________________
3. _________________
4. _________________
5. _________________
6. _________________
7. _________________
8. _________________
9. _________________
10. e.g. paranoid - Dont be paranoid, the teacher doesnt hate you, and youll
pass.
PSYCHIATRIC DISORDERS
I Sort out the following disorders into the categories below and say what
you know about some of them:
ANOREXIA NERVOSA
BIPOLAR

DEPRESSIVE
NARCISSISTIC

PARANOID
PTSD

BORDERLINE
BULIMIA

OBSESSIVE-COMPULSIVE
PANIC

anxiety disorders:

____________________
____________________
____________________

eating disorders:

____________________
____________________

mood disorders:

_____________________
_____________________

personality disorders: _____________________


_____________________
_____________________
_____________________

SCHIZOID

II Now associate each of the following groups of symptoms with one of


the disorders above:
A severe dysphoria, appetite disturbances, insomnia or hypersomnia, fatigue,
hopelessness, difficulty concentrating, recurrent thoughts of death or suicide
B emotionally cold and distant, indifferent to praise or criticism, or the feelings of
others, lack of interest in social relationships or family life
C recurrent thoughts and repetitive acts (rituals) performed to relieve anxiety,
often interfering with personal and social functioning
D instability of mood, interpersonal relationships and sense of self; impulsive
acts and emotions, alternative involvement with and rejection of people
E because of a disturbance in the perception of ones body, the refusal to
maintain a minimally normal weight, achieved by excessive dieting plus
overactivity
F alternating manic and depressive episodes
G intense fear, insomnia, nightmares and other symptoms caused by a
traumatic event
H continually suspicious, mistrustful, jealous; easily offended due to
misinterpretation of peoples remarks and motives, but not psychotic
I binge eating followed by purging
J intense fear or discomfort accompanied by physical symptoms like
palpitations, sweating, breathlessness, dizziness
K intense preoccupation with oneself, sense of self-importance, lack of social
empathy, arrogance, constant need of admiration

DEPRESSION
According to the Harvard School of Public Health projections for the most
frequent causes of disability and death in the next decades, depression
will soon be topping the list along with cardiovascular diseases.
Only during the 90s, the number of people in the US being treated for
depression has grown more than three times, while twice as many people are on
antidepressants than roughly 15 years ago. The ratio between
psychotherapeutic and medical treatment for depression has changed in favor of
drug treatment, which is logical in view of a better class of drugs available now,
besides being much less expensive than therapists.
Work in groups of 3. First exchange any facts on depression that you have
learned, including information on cases you might know from personal
experience. Then each of you should read about one of the approaches to
treatment and share what you have read with the group.
1. MEDICAL THERAPY FOR DEPRESSION
There are different classes of antidepressant medications. Each type works
differently and has different side effects. Studies have shown that up to 70% of
patients on antidepressants recover from their depression in three to six weeks,
provided that the dose is sufficient and that the medication is taken as
prescribed.
Once it has been established which medication works best for the individual, the
goal of treatment is complete relief of depressive symptoms. To achieve that
goal, doctors should get the necessary feedback from their patiens concerning
how they feel. If there is improvement in three to six weeks, but some symptoms
are still present, the doctor will probably increase the dose of the antidepressant.
If the patient cannot tolerate a higher dose, the doctor may switch to another
medication. If symptoms are no better or worse after three or four weeks, the
doctor should suggest a different antidepressant.
If two adequate antidepressant regimens fail to help an individual, he or she is
considered to have treatment-resistant depression and should see a
psychiatrist immediately.
Once depression is under control, patients should continue taking their
medications at the same dose for 9 to 12 months to prevent a relapse. After a
second depressive episode, the doctor may prescribe an antidepressant for as
long as two years. Patients who have had three or more episodes of depression
have a very high risk of recurrence. These patients may need to take an
antidepressant for an indefinite period (possibly for life) to decrease the chance
of future depressive episodes.

2. PSYCHOTHERAPY FOR DEPRESSION


Psychotherapy offers people the opportunity to identify the factors that contribute
to their depression and to deal effectively with the psychological, behavioral,
interpersonal and situational causes. There are several therapheutical
approaches that can help depressed individuals recover. Trained therapists can
work with depressed individuals to:
identify the life problems that contribute to their depression, and help them
understand which aspects of those problems they may be able to solve or
improve;
define options and set realistic goals for the future which will work towards
restoring the patients mental and emotional well-being;
identify negative or distorted thinking patterns that contribute to feelings of
hopelessness and helplessness that accompany depression, and help
support a more positive outlook on life;
help the patient regain a sense of control and pleasure in life.
There is some evidence that ongoing psychotherapy may lessen the chance of
future episodes of depression or reduce their intensity. Through therapy, people
can learn skills to avoid unnecessary suffering from later bouts of depression.
3. DEPRESSION AND NUTRITION
The fact that nutrition plays an important role in depression is not well known
as yet, although a number of studies have found that vitamin deficiencies are
more prevalent among subjects with depression compared to healthy
individuals. Vitamin deficiencies that have been found include vitamin B1,
vitamin B6, vitamin B12 and folic acid deficiency. Besides some other
functions, these vitamins play important roles in neurotransmitter metabolism.
Severe folic acid and vitamin B12 deficiency in themselves causes symptoms
of depression. Other scientific studies have shown that not only can vitamin
deficiencies be detected in patients with depression, but also that a vitamin
supplementation program in some subjects can alleviate many of the
symptoms.
Low folic acid levels, in particular, are also known to result in a poor response
to antidepressants.
Although it is clear that nutrition plays an important role in depression, there is
much more to be learned about it concerning this disorder. Removing vitamin
and mineral deficiencies and optimizing your nutritional status might be of
help in removing your risk of developing depression as well as in treating the
condition.

PLUS...

The latest studies have shown that drugs precribed to treat depression in
children actually increase suicidal thoughts and behavior in about 4% of
them. As a consequence, the British government decided to prohibit use of

most antidepressants in children and teens, while the FDA obliged all
antidepressant manufacturers to add black-box warnings to doctors,
making them aware of the risk.
SCHIZOPHRENIA
Scan the text on new insights into the physiology of schizophrenia and the
new medications in use. Note down some of the information below the text:
In paranoid schizophrenia, the patient becomes convinced of beliefs unrelated to
reality, hears voices or sees images that exist nowhere but in his mind.
Neuroscientists have now traced such hallucinations to malfunctions of the brain.
Scanning the brains of schizophrenics while they were having hallucinations,
they found evidence of activity in the parts of the sensory cortex that process
movement, color and objects.
Deep within the brain during hallucinations, structures involved in memory (the
hippocampus), emotions (the amygdala) and consciousness (the thalamus) all
light up like lamps. Usually sensory signals are transported deep into the brain,
where they link up with memories and emotions. But the neuronal traffic might go
the other way, too, with activity in the emotional and memory regions triggering
voices and visions.
Scientists still cant explain why the content of hallucinations differs from patient
to patient, but it probably reflects personal experience.
So while some parts of the schizophrenic mind are hyperactive, another key
brain area is nearly silent. Schizophrenia is marked by abnormally low activity in
the frontal lobes. These regions control the emotional system, provide insight and
evaluate sensory information. In other words, they provide a reality check.
The absence of a reality check makes willing oneself out of schizophrenia
practically impossible. Even among people who have had the illness for decades,
and who have periods of clarity thanks to medication, only some learn to
distinguish between the voices everyone hears and the voices only they can
hear.
Identifying what happens in the brain during schizophrenic hallucinations doesnt
mean understanding why they happen. The old theory that cold, rejecting
mothers make their children schizophrenic has long been discredited. Scientists
now know that the age of the father may be a contributing factor. The 0.5% risk
of a 25-year-old potential father nearly doubles when he is 40, and triples when
he passes 50. Viruses or stresses that interfere with a fetus's brain development
also raise the risk; mothers who suffer rubella or malnutrition while pregnant have
a greater chance of bearing children who develop the disease. And if there is
schizophrenia in family, you run a higher-than-average risk of developing it.
There is, as yet, no cure for schizophrenia. But drugs can alleviate the
symptoms. Schizophrenics have a surplus of the neurochemical dopamine.
Thorazine, an early antipsychotic, blocked dopamine receptors, with the result
that dopamine had no effect on neurons. But since dopamine is also involved in
movement, Thorazine leaves patients slow and stiff, with the characteristic way

of walking called the Thorazine shuffle. Dopamine also flows through circuits
responsible for attention and pleasure, so Thorazine puts patients in a mental fog
and deadens feelings.
The new antipsychotics, called atypicals, allow people with schizophrenia to
hold jobs and have families. Still, they increase appetite, and may alter
metabolism, resulting in considerable weight gain. Another side effect is foggy
thinking and a loss of libido.
For the time being, even the new drugs are only treatment, and not a cure.

symptoms of paranoid schizophrenia:


brain structures involved in hallucinatory episodes:
less active parts of the brain:
former theory on the cause of schizophrenia:
new theories on possible contributing factors:
excessive neurotransmitter:
function and drawbacks of Thorazine:

ANOREXIA NERVOSA vs. BULIMIA NERVOSA


Read the text which focuses on the type of personality typically affected by
these increasingly frequent eating disorders and fill in some of the
information in the table below:
In many ways, these two disorders are very similar. Both typically begin after a
period of dieting, in some cases accompanied by excessive use of diet pills.
People with either disorder believe that they weigh too much regardless of their
actual weight or appearance. They are pathologically afraid of becoming obese
and preoccupied with food, weight, and appearance. They are also typically
prone to feelings of depression, anxiety, and the need to be perfect.
However, there are also certain differences. Although people with either eating
disorder worry about the opinions of others, those with bulimia nervosa tend to
be more concerned about pleasing others, being attractive and having intimate
relationships. While the typical anorectic is adolescent, bulimia usually presents
in somewhat older persons. On the positive side, people with bulimia nervosa
display fewer of the obsessive qualities that drive people with anorexia nervosa
to control their caloric intake so strictly. On the negative side, they are more likely
to suffer from mood swings, become easily frustrated or bored, and have trouble
controlling their impulses; changing friends and relationships frequently is typical.
Also, more than one-third of bulimics display the characteristics of a personality
disorder.
Another key difference between these two eating disorders are the medical
complications that accompany each. Only half of women with bulimia nervosa
are amenorrheic or have very irregular menstrual periods, compared to almost all

of those with anorexia nervosa. While the main problem in anorectics is


undernourishment which, if untreated, may be fatal, in bulimcs, who usually
manage to maintain an almost normal body weight, repeated vomiting frequently
exposes the teeth and gums to hydrochloric acid, resulting in serious dental
problems, such as the erosion of enamel and even the loss of teeth. Moreover,
frequent vomiting or chronic diarrhea (from the use of laxatives) can cause
dangerous potassium deficiencies, which may lead to weakness, intestinal
disorders, kidney disease, or heart damage.
ANOREXIA N.

BULIMIA N.

Type of
personality
typically
affected

Accompanying
medical
complications

PLACEBO EFFECT IN ACTION


Everyone knows the meaning of placebo effect, but researchers managed to
see it in action, so to speak, by using an imaging technique that maps differences
in blood flow in the brain.
The test persons were first given harmless but painful electric shocks, and then a
cream they were told would relieve the pain, but which actually didnt contain any
active substances. After the cream was applied, scans of the brain nerve activity
showed visible changes. The regions involved in easing pain became more
active, while the regions which sense pain became quieter, thus proving that just
the expectation of relief may actually provide it.
Or, to put it differently, it could be called a victory of mind over matter.

ONCOLOGY
CANCER VOCABULARY
Match these frequent oncological terms with their definitions:
1 ANAPLASIA
2 BENIGN
3 CARCINOGEN
4 CARCINOMA
5 CHEMOTHERAPY
6 DIFFERENTIATION
7 ENCAPSULATED
8 EXCISIONAL BIOPSY
9 GRADING
10 INFILTRATIVE
11 IN SITU
12 INVASIVE
13 MALIGNANT
14 METASTASIS
15 NEOPLASM
16 RADIOTHERAPY
17 RELAPSE
18 REMISSION
9 SARCOMA
20 STAGING

A actively destroying the surrounding tissue


B new, abnormal growth
C disappearance / decrease of the disease
symptoms
D diffusing / accumulating in surrounding tissues
E the transfer of disease from one structure to
another not directly connected with it
F assessment of the spread of a tumor within the
body
G treatment of disease by ionizing radiation
H tumor (usu. malignant) arising from connective
tissue
I loss of differentiation of cells (dedifferentiation)
J treatment of disease by chemical agents
K cancer-producing substance
L biopsy of a lesion including a significant margin of
healthy tissue
M not recurrent, favorable for recovery (opp. of
malignant)
N return of a disease after apparent cessation
O development of specialized forms, character, and
function of cells and tissues
P microscopic assessment of the degeee of
maturity / differentiation of a tumor
Q enclosed in a capsule
R anaplastic, invasive, metastatic (of tumors)
S malignant new growth consisting of epithelial
cells
T in the place of origin, without invasion of
neighbouring tissues

BENIGN OR MALIGNANT?
Sort out the following characteristics of benign and malignant neoplasms
in the form of opposed pairs into the table below:

infiltrative and invasive


grow slowly
well-differentiated
metastatic

encapsulated and noninvasive


multiply rapidly
nonmetastatic
anaplastic and undifferentated

BENIGN NEOPLSMS

MALIGNANT NEOPLASMS

HOW CANCER DEVELOPS


Before turning cancerous, cells go through a series of changes. Only when
scientists understand the way that happens at the molecular level, they can
design drugs to stop the process.
A decade ago, there were about one hundred medicines in research as potential
anticancer agents. Today there are more than four hundred of them.
I Look at the 5 STEPS OF CANCER DEVELOPMENT and put them into
sequence:
A The tumors appetite grows
In a process called angiogenesis, malignant cells secrete chemicals that attract
and promote the formation of new blood vessels. With a steady supply of
nutrients, the tumor can grow without limits.
B The mistakes accumulate
It becomes harder and harder for the cells to maintain normal growth, as genes
that should be on get turned off and those that should be off are turned on.
C The cancer spreads
Particles of the tumor break off and, in a process called metastasis, migrate
through the blood and lymphatic systems. Eventually the runaway cells invade
other parts of the body and initiate the growth of distant tumors.

D A mistake happens in the cell


Sooner or later, exposure to ultraviolet light, chemicals from the environment or
even the byproducts of normal metabolism damage one of the genes in a cell. In
most cases this does not lead to cancer.
E

The cells turn cancerous

Free of normal restraints, the now malignant cells break all the rules. They divide
uncontrollably, become less attached to their neighbors and invade the space
occupied by normal cells.
II Now look at the WEAPONS in development and those currently used
TO FIGHT THE CANCER at each level and assign them to the
appropriate steps above:
Step _____ weapons: Antiangiogenesis
In clinical trials, agents attack the tumors blood supply in an effort to prevent the
flow of nutrients.
Step _____ weapons: Cancer prevention
This stage involves eating right, not smoking and avoiding sunburns.
Antioxidants like vitamin E may also help.
Steps _____ and _____ weapons: Surgery / Chemotherapy / Radiation
Surgery: Early detection leads to less invasive operations and more cures.
Chemotherapy: Although less toxic than before, these poisons still kill both
healthy and cancerous cells.
Radiation: Even though radiation beams are localized, they still kill a large
amount of healthy cells.
Step _____ weapons: Targeted therapy
Targeted therapy is a general term that refers to a medication that targets a
specific pathway in the growth or development of a tumor. The targets are
various molecules known or suspected to have a role in cancer formation. The
various approaches include:
Antigrowth: A generation of new drugs that aim to block the biological signals
that promote cancer-cell growth.
Cell suicide: Unlike healthy cells, which die at the end of their natural life span
or when they are no longer needed, cancer cells continue

reproducing themselves uncontrollably, and often resist


conventional treatments aimed at forcing them to commit suicide
by damaging them. Experimental drugs try to destroy their specific
defense mechanisms.
Immune booster: Since cancer cells typically evade the immune system,
different vaccines are currently evaluated in large human
trials. They fall into 2 groups: therapeutic vaccines, designed
to stimulate the immune system to recognize and attack
cancer cells withoput harming healthy cells, and prophylactic
vaccines, which stimulate the immune system to attack
cancer-causing viruses and prevent viral infections.
Microradiation: Radioactive isotopes attached to monoclonal antibodies form
radioimmunotherapy agents which target specific cancer cells.

CANCER NEWS

I In groups of 3, read two of the items each and suggest headlines for
them. Then share your information with the group:
______________________________
It is well known that stress is accompanied by increased glucocorticoid secretion,
which can inhibit the immune system (that is why these steroids are often used in
the treatment of autoimmune diseases). Consequently, there is evidence that
prolonged stress can result in an increased incidence of cancer.
______________________________
According to a recent paper in the British Medical Journal, 20% of mothers who
had been diagnosed with cancer dont discuss their illness with their children,
even if they are facing surgery.
It is felt that parents in this situation should get expert help in deciding if, how and
when to talk with their children in order to avoid unnecessary anxiety within the
family. But in any case, talking is better than keeping silent, experts agree.
______________________________
Three recent studies revealed potentially useful, but by no means conclusive,
links between breast cancer and diet.
Mexican researchers reported that a high-carbohydrate diet, typical of that
country, seems to increase the risk of breast cancer. They explained that
increased levels of insulin resulting from such a diet could trigger cells to grow
abnormally.
A Danish study found that girls experiencing peak growth early in puberty, before
the age of 14, were at greater risk of developing breast cancer later in life. The

study linked the tendency of increased average height in women to increased


milk consumption.
A US study showed that women taking aspirin at least 4 times a week for 3
months reduced their risk of developing breast cancer by 30%.
______________________________
Researchers have concluded that physical activity doesnt need to be too
demanding to reduce the risk of breast cancer. They investigated the relation
between recreational physical activity in adulthood and incidence of breast
cancer in about 75,000 postmenopausal women from the Womens Health
Initiative study. It was found that women who did up to 2.5 h per week of brisk
walking had an 18% decreased risk of breast cancer compared with inactive
women. Additionally, the effect of exercise was most pronounced in women in
the lowest bodyweight group.
______________________________
Heat-sensitive pads that are put into the bra at the doctors office may be a new
way to detect breast diseases, including cancer. Unlike mammograms, the pads
produce no radiation exposure, and they are effective on the dense breast tissue
of younger women. They rely on the fact that cancer cells have a more active
metabolism than normal cells and consequently generate more heat.
The pads have been approved for use as an addition to standard screening.
They may prove most useful to women under 40, who do not usually have
mammograms unless a suspicious lump is detected by physical examination.
The pads may be able to detect breast changes much earlier than a physical
exam, although it is not certain yet whether they can detect breast diseases
earlier than mammograms; but they might be used by older women between
mammograms.
_____________________________
Researchers say that its best to follow a lumpectomy for noninvasive breast
cancer with radiation therapy. An eight-year study shows that women who have
had the X-ray treatments cut their odds of a relapse in half.
II Now translate the last item.
III Work in groups of 4. Read the text and do 2 of the following tasks each.
Then show your group the results and compare them with the other
groups:
NO MORE HAIR LOSS
US scientists have discovered a substance that could protect cancer patients
from chemotherapy-induced hair loss. Applied to the skin, the substance slows
down the division cycle of epithelial cells.

It is well known that cytotoxins used in the therapy of certain kinds of cancer
usually affect hair cells as well, because they, like cancerous tissue, also divide
rapidly.
The newly discovered substance, currently undergoing clinical trials, is
expected to be specially effective in cases of lung cancer.
Task 1: Suggest an alternative headline for the text.
Task 2: Write down synonyms for the bolded words.
Task 3: Write a one-sentence summary of the text.
Task 4: Decide whether the following statement is true or false and mark the
line(s) which helped you: Cancer cells, unlike hair cells, are
characterized by rapid division.
Task 5: Write down all the verbs and adjectives commonly used with the word
therapy.
Task 6: Use some of the underlined and bolded words in sentences of your
own, in a different medical context.
Task 7: Write short explanations of the underlined words.
Task 8: Translate the sentence in italics.
BREAST SELF-EXAMINATION
Read about possible DRAWBACKS OF BREAST SELF-EXAMINATION and
then decide whether the following statements are true or false. Mark the
lines that helped you decide:
A new study of women with a family history of breast cancer in the UK adds to
evidence that excessive breast self-examination is counterproductive, because it
increases anxiety and may make early detection of breast cancer more difficult.
833 women aged 17-77 years, from families with histories of breast cancer were
surveyed. 18% claimed to examine their breasts daily or weekly, 56% once or
twice a month, and 26% rarely. General anxiety and cancer-specific anxiety were
lowest among women who examined themselves least often, and highest among
the hypervigilant women.
Women who examine their breasts may be unaware that it can be normal to have
lumps and may interpret any they find as evidence of cancer. This reinforces
feelings of anxiety and may reduce the efficacy of the procedure itself.

The official policy is now one of breast awareness rather than self-examination.
In self-examination, if a woman finds a lump she gets worried, but if she does not
find a lump, she still gets worried because she assumes that she has missed it.
The emphasis now is to check for normality and only see your doctor if
something in your breast suddenly changes. Women are advised to examine
their breasts once a month.
Fundamentally, there is no evidence that frequent self-examination allows
women to detect cancers in a way that will influence the course of the disease
although time is important, a few days more or less dont matter for the chance of
a successful outcome.
1.
2.
3.
4.

The women that took part in the study belonged to a higher risk group.
Anxiety grew proportionally with the frequency of self-examination.
All lumps are evidence of cancer.
Physicians suggest women should check for normality rather than
abnormality.
5. The advice for women is a weekly check.
6. The outcome of the detected cancer depends on whether the woman has
consulted a doctor within days of its detection.
Vocabulary check - suggest SYNONYMS for the words from the text that could
be substituted in the same context:
excessive

_______________

efficacy

_______________

detection

_______________

awareness

_______________

surveyed

_______________

assumes

_______________

claimed

_______________

fundamentally

_______________

CANCER RESEARCH
Scan the paragraphs below and mark the key words that helped you decide
which of them talk about:
1
2
3
4
5

traditional cancer treatments


new approaches to the problem
the importance of timing in prevention, detection and diagnosis
research costs
future goals and hopes in cancer treatment

A
Experts agree: the earlier a cancer is diagnosed, the better your chances of
controlling it. And thanks to a growing understanding of the cancer cell's natural
life cycle, doctors are learning how to detect the disease at its very earliest
stages.
Other researchers are focusing on an even earlier stage, trying to lower the risk
of developing cancer at all. Here the most exciting work centers on the
cyclooxygenase inhibitor called COX-2. This compound is contained in a pain
reliever originally developed to fight pain and inflammation in many medical
conditions (e.g. osteoarthritis and rheumatoid arthritis) by blocking specific
enzymes produced in response to inflammation. Since it was found that the same
enzymes are produced by precancerous and cancerous tissue, the hope is that
COX-2 may be useful in preventing a wider range of cancers, including
colorectal, head and neck, bladder, lung and breast cancers.
B
Eventually, the goal is to detect precisely which molecular processes have gone
wrong in an individual patient's cancer. Rather than being identified as lung
cancer or breast cancer or kidney cancer, tumors will be tagged as, for example,
COX-2 positive.
In conclusion, the researchers' biggest hope for the future is that the new
therapies could transform cancer from an uncontrollable, frequently lethal illness
to a chronic but manageable one similar to diabetes and high blood pressure.
C
Although in many countries basic research into cancer biology is funded by
national cancer institutes, the majority of drug development is done by for-profit
pharmaceutical firms. US companies claim that it costs them between $500
million and $1 billion to bring a single new medicine to market partly because it
can take 15 years for the elaborate testing in animals and humans required by
the law and partly because for every medicine finally approved, there are 5,000
others that fail. The drug companies count on that one success to pay for the
5,000 failures.
D
In the course of their research scientists have accumulated a lot of information
about how cancer works at the molecular level, from its first awakening in the
DNA of a single cell's nucleus to its overall attack on the body. Armed with that
information, they have been developing an assortment of weapons to attack the
disease at every stage of its development. Many of these therapies are just
beginning to reach clinical trials and won't be available to save lives for years to
come.

Unlike chemotherapy and radiation, which use carpet-bombing tactics that


destroy cancer cells and healthy cells alike, these new medicines are like
snipers, targeting cancer cells alone.
E
Apart from surgery, which almost always leaves behind some malignant cells, the
standard treatment for most cancers continues to be radiation and chemotherapy
relatively crude disease-fighting weapons that have limited effectiveness and
leave patients weak and nauseated.

MALE REPRODUCTIVE SYSTEM


STRUCTURES AND FUNCTIONS
Write the name of the appropriate structure beside its function. Choose
from the following list:
BULBOURETHRAL GLANDS
EJACULATORY DUCT
EPIDIDYMIS
GLANS PENIS
INTERSTITIAL TESTICULAR CELLS
PREPUCE (FORESKIN)

PROSTATE
SCROTUM
SEMINAL VESICLES
SEMINIFEROUS TUBULES
TESTES
VAS (DUCTUS) DEFERENS

sac which contains the testes: _________________________________


location of the urethral orifice: _________________________________
produce spermatozoa and testosterone: _________________________
produce testosterone: ________________________________________
glands below the prostate, secrete fluid
into the urethra: _____________________________________________
glands that secrete fluid into the
vas deferens: _______________________________________________
gland at the base of the urinary bladder,
secretes fluid into the urethra: __________________________________
carries sperm from the epididymis
toward the urethra: ___________________________________________
carries sperm from the seminiferous
tubules to the vas deferens: ____________________________________
produce sperm in the testes: ___________________________________
carries semen from the seminal vesicles
to the urethra: _______________________________________________
covers the glans penis in uncircumcised
men: ______________________________________________________

DID YOU KNOW THAT...?


Work in groups of 4. Each of you should read 2 of the following items
and tell the group about it.

the penis of many mammals contains a bone of variable shape called os


penis or baculum?

it was as late as 1878 that the practice of castration for male singers in the
papal choir was finally abolished by Pope Leo XIII?

male sexual function requires the synergistic, rather than antagonistic,


action of the parasympathetic (to stimulate the bulbocavernosus muscles
at the base of the penis) and sympathetic systems (to stimulate the
smooth muscles in the urethral wall whose contractions help to eject the
semen)?

ejaculated spermatozoa can live up to 5 days at body temperature?

the terms sperm and semen are often used as synonyms, although the
first means spermatozoa, and the second sperm plus glandular secretions
that are contained in the ejaculated fluid?

the terms infertility and sterility are often used as synonyms, although
infertility is in many cases treatable, while sterility is a permanent
condition?

in some cases male infertility may be the result of the testes being
exposed to excessively high temperatures over a prolonged period of time
(e.g. in saunas, or at work as long-distance drivers)?

vasectomy, the procedure of making a male sterile by removing a segment


of the vas deferens, can in some cases be reversed through a procedure
called vasovasostomy (the reconnection of the cut-off ends of the vas
deferens)?

Translate:
MUKA MENOPAUZA - MIT ILI STVARNOST?
Andropauza je naziv za pad proizvodnje mukih hormana u tijelu
sredovjenog mukarca. Sindrom ima mnoge simptome, meu kojima je
najei nemogunost postizanja erekcije i/ili smanjen libido, depresija te
smanjivanje miine snage.
Neka su istraivanja pokazala da lijeenje testosteronom djeluje na isti nain
kao i lijeenje estrogenom kod ena u menopauzi. No kako su nepoeljni
uinci testosterona pogoravanje postojeih bolesti prostate ili krvoilnog
sustava, mukarci s takvim bolestima nee biti obuhvaeni u daljnjim
istraivanjima.
Translate:
SMOKING AND ERECTILE DYSFUNCTION
Beside other possible physiological causes, vascular impairment in particular
is highly associated with erectile dysfunction. Among vascular diseases that
are recognized as risk factors, myocardial infarction, coronary bypass surgery,
stroke, peripheral vascular disease, and hypertension have been singled out.
Recent interest has turned towards cigarette smoking as a risk factor of
erectile dysfunction, presumably along the basis that the harmful substances

contained in cigarettes exert adverse effects on penile circulation relevant to


erectile function.

PROSTATE CANCER
Read about a study on how lifestyle may relate to the risk of developing
prostate cancer, and then decide if the statements below are true or
false. Mark the lines which support your decision:
A recent study suggests that men might reduce their risk of developing
prostate cancer by adopting a low-fat diet and taking regular aerobic exercise.
It was reported that the growth of androgen-dependent prostate cancer cells
in serum samples from overweight, sedentary men aged 42 73 years old
decreased by 30% after 11 days on a low-fat, high-fibre diet, and exercise
program. Serum from men of the same age who had been following the same
diet and exercise program in an extended study lasting approximately 14
years reduced cell growth by a further 15%. There was no difference in the
growth of androgen-independent prostate cancer cells in serum samples
taken before and after the program of diet and exercise.
1.
2.
3.
4.
5.

Regular exercise and a low-fat diet may help to prevent prostate cancer.
The surveyed men were fat, middle-aged to elderly and manual workers.
They went on a two-week diet and exercise program.
Urine samples were obtained from the surveyed men.
The growth of androgen-dependent cancer cells in the trial diminished by
30%.
6. There was another long-term study group of men involved.
7. The growth of androgen-independent cancer cells was reduced by 15%.
PLUS...
Complete the text with the correct form of the words in brackets:
The accepted fact that early (1) _____________ (DETECT) of prostate cancer
depends on the PSA (prostate-specific antigen) test, in which an
(2) _____________ (ELEVATE) of the protein tumor marker PSA indicates
the (3) _____________ (PRESENT) of prostate cancer, is becoming
(4) _____________ (CONTROVERSY). First, studies showed that many men
who were (5) _____________ (DIAGNOSIS) with cancer had normal levels of
PSA. Further research then suggested that the rate of increase of PSA, and
not its absolute level, was a (6) _____________ (RELY)
(7) _____________ (INDICATE) of the risk of the (8) _____________
(POSSIBLE) fatal disease.
(9) _____________ (CONSEQUENCE), new standards for interpreting PSA
readings have been (10) _____________ (SUGGESTION), but for the time
being it is felt that they would result in too many unnecessary biopsies and
cases of overtreatment.

SEXUALLY TRANSMITTED DISEASES (STD)


I Match the names of the following STDs with their short descriptions:
1
2
3
4
5

CHLAMYDIAL INFECTION
GENITAL HERPES
GONORRHEA
HPV INFECTION
SYPHILIS

A bacterial infection of the genital mucosa, but possibly of other mucous


membranes as well; characterized by urethritis with pain and purulent
discharge in men, while often asymptomatic in women; treated with
antibiotics; if untreated, can result in sterility, arthritis, meningitis and
endocarditis
B bacterial infection characterized by skin lesions called chancres in the
primary stage; treated with antibiotics; if untreated, progresses to
secondary and tertiary stages and results in degenerative changes in
various body systems
C bacterial infection, usually asymptomatic except for an occasional whitish
discharge from the penis or vagina; treated with antibiotics; if untreated,
can cause chronic inflammation, damage to the fallopian tubes and
consequently infertility in women
D viral infection; usually asymptomatic, resolves spontaneously; if
persistent, closely associated with the incidence of cervical carcinoma
E viral infection of the genital skin and mucosa characterized by small, fluidfilled blisters and ulcers, initially accompanied by inguinal
lymphadenopathy, fever, and malaise; remission and relapse periods are
typical; no effective cure

PLUS...
Researchers have developed a vaccine that appears to be 100% effective
against the two strains of HPV that cause 70% of cervical cancers.
II Read the text about SAFE SEX and mark the lines that give you the
following information:
-

which two concerns can be avoided by practising safe sex


how the number of STD cases has changed recently
why older women often put themselves at risk
why most STD infections pass undetected
why an STD infection should always be treated seriously
how the majority of new HIV infections occur
how gonorrhea is related to a possible HIV infection

An unwanted pregnancy is probably the primary concern of individuals using


condoms, but the prevention of contracting a sexually transmitted disease
(STD) should be of equal importance.
Latest figures concerning the number of new STD infections worldwide reveal
that the incidence of STDs has never been higher, and is still rising, which
means that every act of unprotected sex with a new partner can put one at
risk of catching an STD, with a range of long-term health implications.
It is not just people in their teens and twenties who are at risk. Surveys show
a surprising number of women over 30 being extremely careless when having
sex with a new partner. Older women, single after separating from a long-term
partner, comfortable using the pill, forget about other health concerns as their
sex life changes. But just taking a risk once may be enough.
According to latest statistics, only in Britain more than one million people a
year are infected with an STD. However, most of them don't even realize they
are infected, because often the infection is symptomless or symptoms are
non-specific, such as a light discharge that can be mistaken for something
more harmless, like thrush.
These infections should not be taken lightly. If left untreated, they can cause
serious long-term complications, including pelvic inflammatory disease (PID),
an infection of the female reproductive organs. PID may cause infertility
because it can result in scarring in the fallopian tubes and thus preventing the
eggs from travelling down the tube into the uterus. Damage to the fallopian
tubes causes increased susceptibility to further episodes of PID as well as the
risk of an ectopic pregnancy.
With all STDs, prompt action is the key. If identified early and treated
promptly, STDs usually clear up without any serious complications.
Experts warn that the increase in STDs indicates a pattern of unsafe sex,
particularly in heterosexuals, following the suspension of the anti-HIV/AIDS
campaigns of the nineties. More than 70% of HIV cases diagnosed recently
involve people infected through heterosexual intercourse.
Gonorrhoea, whose incidence has also risen sharply, is an indicator of the
frequency of unsafe sex. Of greater concern is the fact that gonorrhea is a
cofactor in the transmission of HIV; having gonorrhoea increases both the
chances of contracting the HIV virus and of transmitting it.
Meanwhile, the silent STD, chlamydia, is threatening to create a fertility crisis.
Recent research shows it is the biggest threat to a woman's ability to
conceive. Although
chlamydia can easily be treated with a single course of antibiotics, the
majority of people infected are unaware of their condition.
Although regular screening for STDs has been considered, so far no
extensive measures have been taken. Therefore the concern about sexual
health still stays with each individual.
III Now read about the silent STD, CHLAMYDIA, and complete the text
with appropriate words. In most cases, there are several
possibilities:
Chlamydia is easily ____________, but often goes undetected until it is too
late, causing ____________. The British Family Planning Association
____________ that as many as a third of infertility cases could be caused by
5

chlamydia. The FPA fears the official ____________ are only the tip of the
iceberg, as many of the infected are unaware of it because the infection is so
often ____________.
If symptoms occur, women may ____________ discharge or pain during sex.
Men are likely to feel burning while ____________ urine.
Chlamydia can spread quickly and unnoticed into a womans internal
genitals, where it causes the majority of cases of pelvic ____________
disease (PID). Untreated, it can damage the ____________, resulting in
infertility. In-vitro fertilization (IVF) may then be the last resort.
Chlamydia can also cause ____________ pregnancy, where the fertilized egg
____________ in the fallopian tube instead of the uterus. A single
_______________ of antibiotics is usually prescribed to ____________ the
infection.

CASE REPORT: INFERTILITY DUE TO OLIGOSPERMIA


Supply the missing medical terms from their definitions below:
A 30-year-old infertile married man presented to the clinic.
(1) ____________ analysis (SA), performed 8 times during the previous 3
years, showed a (2) ____________ ____________ (C) of 2 10 x106 per mL
(mean 3.8 x106 per mL), (3) ____________ ____________ (M) at one hour 110 % (mean 6%) and normal (4) ____________ ____________ (N) less than
10%. The volumes of the (5) ____________ were always normal, and
bacterial cultures of the semen showed no growth of any microorganisms.
Despite previous medications, the sperm counts did not improve. The couple
had a failed attempt at IVF.
The patient did not have a past history of any significant illness or sexual
dysfunction, nor had he a history of being exposed to heat or chemicals. He
reported no (6) ____________. He had a family history of infertility.
(7) ____________ was not done.
He was (8) ____________ and nondiabetic. On physical examination, he was
(9) ____________ without (10) ____________; his testes were of normal
consistency, but small in size. There was no evidence of any
(11) ____________ or any (12) ____________.
.
.
.
After taking written informed consent, the patient was given one combination
tablet of ethinyl estradiol 0.0044 mg and methyl testosterone 3.6 mg orally
daily. The patient did not report of any (13) ____________ ____________
and no problems were detected upon physical examination.
After a period of 2 months, the pelvic ultrasonography of the patients wife
showed a live intrauterine single pregnancy of seven weeks. In due time, she
delivered a boy weighing 2800 grams by cesarean section.
1
2
3
4
5
6

substance expelled during ejaculation


number of male sex cells in a given volume of semen
ability to move of male sex cells
form and structure of male sex cells
ejaculated substance
dependence on a psychoactive substance

7
8
9
10
11
12
13
13

arranging chromosomes according to a standard classification


of normal BP
showing a response to androgens (secondary male sex characteristics)
excessive growth of the male mammary glands
enlarged veins near the testis
inflammation of the epididymis
unwanted side effects
unwanted, negative side effects

URINARY SYSTEM
THE PRODUCTION OF URINE
I Put the following sentences into sequence to describe the process of
urine production:
A Each renal tubule containing urine ends in a larger collecting tubule.
B From the Bowman capsule they pass through the renal tubule, from where
most of the water, all of the sugar and some salts (sodium) return to the
bloodstream. (glomerulus + renal tubule = nephron)
C All collecting tubules lead to the renal pelvis, an area in the central part of
the kidney, which contains small cup-like regions called calices.
D Blood enters the kidneys through the renal arteries.
E In the final process of urine formation, some metabolic waste products
(acids, potassium, drugs) from the bloodstream are SECRETED into the
renal tubule.
F There are about 1 million glomeruli in the renal cortex.
G The renal pelvis narrows into the ureter, which carries the urine to the
urinary bladder, where the urine is temporarily stored before being
EXCRETED through the urethra out of the body.
H The renal arteries branch into smaller arteries. The smallest, arterioles, are
located throughout the cortex of the kidney.
I As blood passes through the glomerular walls, it is FILTERED water,
salts, sugar and urea with creatinine and uric acid leave the bloodstream,
while proteins and blood cells are retained.
J This process is called REABSORPTION.
K The filtered materials are collected in the Bowman capsule, which
surrounds each glomerulus.
L Each arteriole branches into a mass of tiny intertwined capillaries, shaped
like a ball and called glomerulus.

II Now match the following definitions with some of the bolded


structures from above:

sac-like pelvic structure for temporary storage of urine: _______________


structure surrounding the glomerulus and forming the beginning of the
renal tubule: _______________
structure that brings blood to the kidney: ______________
tubular structure leading from the urinary bladder to the outside of the
body: _______________
ball-shaped mass of capillaries in the renal cortex: _______________
structure continuous with the Bowman capsule where reabsorption occurs:
_______________
tubular structure leading from the kidney to the urinary bladder: ________
basin-like area in the central part of the kidney: ______________
basic functional unit of the kidney: _______________

tubular structure continuous with the renal tubule, leding to the renal
pelvis: _______________

DID YOU KNOW THAT...?


Work in pairs. Read the following bits of information and then take turns
to pass them on to your partner.

a hard blow to the lumbar region can cause blood in the urine, because
the kidneys are highly vascular organs that are especially susceptible to
hemorrhage?
the pain from a kidney stone (renal calculus) that has been lodged in the
ureter is one of the most extreme kinds of pain?
women are much more susceptible to urinary bladder infections (cystitis)
because the female urethra is much shorter than the male one?
since urine is sterile at the moment of leaving a healthy body, it has been
used as a disinfectant in emergency situations?
since both alcohol and caffeine are diuretics, beer is not the best choice to
quench your thirst, neither is coffee a cure for a hangover, which is caused
by dehydration?

-URIAS AND -ITISES


I In the description of urinary symptoms and disorders, there are a
number of terms containing the element URE/URI. Match the most
frequent ones with their definitions:
1
2
3
4
5
6
7
8
9
10
11
12

ANURESIS
ANURIA
DIURESIS
DYSURIA
ENURESIS

A
B
C
D
E

GLYCOSURIA
HEMATURIA
OLIGURIA
POLYURIA
PROTEINURIA
PYURIA
UREMIA

F
G
H
I
J
K
L

13 URINALYSIS

presence of serum proteins in the urine


presence of blood in the urine
decreased urine production and excretion
presence of pus in the urine
excess of urea+creatinine+nitrogenous waste in the
blood
increased excretion of urine
physical / chemical / microscopic analysis of urine
urinary incontinence (often used for bedwetting)
supression of urine formation and secretion
painful / difficult urination
urinary retention
excretion of a large volume of urine in a given
period
presence of glucose in the urine

II Write down the terms for the following INFLAMMATIONS:

inflammation of the urinary bladder


inflammation of the kidney + glomeruli
inflammation of the kidney
inflammation of the renal pelvis
inflammation of the renal pelvis + renal medulla
inflammation of the ureter
inflammation of the urethra

_______________
_______________
_______________
_______________
_______________
_______________
_______________

PROBLEMS AND PROCEDURES


Work in groups of four. Each of you should read one of the texts and do
the task involved. Then report what you have learned to the group.
1. HEMATURIA
Hematuria is the presence of erythrocytes in the urine. It can be noticed if a
large amount of blood colors the urine red, or it can be established by
microscopic analysis of the urine.
In many cases no specific cause can be found and the condition resolves on
its own. But since it can also be caused by a tumor or some other serious
problem, a doctor should be consulted. In the case of hematuria, urinalysis,
blood tests, intravenous pyelogram and cystoscopy are usually indicated.
By urinalysis, in addition to RBCs, white blood cells that signal a urinary tract
infection can be found, or casts (groups of cells molded together in the shape
of the renal tubules) that signal kidney disease. Excessive protein in the urine
also signals kidney disease.
Blood tests may reveal kidney disease if the blood contains high levels of
nitrogenous wastes that the kidneys are supposed to remove.
An intravenous pyelogram (IVP) is an X-ray of the urinary tract. An IVP may
reveal a tumor, a kidney or bladder stone, an enlarged prostate, or other
blockage to the normal flow of urine.
Cystoscopy, the visual examination of the urinary bladder, may provide a
better view of a tumor or bladder stone than can be seen in an IVP.
Treatment for hematuria depends on the cause. If no serious condition is
causing it, no treatment is necessary.
Medical terms for:
protein in the urine
kidney stone
bladder stone

_________________________
_________________________
_________________________

10

2. CYSTOSCOPY
In the case of a urinary problem, a cystoscopy, the visual examination of the
urinary bladder through the urethra, may be indicated. The cystoscope, a thin
tubular instrument with a light at the tip, allows the doctor to focus on the inner
surfaces of the urinary tract by means of lenses. Many cystoscopes have
extra tubes to guide other instruments for procedures to treat urinary
problems.
Concerning the procedure itself, it requires no special preparation, apart from
signing a consent form, since it involves a small risk of injury. After gently
inserting the tip of the cystoscope into the urethra, the doctor will slowly glide
it up into the bladder. A sterile liquid (water or saline) will flow through the
cystoscope to slowly fill the patients bladder and stretch it so that the doctor
has a better view of the bladder wall.
As the bladder reaches capacity, the patient will feel some discomfort and the
urge to urinate.
The time from insertion of the cystoscope to removal may be only a few
minutes, or it may be longer if the doctor finds a stone and decides to remove
it. Taking a biopsy will also make the procedure last longer. In most cases, the
entire examination, including preparation, will take about 15 to 20 minutes.
After the procedure, the patient may have a mild burning feeling when
urinating, and may see small amounts of blood in the urine. However, these
problems should not last more than 24 hours.
Medical term for urination ___________________
biopsy = ___________________

3. CHRONIC KIDNEY DISEASE (CKD)


Chronic kidney disease (CKD) is the permanent loss of kidney function. It may
be the result of physical injury or a disease that damages the kidneys, such as
diabetes or hypertension.
CKD is a hereditary disorder, so individuals with a blood relative who suffers
from it may be at risk. It is also a silent condition. In the early stages, it is
asymptomatic and it often develops so slowly that many patients don't realize
their problem until the disease is advanced, in which case cardiovascular
problems may appear without the patient realizing the underlying problem of
CKD.
Apart from the genetic component, the biggest risk factors for CKD are
diabetes and high blood pressure. So a screening for these conditions by
means of simple medical blood and urine tests makes sense.
If any of these conditions or CKD has been established, they have to be
controlled through medication and diet, as well as regularly and carefully
monitored to slow down the damage to the kidneys, retain their function as
long as possible and prevent kidney failure.
Since CKD is on the increase in developed countries, and thus a considerable
burden on health budgets, many research programs have been started with
the aim of slowing down its progression.

11

Explain:
hereditary
= ______________________________________
asymptomatic = ______________________________________

4. HEMODIALYSIS
Dialysis is a treatment for people in the later stage of chronic kidney disease
(kidney failure). This treatment cleans the blood and removes wastes and
excess water from the body, which is normally done by healthy kidneys.
Sometimes dialysis is a temporary treatment. However, when the loss of
kidney function is permanent (as in end-stage kidney failure), it must be
continued on a regular basis. The only other treatment for kidney failure is a
kidney transplant.
There are two types of dialysis: hemodialysis and peritoneal dialysis. In
hemodialysis, the patients blood is passed through an artificial kidney
machine to clean it. Peritoneal dialysis uses a filtration process similar to
hemodialysis, but the blood is cleaned inside the body rather than in a
machine.
Hemodialysis means cleaning the blood - and that is exactly what this
treatment does. Blood is circulated through a machine which contains a
dialyzer (also called an artificial kidney). The dialyzer has two spaces
separated by a thin semipermeable membrane. Blood passes on one side of
the membrane and dialysis fluid passes on the other. The wastes and excess
water pass from the blood through the membrane into the dialysis fluid, which
is then discarded. The cleaned blood is returned to the patients bloodstream.
Each hemodialysis treatment normally takes four to five hours, and usually
three treatments a week are needed. More frequent, shorter treatments or
longer treatments may be indicated for certain patients. Only a small amount
of the patients blood is out of the body at one time. Therefore the blood must
circulate through the machine many times before it is cleaned.
Explain:
peritoneal =
___________________________________________
semipermeable = ___________________________________________

CASE REPORT: ACUTE RENAL FAILURE


Read the case report and explain the medical terms in italics using
general English. Supply the missing medical terms concerning urinary
symptoms from their definitions below:
History: A 48-year-old female presented to the emergency department
reporting nausea, vomiting, anorexia, chills, myalgia, arthralgia and headache.
She denied fever, cough, exanthem, diarrhea, (1) ____________ or
(2) ____________. She also reported sharp left flank pain radiating to her
neck, shoulders and chest, which occurred three consecutive nights and then
spontaneously resolved. The day prior to presentation, the patient reported
abdominal distention and (3) ____________.
12

The patient's past medical history included hypertension, thyroidectomy, mitral


valve prolapse and gastric bypass surgery.
Physical exam: On examination, the patient was afebrile, BP 149/82, pulse
65, RR 16 with an oxygen saturation of 100%. Oral mucosa was moist without
lesions. Neck was supple without lymphadenopathy. Lungs were clear to
auscultation bilaterally. Abdomen had normal active bowel sounds, mild
epigastric tenderness and was not distended. Musculoskeletal exam revealed
no joint erythema or tenderness and normal range of motion of all joints.
Neurological exam was without focal deficits.
Differential diagnosis: Initial differential diagnosis included
(4) ____________ as the patient had reported flank pain and a recent
decrease in urination.
.
.
.
As the patient had no history of (5) ____________ ____________,
investigation into her acute renal failure was initiated.
The patient had no rash or new medication use that could suggest
(6) ____________ ____________ ____________. A
(7) ____________ or obstructive lesion remained the most likely reason for
the patients acute renal failure.
A computerized tomography of the abdomen and pelvis was performed and
revealed a right kidney with moderate to severe (8) ____________ with a 1cm
stone at the pelvic inlet and left moderate (8) _____________ with several
(9) ____________ including an 8 mm stone. This established the diagnosis of
(10) ____________ ____________ ____________ resulting in acute renal
failure.

1
2
3
4
5
6
7
8
9
10

presence of blood in the urine


painful / difficult urination
decrease of frequency and quantity of urination
presence of kidney stones
disordered function of the kidneys verified by tests
sudden-onset inflammation of the renal interstitium
occurring after leaving the kidney
distention of the kidney with urine as a result of obstruction of the ureter
stones
blockage of the ureters on both sides

13

FEMALE REPRODUCTIVE SYSTEM


MENSTRUAL CYCLE
Put the passages describing the menstrual cycle into sequence:
A In mid-cycle the graafian follicle ruptures and releases an ovum in the
process called ovulation.
B During the period itself, resulting from a drop in ovarian secretion of
estrogen and progesterone, disintegrated endometrial cells with glandular
secretions and blood cells contained in a fluid are discharged from the
uterus.
C The secretory phase occurs when the ovary is in its luteal phase.
D The ovum leaves the ovary and passes into the fallopian tube.
E If fertilization does not occur, the corpus luteum stops hormone production,
which after a few days induces the next menstrual period.
F The ovum grows in the graafian follicle.
G The proliferative phase of the endometrium occurs while the ovary is in its
follicular phase, releasing estrogen which helps in the repair of the uterine
lining.
H Each menstrual cycle, lasting approximately 28 days, can be divided into 4
phases.
I Menarche, the first menstrual cycle, occurs at the onset of puberty.
J The empty graafian follicle fills with an adipose substance and becomes the
corpus luteum, which starts secreting estrogen and progesterone in
anticipation of fertilization and pregnancy.
PLUS...
In pairs, read 2 items each and exchange information.

The average age of menarche is later in girls who are very active
physically, due to a requirement for a minimum percentage of body fat for
menstruation to begin.
Menstrual cycles as long as 36 days or as short as 21 days are considered
within the normal range.
Although most women dont know when exactly their ovulation occurs,
about 30% of them experience a sharp, cramplike pain at the time of
ovulation, which is sometimes confused with the symptoms of appendicitis.
Amenorrhea, the absence of menstruation, is classified as normal,
primary, or secondary. Normal amenorrhea occurs during pregnancy
(sometimes also during lactation) and after menopause. Primary
amenorrhea is the absence of menarche, usually due to endocrine
disorders. Secondary amenorrhea occurs in women who have
menstruated previously, and can be due to various endocrine
disturbances, but also to psychological causes, intense training, or eating
disorders (anorexia nervosa), which considerably reduce the percentage
of body fat.

14

PREGNANCY AND BIRTH

I DID YOU KNOW THAT?

in pregnancy the uterus, a fist-sized organ, enlarges to occupy practically


the whole abdominal cavity, becoming 16 times heavier in the process?
the amount of milk the mammary glands produce during lactation is not
related to breast size, because their size depends only on the amount of
fatty tissue deposited at puberty?
milk flow can be a conditioned reflex in response to, for instance, the
babys crying; but it can also be supressed by adrenergic effects caused
by stress and anxiety?

II Vocabulary check - to complete each sentence, choose an adjective +


Noun combination from the lists below:
Adjectives: AMNIOTIC, BIRTH, BREECH, DILATED, ECTOPIC,
FALLOPIAN, FETAL, MENSTRUAL, MULTIPLE, PREMATURE,
UMBILICAL, UTERINE
Nouns:

CANAL, CERVIX, CONTRACTIONS, CORD, FLUID, INFANT,


MONITORING, PERIOD, POSITION, PREGNANCY (2x), TUBES

1. Fertilization normally occurs in the __________ __________, except in the


case of an __________ __________.
2. A full pregnancy lasts 280 days from the first day of a womans last
__________ __________.
3. The result of ovarian hyperstimulation as part of infertility treatment may
result in a __________ __________.
4. The fetus is connected to the placenta by the __________ __________.
5. In a procedure called amniocentesis, a sample of __________ _________
is obtained for testing if chromosomal abnormalities are suspected.
6. Labor is the term for __________ __________ during delivery.
7. __________ __________ means checking the babys heart rate during
labor.
8. During delivery, the baby passes through the fully __________ _________
Into the __________ __________.
9. One of the possible complications during delivery may be the result of a
__________ __________.
10.A baby born before full term and weighing less than 2,500 g is considered
to be a __________ __________.

15

III Look at some advice for pregnant women on how to REDUCE THE
RISK OF AN EMERGENCY C-SECTION and supply the missing
words:
Before the birth:

Take early ____________ classes, from around 30 weeks.

Ask about the hospital policy, e.g. if you can walk during ____________.
Do __________ exercises to train and firm the muscles which will be
involved in the process of giving birth. A yoga class for pregnant women
may also be a good idea.
For a few weeks before the birth, massage the ____________, i.e. the
area between the vagina and anus. Massaging with vegetable oil
encourages the skin to stretch to double its normal capacity.
If your baby is ____________, ask about external version and vaginal
delivery in spite of this position.
If you are significantly ____________ before pregnancy, get advice on an
appropriate eating plan.

During labor:

Eat and drink lightly to sustain your energy: a ____________ in blood


sugar disrupts the flow of labor.
Ask for ____________ fetal monitoring, because constant monitoring
increases the likelihood of unnecessary intervention.
If possible, dont be on your ____________ during the whole labor.

IV Look at the jumbled PROS AND CONS OF C-SECTIONS for mother


and baby and sort them out below:
-

increased risk of death (up to 11 times in an emergency Cesarean)


fewer minor musculoskeletal problems
longer recovery period
less risk of infection if the mother has HIV or herpes
obstetrician of the womans choice
post-operative pain
possible head and lung problems, as the squeezing process during vaginal
birth is thought to encourage the lungs to work properly
reduction in pelvic-floor damage
possible emergency hysterectomy
less pain and tiredness
major surgery, with potential for infections, endometriosis, internal
adhesions, thrombo-embolism
potential future pregnancy problems
more control; convenient

16

PROS for the mother (4):


PROS for the baby (2):
CONS for the mother (6):
CONS for the baby (1):
V Put the verbs below where they belong in the following passage
describing a C-SECTION, using the Present Simple passive
throughout:
CLAMP
DRAIN

CLEAR
HAND

CUT
MAKE

DELIVER
OPEN

DO
PERFORM

A C-section delivery _______________ when a vaginal birth is not possible or


is not safe for the mother or child.
Surgery ____ usually _______________ while the woman is awake but
anesthetized from the chest to the legs by epidural or spinal anesthesia. An
incision _______________ across the abdomen just above the pubic area.
The uterus _______________, the amniotic fluid _______________, and the
baby _______________.
The babys mouth and nose _______________ of fluids, and the umbilical
cord _______________ and _______________. The baby _______________
to the pediatrician or nurse who will make sure that breathing is normal. The
mother is awake and she can hear and see her baby.
Due to a variety of medical and social factors, C-sections have become
quite common about 26% of all births in the US in recent years have
been C-sections.

VI CASE REPORT: ECTOPIC PREGNANCY


Explain the underlined medical terms using general English:
Diagnostic and therapeutic (1) laparoscopy was indicated and, as expected,
(2) preoperative pelvic exploration showed an extensive (3) hematocele in the
ampullar region of the left tube. Initial (4) salpingostomy confirmed the
(5) hemorrhagic content of the tube, which supported the diagnosis of left
tubal pregnancy. Rapid inspection of the (6) contralateral adnexa identified a
slight (7) tumefaction in the (8) interstitial portion of the right tube. This aspect
suggested a (9) minor tubal alteration and was therefore neglected. On
account of the extended tubal injury, we decided to do a left
(10) salpingectomy. The patient left hospital with a noticeable improvement of
her clinical symptoms.
1 ___________________________________________________________
2 ___________________________________________________________
3 ___________________________________________________________
4 ___________________________________________________________

17

5 ___________________________________________________________
6 ___________________________________________________________
7 ___________________________________________________________
8 ___________________________________________________________
9 ___________________________________________________________
10 ___________________________________________________________
VII Translate:
Trudovi i porod mogu izazvati simptome posttraumatskoga stresa, koji se
obino povezuje s ratom i drugim oblicima nasilja, izvjeuju britanski
psiholozi. Ustanovili su da treina majki proivljava strah, tjeskobu i none
more, te da ih jo tjednima poslije poroda progone neugodne misli i
flashbackovi, posebno ako je trudnoa bila neeljena ili se ena bojala za
svoje zdravlje.
Nedavno je objavljena prva opsena studija koja je trudove i porod povezala s
traumom. Ispitane su 264 ene 72 sata poslije poroda i ponovno nakon 6
tjedana. U drugom ispitivanju etvrtina ena pokazivala je znakove
zabrinutosti, razdraljivosti ili nervoze, a tri posto ena pokazivalo je jake
simptome posttraumatskoga stresa.
FERTILITY
I In groups of 3, read the short description of one of the METHODS OF
ART (ASSISTED REPRODUCTION TECHNOLOGY) each, and then
report to the group.
Together, decide which underlying problem might be the basis for
each of these procedures:
In vitro fertilization (IVF)
A woman's eggs are extracted and mixed with her partner's sperm in a Petri
dish. The resulting embryo is transferred to her uterus through the cervix.
At least 60,000 IVF procedures are performed in the US annually, with an
average birthrate of 25%.
Intracytoplasmic sperm injection (ICSI)
To counteract problems with sperm count, quality or mobility, doctors inject a
single sperm directly into a mature egg to increase the chance of fertilization.
ICSI accounts for approximately 24,000 IVF procedures annually. Average
birthrate: 30%.
Egg donation
When the problem is aging eggs, a young woman may donate her eggs to a
couple. Fertilized with the man's sperm, the resulting embryo is implanted in
the older woman's womb.
More than 5,000 eggs are donated yearly. After the eggs are fertilized, the
birthrate is approximately 40%.

18

Now discuss the ETHICAL IMPLICATIONS OF EGG DONATION AND


SURROGATE MOTHERHOOD. European legislation is much more
restrictive than American concerning these matters do you think that
things should change?

II Read about the RISKS INVOLVED IN PREGNANCIES RESULTING


FROM ART, and make a list of potential problems that may arise for
The mother and the baby:
Assisted reproductive technology is one of the great medical success
stories of the late 20th century. Thanks to fertility drugs, IVF and a growing
number of even more sophisticated techniques, tens of thousands of healthy
babies are born each year that otherwise might never have been conceived.
But the process is neither simple nor risk free. There are limits to what
science can do for infertile couples, and the more doctors have to intervene
with drugs, needles and surgery to get sperm to meet egg, the greater the
chance that something will go wrong. To list just a few potential problems:
Ovarian hyperstimulation: The first step in most assisted-fertilization
techniques is to stimulate the ovaries to produce a lot of eggs at once. But the
hormones doctors use to do this are powerful drugs and in rare cases can
cause serious complications, including blood clots and kidney damage.
Multiple gestation: About 20% to 35% of IVF pregnancies produce multiple
fetuses, usually twins. Having more than two or three babies at once is often a
medical disaster. Babies that develop in a crowded uterus or are born too
early are at risk for a lifetime of developmental problems, including mental
retardation, paralysis and blindness. Trying to reduce the number of fetuses
through selective abortion has its own problems, not the least of which is an
increased chance of miscarriage.
Low birth weight: Twins and triplets (not to mention septuplets) often weigh
less than normal at birth. But a recent study suggests that even single babies
conceived through IVF are more likely to be born underweight. Whether that
also puts them at greater risk of developmental problems is uncertain.
Birth defects: Although it has been reported that IVF children are twice as
likely to suffer birth defects such as cleft palate, a hole in the heart or kidney
problems as children conceived the usual way, several earlier studies have
shown no differences between the two kinds of babies, so further research is
needed. But even if the apparent increase is real, it might not be clear
whether the birth defects are caused by the artificial reproductive technology
or by the underlying problem that caused the infertility.
Potential problems for the mother:
Potential problems for the baby:

19

CONTRACEPTION
I Put the following CONTRACEPTIVE DEVICES AND MEASURES into
the categories below. In small groups, discuss what you know about
each of them:
CONDOM
INTRAUTERINE DEVICE (IUD)
CONTRACEPTIVE PATCH
MORNING AFTER PILL
DIAPHRAGM
ORAL CONTRACEPTION (THE
PILL)
FEMALE CONDOM
SPERMICIDE (CREAM, FOAM,
GEL)
HORMONAL IMPLANT
STERILIZATION (TUBAL
LIGATION, VASECTOMY)
INJECTABLE CONTRACEPTION
(THE JAB)
Hormonal methods (5):
Long-term mechanical methods (2):
Barrier methods (4):
II Read about the ORAL CONTRACEPTIVE PILL (OCP) and supply the
missing words:
The female sex hormones ____________ and ____________are two
hormones which direct many of the processes surrounding the ____________
cycle. Artificial analogues of these have proven an efficient form of
____________ control. To ____________ pregnancy a woman takes a pill
daily which ____________ both of these hormones. This is the combination
pill, or simply the pill.
The estrogen works by preventing an egg from being ____________ from the
ovaries (i.e. ____________) most of the time. Both hormones make the
____________ a hostile environment for a(n) ____________ by causing a
thinning of the uterine ____________. As modern combination pills contain
less estrogen than their predecessors, an egg will leave the ovaries 2 10%
of the time. If ____________occurs, the embryo will be unable to
____________ in the uterus, resulting in its death.
Oral contraceptives also have some uncommon but serious risks associated
with their use, especially among smokers; these include abnormal blood
____________ and heart attacks, cancer, and gallbladder disease.
____________ effects include headaches, acne, weight ____________,
vaginal infections, and depression.

20

III As you read about the INTRAUTERINE DEVICE (IUD), supply the
correct forms of the words in brackets:
The intrauterine device is a small plastic or metal device that is placed inside
the uterus by a gynecologist for an (1) ____________ (EXTENSION) period
of time.The IUD prevents (2) ____________ (IMPLANT) of an embryo and
makes it more difficult for sperm to enter. The copper in some types of IUD
has a (3) ____________ (SPERMICIDE) effect, and recent studies cite the
(4) ____________ (INTERCEPTION) action as a major factor. To make the
IUD more (5) ____________ (EFFECT), some contain progestin, which also
(6) ____________ (INTERFERENCE) with implantation. It has also been
stated that the IUD may mechanically dislodge an embryo after implantation.
The IUD carries with it a number of serious health risks, which have caused
many manufacturers to stop (7) ____________ (DISTRIBUTE). These risks
include pelvic (8) ____________ (INFLAMMATION) disease, permanent
(9) ____________ (INFERTILE), and ectopic pregnancy. Because the risks to
fertility, the IUD is usually not recommended to women who havent had
children yet.
IV Translate:

PRVI KONTRACEPCIJSKI FLASTER ODOBREN ZA PRODAJU


Tvrtka Johnson & Johnson napokon je od amerike Agencije za hranu i
lijekove dobila dozvolu za prodaju kontracepcijskog flastera nazvanog Evra,
prvog takvog proizvoda u svijetu.
Flaster djeluje tako da u kou ene isputa hormone koji ulaze u krvotok i tako
sprjeavaju zaee.
Evra je izazvala golemo zanimanje u cijelom svijetu im je objavljeno da je
proizvedena.
V Which METHOD OF CONTRACEPTION would you suggest in the
following cases? (Dont forget the issue of STDs!) Give arguments for
your suggestions:

Teenage couple, infrequent sex.


Woman in her 20s, no permanent partner, working on career, no family
plans for the next 10 years.
Married couple in their 30s, 1 child, possible further pregnancy, but not
immediately.
Single bisexual man in his 40s, frequent sex with varying partners, no
family plans.
Married couple with children in their 40s, woman smokes, no more
pregnancies wanted.

21

MENOPAUSE
I DID YOU KNOW THAT...?

contrary to other mammals, only human females lose their ability to


reproduce? Females of wild animals in zoos, which live to an advanced
age, continue to breeed, although less frequently.

II Read about the largest long-term study into the EFFECTS OF


HORMONE REPLACEMENT THERAPY (HRT), parts of which had to be
shut down because of some preliminary findings that raised concern.
The Womens Health Initiative (WHI), begun in 1991 by the US National
Institutes of Health, is one of the largest studies of women's health ever
undertaken. More than 160,000 post-menopausal women ages 50 to 79 were
involved into a variety of clinical trials designed to find the best ways to
prevent heart disease, breast and colorectal cancers, and osteoporosis. Due
to the study's strict design, most doctors view the WHI as the definitive word
on women's health. Final results were due to be published in 2005. But one
part of the study, involving more than 16,000 women taking a combination of
estrogen and progesterone called hormone-replacement therapy, was
stopped in 2002, after researchers concluded that the risks of HRT clearly
outweighed the benefits (see table below).
In 2004 another arm of the study was shut down involving women who had
had a hysterectomy and were on estrogen therapy without progesterone,
because it was established that taking estrogen alone also raises a womans
risk of stroke and blood clots.
Although other parts of the giant WHI study still continue, doctors have come
to the conclusion that HRT may be appropriate as a short-term therapy for
menopausal distress, relieving hot flashes and contributing to bone health,
rather than as long-term protection against aging-related diseases. It seems
that after two years the benefits no longer outweigh the risks.
III Look at some of the STATISTICS resulting from the WHI study on
HRT. More cases of certain disorders were reported among women
taking HRT than among those in the placebo group, although the
absolute risk for an individual woman is quite small. HRT proved
beneficial for some other disorders.
Write three sentences based on the data shown.

22

Cases per 1,000 women per year


HRT

Placebo

Difference

Breast cancer

3.8

3.0

+ 26%

Heart disease

3.7

3.0

+ 23%

Stroke

2.9

2.1

+ 38%

Blood clots

2.6

1.3

+ 100%

Hip fractures

1.0

1.5

- 33%

Colon cancer

1.0

1.6

- 37%

1. __________________________________________________________
2. __________________________________________________________
3. __________________________________________________________

23

THE EAR
HOW WE HEAR
Most of the missing words in this simple description of the process of
hearing are the MAIN STRUCTURES OF THE EAR:
Sound travels down the ____________ ____________ and strikes the
____________, which is taut like the skin of a real drum, so it ____________
when sound waves hit it. The motions are passed down the ____________
(____________, ____________ and ____________) to the ____________ ,
which is the main structure in the ____________ ____________. The
vibrations make the ____________ in the ____________ move. This
movement in turn makes the ____________ ___________ move, thus
producing tiny ____________ ____________ which are picked up by the
____________ ____________. The cells at one end of the cochlea send low
pitch sound information in these signals and those at the other end send high
pitch sound information. These electrical signals pass up the ____________
____________ to the ____________, which interprets them as
____________.

DID YOU KNOW THAT...?

a ruptured tympanic membrane can generally regenerate within days?


there are two basic types of deafness: conduction deafness, caused by an
interference with the sound waves in the outer or middle ear resulting from
trauma or pathological conditions, usually improved by medical treatment,
and perceptive deafness, caused by disorders of the inner ear or the
auditory centers within the brain, which is permanent, and in some cases
may be helped with hearing aids?
permanent exposure to noise can be a serious health hazard resulting
not only in impaired hearing or hearing loss (from frequent or prolonged
exposure to sounds above 90 decibels), but also in hypertension, stress
and depression?

COCHLEAR IMPLANTS

I In this short explanation of HOW COCHLEAR IMPLANTS WORK,


Supply the appropriate verbs from the list. Be sure to use the correct
forms (active or passive):
CODE, CONSIST, PICK UP (2x), RECOGNIZE, SEND (2x), TRANSMIT (2x),
TRAVEL, TURN

24

The whole device _____________ of an outer (microphone, speech


processor, coil) and an inner part (the implant itself).
Sounds _____________ by a microphone and _____________ into an
electrical signal.
This signal _____________ to the speech processor where it is
_____________ (turned into a special pattern of electrical pulses).
These pulses _____________ to the coil and then ____________ across the
intact skin (by radio waves) to the implant.
The implant _____________ a pattern of electrical pulses to the electrodes in
the cochlea.
The auditory nerve _____________ these tiny electrical pulses and
_____________ them to the brain.
The brain _____________ these signals as sounds.

II This is the description of a COCHLEAR IMPLANT OPERATION. Put


the 10 steps into sequence:
A The skin and tissue are reattached, and the wound is stitched up.
B The incision is made and the skin and tissue are lifted back to expose the
skull.
C A hole is drilled into the cochlea.
D A general anesthetic is normally given.
E Patients are usually up and about the next day. The length of stay in the
hospital depends on local practice and can be as short as 3 days.
F The electrode array and the implant itself are secured in place.
G There is usually little discomfort when the patient wakes up. Pain
medication can normally be given if required.
H A bed is drilled out in the bone behind the ear for the implant.
I The skin is shaved around where the incision is to be made.
J The electrode array is inserted into the cochlea.
III Now translate the procedure. What happens to all the passives in
Croatian?
IV As you read about the past and present of cochlear implants, mark
the passages that give you the following information:
-

the breakthrough realization in the 1970s that is the principle on which


cochlear implants work
the main difference between the first generation of cochlear implants and
todays devices
why time matters in implants for children who were born deaf
drawbacks that still need to be improved

25

BIONICS - NEW HOPE FOR THE DEAF?


The idea of electrically stimulating the auditory system is nothing new. In the
18th century, Italian physicist Count Volta connected two metal rods to his
most famous invention, the battery, and inserted them into his healthy ears,
generating some sort of sound.
By the 1970s, scientists had found out that electrical pulses needed to be
targeted to localized groups of hair cells which transmit different sets of tonal
and pitch information to the brain. While the first generation of cochlear
implants used a single, clumsy electrode to convey the whole spectrum of
sound, todays devices have up to 24 electrodes, each stimulating a different
group of neurons.
Cochlear implants first appeared on the market in 1985, but they were
controversial through the mid-90s, partly because they improved hearing in
only 30% to 40% of patients. Today, technology has advanced considerably.
Doctors say they can now significantly enhance hearing in 80% of patients,
and even children born deaf are candidates for the procedure. But they should
be treated within their first five years, before the brain loses its ability to
process sound.
The results still arent perfect. People with the implant often say voices sound
metallic, like a radio broadcast. And few are able to enjoy the tonal richness of
music. Many also still lip-read to complement their new hearing, particularly in
loud environments - though they are able to use the telephone, where theres
little background noise.

26

ENDOCRINE SYSTEM
WHICH GLAND IS IT?
I Match the GLANDS (numbers) with their LOCATIONS (capital letters)
and FUNCTIONS (small letters):
1. ADRENAL GLANDS
2.
3.
4.
5.

OVARIES
PANCREAS (ISLETS OF LANGERHANS)
PARATHYROID GLANDS
PINEAL (EPIPHYSIS)

6. PITUITARY
(HYPOPHYSIS)
7. TESTES
8. THYMUS
9. THYROID

A in the sella turcica, at the base of the brain


B on either side of the trachea, below the cartilage covering the larynx
C on top of each kidney
D in the mediastinum, behind the sternum
E on either side of the pelvic cavity in a female
F in the central portion of the brain
G suspended from the inguinal region, inside the scrotal sac
H on the dorsum of the thyroid
I partially behind the stomach, in the region of L1 and L2

a
b
c
d

important in the development of immune responses in newborns


stimulates the growth and hormone secretion of other glands
maintains a normal level of metabolism in all body cells
responsible for the metabolism of sugars, fats and proteins, the balance of
electrolytes, the development of secondary sex characteristics;
sympathomimetic action
e production of male gametes
f production of female gametes
g mobilization of calcium from bones into the bloodstream
h responsible for the proper metabolism of sugars and starches in the body
i supports the bodys biological clock

II Write down the name of the gland(s) which produces the hormones
below:

______________________:
______________________:
______________________:
______________________:
______________________:
______________________:

PTH
thymosin
estrogens, progesterone
STH, TSH, ACTH, FSH, LH, PRL; ADH, OT
T4, T3, calcitonon
corticosteroids, catecholamines
1

______________________: testosterone
______________________: insulin, glucagon
______________________: melatonin

III Which endocrine tissues, apart from the major glands, secrete the
following hormones / hormone-like substances? What do you know
about their functions?

cholecystokinin ____________________________
erythropoietin ____________________________
gastrin
____________________________
HCG
____________________________
prostaglandins ____________________________
secretin
____________________________
vitamin D
____________________________

PLUS...

The endocrine system is different from all other body sytems in that its
glands are widely scattered over different locations in the body, and not in
any way physically connected.

VOCABULARY CHECK complete the following sentences with the


words below:
ACCELERATED
BIND
CONSTANT
CONTINUOUSLY
CYCLICALLY

FLUCTUATE
HYPERSECRETION
HYPOSECRETION
INHIBITED

INTERMITTENTLY
MAINTAIN
REGULATE
SECRETE

1. Endocrine glands ____________ their products directly into the


bloodstream.
2. Hormones ____________ to receptors, recognition sites in their various
target tissues.
3. The activity of the target tissues is either ____________ or ____________
under the influence of hormones.
4. Some hormones are produced ____________, while others are
released ____________, or, as is the case with female sex hormones,
_____________.
5. Hormone levels arent ____________, but ____________ within certain
limits.
6. Some hormones ____________ the balance of important substances like
electrolytes, others ____________ metabolic processes.
7. Endocrine disorders usually occur due to either ____________ or
____________ of certain glands.

ENDOCRINE DISORDERS
Put the following disorders into the table below:

ACROMEGALY
ADDISON DISEASE
CRETINISM
CUSHING DISEASE
DIABETES INSIPIDUS
DIABETES MELLITUS
DWARFISM
GLAND
Adrenal
Pancreas
Parathyroid
Pituitary

ENDEMIC GOITER
EXOPHTHALMIC GOITER
GIGANTISM
GRAVES DISEASE
HYPERINSULINISM
HYPERPARATHYROIDISM
HYPOPARATHYROIDISM
HYPERSECRETION

HYPOSECRETION

Thyroid

CASE REPORT: GRAVES DISEASE (HYPERTHYROIDISM)


Supply the missing medical terms from their definitions below:
The 42-year-old female patient presented with an immediate past history of
permanent nervousness, (1) ____________, (2) ____________ of
depression, (3) ____________ insomnia and (4) ____________.
The physical exam revealed (5) ____________, palmar
(6) ____________, brisk reflexes and (7) ____________ with atrial
(8) ____________. The classical (9) ____________ was also present.
The patients skin was characteristically soft and smooth as well as warm and
moist from peripheral (10) ____________. Examination of the
(11) ____________ revealed (12) ____________ patches and plaques over
the (13) ____________ of the legs and feet.
1
2
3
4
5
6
7
8

unpleasant emotional state of fear, powerlessness and tension


accompanied with appropriate physiological reactions
episodes, attacks
occurring at intervals
absence of menstrual period
involuntary trembling
redness of the skin
rapid heartbeat
rapid, irregular contractions

9
10
11
12
13

abnormal protrusion of the eyeball


dilation of blood vessels
legs and feet
of abnormally increased pigmentation
back side

DIABETES
I Put the passages about NORMAL AND DIABETIC SUGAR
METABOLISM into sequence:
A Insulin-resistant cells can't create the portals needed to absorb glucose
from the blood.
B Normally, insulin molecules bind to receptors on muscle cells, preparing
them to absorb glucose from the blood.
C As glucose is absorbed and burned by muscle cells, the level in the
bloodstream quickly returns to normal.
D Any increase in blood glucose causes the pancreas to release insulin, a
hormone that helps the cells absorb the sugar.
E Many foods contain carbohydrates, chains of sugar molecules of
different shapes and sizes.
F In the case of diabetes, overworked receptors on cell surfaces become less
responsive to insulin.
G As carbohydrates enter the small intestine, they break down into glucose, a
Simple sugar that can enter the bloodstream.
H Consequently, glucose builds up in the bloodstream, obstructing vital
Processes and damaging vessel walls.
I Activated by insulin, the muscle cells create portals which glucose
molecules can enter.
II In pairs, go through the CHECKLIST OF WARNING SIGNS FOR
DIABETES and ask each other about possible symptoms. Be sure to
use the correct question forms!
e.g. Do you feel hungry and / or thirsty all the time?

constant thirst or hunger


frequent urination
blurred vision
numb or tingling extremities
frequent skin infections
slow healing of cuts and bruises

III The World Health Organization predicts that DIABETES CASES


WORLDWIDE will more than double by the year 2025 and reach
300 million. Scan the paragraphs below and mark the key words that
helped you decide which of them talk about the following:

1 type-1 diabetes
2 type-2 diabetes
3 physiological causes of diabetes and its symptoms
4 long-term effects of diabetes
5 the spread of diabetes relating to onset-age
6 diabetes and obesity
7 genetic factors
8 new drug therapies
9 recommended diet and lifestayle
A
The correlation between type-2 diabetes and obesity is overwhelming: as
people get fatter, the risk of diabetes goes up dramatically. The exact nature
of the relationship is extremely complex and not yet fully understood, but the
simplest way to think about it may be that for unknown reasons, the same
factors that make you fat - lack of exercise and a high-calorie diet - also put
you at risk for diabetes.
B
Researchers are still investigating all the ways in which high blood-sugar
levels do damage. One obvious effect is on the arteries, especially in the
eyes, kidneys and extremities; sugar seems to both weaken the capillary walls
and clog the small vessels. Hemorrhages destroy the retina; impaired
circulation leads to ulcers in the legs and feet for which amputation may be
the only cure. The risk of heart disease doubles for men and increases four
times for women.

C
Most alarming is the spread of diabetes across age barriers. The enormous
rise in type-2 disease, which has so far been considered the adult variety,
among teenagers should be a great concern because diabetes can take
decades to reveal its effects including ulcerating sores, blindness, kidney
failure, strokes and heart disease. The implications for health-care systems
are obvious.
D
With conscientious monitoring of their blood sugar, regular exercise and the
right attitude, many diabetics can now allow themselves an occasional sweet,
if it is part of the same low-fat, high-fiber, low-calorie diet that researchers
recommend for practically every other major health problem. Sophisticated
patients don't just stick to a diet, but monitor what they eat strictly, measuring
their blood-sugar levels with a blood-glucose meter and a drop of blood from
a finger as often as five times a day.

E
Type 2 is a more complicated disease, a spiraling disorder linking the
pancreas, liver (which stores and releases glucose), muscles, nerves, fat cells
and brain. In diabetics, the muscle cells refuse to absorb glucose from the
blood, a phenomenon called insulin resistance. At least in the early stages of
the disease, type-2 diabetics usually have normal insulin production. They
may even have above-normal insulin, as their pancreas produces more and
more of it in an unsuccessful attempt to keep up with the rise in blood sugar.
However, gradually patients may need more insulin than their pancreas can
supply and so become dependent on insulin injections.
F
The complexity of the glucose-insulin cycle provides various ways to intervene
with therapies. The obvious therapy is insulin. For years the only available
form was harvested from cows or pigs, but now human insulin is being
manufactured directly by recombinant DNA techniques. Besides, drug
companies are coming out with new and improved insulin, engineered with
molecular changes to make it last longer in the body or be absorbed more
easily into cells.
For decades, many diabetics have been taking some form of drugs that
stimulate production and release of insulin by the pancreas. But newer drugs,
available for the last few years, offer far more possibilities for control.
Glucophage is one - it controls blood sugar directly by promoting glucose
storage in the liver. A class of drugs called TZDs make muscle and fat cells
more sensitive to insulin, fighting the disease right at the source. And there
are drugs that work in the gut to inhibit starch digestion, slowing the process
enough to flatten the glucose spike.
G
Genetics seems to play a role in this disorder as well. Recently researchers
have identified a variant form of a gene on human chromosome 1 that
appears to increase the risk of type-2 disease by about 25 %.
H
The relatively uncommon type-1 diabetes is marked by a straightforward
shortage of insulin, which typically occurs around puberty, and is due to a
hyposecretion of the posterior lobe of the pituitary gland. Researchers
consider it an autoimmune disease, possibly caused by a viral infection. The
treatment is simple in concept, if not always in practice: the missing insulin is
supplied, if necessary by injecting it before meals. Although the name
juvenile diabetes is common, it's a life-long disease. Fortunately, there's no
evidence that its incidence is on the rise.
I
Diabetes is a disorder that takes place at the molecular level. Its main
characteristic is a failure to metabolize glucose, the sugar molecule carried

by the bloodstream to fuel every part of the body. If they dont get their prime
energy supply, muscle and nerve cells slow their function, which is why early
diabetes may manifest itself as permanent tiredness and irritability.
At the same time, glucose accumulates in the patient's blood, and can reach
concentrations two to three times normal and even higher. The excess is
eventually excreted by the kidneys, which require large quantities of water as
a dilutant thats why constant thirst is another typical symptom.

CASE REPORT: MULTIPLE ENDOCRINE NEOPLASIA PRODUCING


GH-RELEASING FACTOR IN AN ENDOCRINE
PANCREATIC TUMOR
Supply the missing medical terms from their definitions below:
Patient: A 27-year-old male.
Chief complaint: (1) ____________ and back pain.
Past history: The patient, who had developed (2) ____________ and visual
disorder, had undergone surgical (3) ____________ of a pituitary tumor which
was pathologically diagnosed as a (4) ____________ ____________.
Physical findings on admission: Height 178 cm; weight 75 kg; blood
pressure 120/60 mmHg; pulse 62/min and regular; no (5) ____________ or
(6)______________. No abnormal findings for the neck, chest or abdomen.
Acromegaly was noted of the face and (7) ____________ ends of the
extremities.
Abdominal MRI findings: A tumor, 7 cm in diameter, possessing a signal
intensity lower than that of the spleen, was found in the (8) ___________
____________ on T1-weighted images. The central part of the tumor had an
even lower signal intensity suggesting (9) ____________.
Selective (10) ____________ of the (11) ____________ ____________:
A (12) ____________ with a vague tumor stain was found to have a main
feeder from the (13) ____________ ____________.
Surgical findings: The above findings suggested the diagnosis of MEN 1
associated with GH-producing pituitary adenoma and endocrine pancreatic
tumor. The patient underwent excision of the pancreatic tumor. The tumor,
covered with a well- (14) ____________ capsule, tightly adhered to the
spleen, compressing the splenic artery and vein. The tail of the pancreas was
removed along with the spleen.
1
2
3
4
5
6
7
8

pain in the area above the stomach


enlargement of the jaws, fingers, toes
removal
pituitary tumor composed of nonstaining cells
reduced concentration of erythrocytes or hemoglobin in the blood
jaundice
far
slender extension (cauda) of the pancreas

9
10
11
12
13
14

tissue death
radiographic examination of blood vessels
afferent blood vessel of the spleen
pathologically changed tissue
afferent blood vessel of the pancreas
supplied with blood vessels

THE EYE
HOW WE SEE
Most of the missing words in this simple description of the process of
seeing are the MAIN STRUCTURES OF THE EYE:
Light rays enter the dark center of the eye, the ____________, passing
through two transparent structures that extend over it, the ____________ and
the ____________, as well as through the ____________ ____________,
which is filled with ____________ ____________ and positioned anterior to
the pupil.
Then the rays hit the _____________, a refractive structure whose shape is
adjusted for close or distant vision with the help of the ____________
____________ on either side of it. This process is called ____________.
Next the light rays pass through the ____________ ____________, which
contains ____________ ____________, a soft jelly-like substance that fills
and shapes the eyeball. Finally the rays fall on the ____________, a delicate
inner nerve layer of the eye. It contains millions of sensitive receptor cells:
____________ (responsible for dim and peripheral vision) and ____________
(responsible for bright, color and central vision). They initiate ____________
____________ that travel to the brain by means of the ____________
____________, which meets the retina in the region called the ____________
____________ or the blind spot. The area beside it is the location of sharpest
vision called the ____________.
SIGHT or VISION?
sight

1. special sense by which objects are perceived by means of


light
2. act of seeing

vision

1. sight def. 1
2. sight def. 2
3. visual clarity

e.g. Although theres nothing wrong with his sight, his vision was blurred
because of the rain.
The following adjectives, used with SIGHT or VISION, all form medically
relevant collocations. (Two of them are used with both words, although
with different meanings find out which with the help of a medical
dictionary.)
Sort them out:
CENTRAL
COLOR
DAY

FAR / LONG
NEAR / SHORT
NIGHT

PERIPHERAL
SECOND
TUNNEL

DOUBLE
OLD

sight:

________________
________________

________________
________________

________________
________________

vision:

________________
________________
________________

________________
________________

________________
________________

DID YOU KNOW THAT...?


In small groups, read the following items and then exchange
information:

all newborns have blue eyes because melanin, which determines eye
color, is concentrated in the folds of the iris at the time of birth, and only
moves to the surface after a few months, giving the baby his / her
permanent eye color?

humans are the only beings that weep in response to emotional


stimulation?

blinking, which occurs every 7 seconds on average, keeps your eyes from
drying out by moving fluid over the eyeballs?

color vision deficiency, popularly called color blindness, which results from
a lack of one type of cones, is a hereditary condition affecting males much
more than females?

puncture wounds to the eye may cause blindness, especially if the object
stuck in the eyeball is removed, because this can allow the humors to
drain and the retina to detach?

THE IMPORTANCE OF SUNGLASSES


Scan the article on protecting your eyes from sunlight and find out
the types of damage to the eyes caused by exposure to sunlight:

____________________________________________

____________________________________________

____________________________________________

Mark the lines that advise you about what to watch out for when
shopping for sunglasses:

10

Many sunglasses on the market don't fully block the two main types of
ultraviolet radiation (A and B) that have been linked to skin aging, sunburn
and skin cancer. Ophthalmologists warn that prolonged exposure can lead to
various vision problems as well.
Sometimes the effects of sun exposure are immediate - water skiers, or snow
skiers, can develop a type of temporary blindness called photokeratitis, that
occurs when reflected sunlight burns the cornea. Other conditions may be
triggered after years of exposure. Studies have linked UV radiation to an
increased risk of cataracts, in which the lenses of the eyes become cloudy, as
well as to macular degeneration, in which the cells in the central part of the
retina slowly deteriorate.
The right pair of sunglasses doesn't have to be expensive. You just have to
make sure there's an official label indicating that the lenses absorb 99% to
100% of the UVA and UVB rays. However, you should be suspicious of any
nonspecific wording such as UV block or maximal UV protection. Specific
numbers should always be attached.
Also, deep tinting without the right label can actually be more harmful:
normally our eyes widen and let in more light when we wear dark glasses.
Unless the lenses are already designed to absorb ultraviolet light, the risk of
damaging the eyes only grows.
For extra safety, choose lenses that are polarized. This decreases glare by
deflecting the sunlight that is reflected off such smooth surfaces as sand,
water and pavement.
The style of the frames can be as important as the lenses, as they should
protect your eyes not only from direct light but also from the light that comes
from different angles, from above or the sides. Wraparound frames that fit
against your face are most effective at closing off the open spaces around the
eye sockets.
And dont put away your sunglasses when summer is over - ultraviolet
radiation doesnt disappear on cloudy days or in winter!

WHAT OPHTHALMOSCOPY CAN REVEAL


Read how a simple ophthalmoscopic examination can be indicative in
relation to the patients general health. In the table below, list all the
problems that can be detected from the appearance of / changes on
certain structures:
Different parts of the eye can be visualized by the ophthalmoscope, an
instrument
consisting of a mirror that reflects light into the eye, a series of lenses that
magnify the interior of the eyeball, and a viewing aperture. By changing focus,
the anterior or posterior eye structures can be inspected.
In a hypertensive patient, the retinal arterioles are significantly narrowed. If
the hypertension is long-standing, they will have thickened walls and a
reddish-brown discoloration. Yellow spots on the retina, known as Hollenhorst
plaques, are tiny particles of cholesterol that have usually broken off bigger

11

plaques in the carotid artery, and indicate a high probability of serious


cardiovascular disease such as CVAs.
Tiny aneurysms on the retina are usually associated with diabetes mellitus
which has progressed for more than 10 years. Diabetic patients may also
develop other retinal changes, retinal detachment and cataracts (clouding of
the lens).
Convoluted retinal vessels are produced by sickle-cell anemia, while smallvessel bleeding in the retina reflects certain protein-producing malignancies.
A blurring of the optic disc can be caused by increased pressure in the brain
resulting from infection, stroke, or trauma.
In the anterior structures, clouding of the cornea can be a sign of some
inherited disorders of sugar metabolism; a brown ring around the outer margin
of the cornea often helps the diagnosis of Wilson disease, characterized by
high levels of copper in the body.
A yellowish (jaundiced) sclera usually indicates liver and / or gallbladder
problems.
Dilated pupils are an indication of shock, intoxication, or various nervous
system disorders.

STRUCTURE
Appearance / changes
RETINA

CORNEA

LENS

SCLERA

PUPIL

OPTIC DISC

INDICATED CONDITIONS /
POTENTIAL PROBLEMS

12

DO YOU WEAR GLASSES?


At your age, your problem is most probably MYOPIA
(NEAR/SHORTSIGHTEDNESS). Here is a simple description of what
causes it:
The physiological cause of myopia is either that the eyeball is too long, or the
refractive power of the lens is too strong, so that the light rays focus in front of
the retina instead on it. Consequently, the image perceived is blurred. This
condition is corrected by means of concave lenses.
The opposite problem is HYPEROPIA (FARSIGHTEDNESS). On the basis
of the above, write a short explanation of the physiological causes of
this condition:
_____________________________________________________________________________________________
_____________________________________________________________________________________________
__________________________________________________________________________________________

However, the typical farsightedness in older people (PRESBYOPIA) has


another underlying cause. Do you know which?

BIONICS - NEW HOPE FOR THE BLIND?


Read about research into new technologies aiming at restoring vision to
the blind and decide if the statements below are true or false. Mark the
lines that helped you decide:
Retinal implants are the big news in neural prostheses research. Fifteen years
ago, the idea of placing a tiny electronic device into the human eye seemed
like pure sci-fi. Compared with the ear, where cochlear implants have been
effectively converting sound into electrical impulses since the mid-80s, the
eye is a more sensitive and complex piece of human anatomy. But today, a
dozen teams of scientists around the world have made artificial eyesight for
the blind their goal. They are proving that such technology is safe, and that it
can improve the quality of life for the vision-impaired, although it wont give
them what we consider normal sight.
Researchers have already demonstrated that sending current to tiny
electrodes placed on the retina of a blind person can cause some of them to
see light and shapes. Teams in Europe, the US and Asia are presently trying
different ways to implant these devices onto a retina.
Theoretically, a camera mounted on the patients glasses would transmit
images in the form of electric currents onto the retina the way a TV camera
sends pictures to your room. The optic nerve would then take the impulses to
the brain. Its not clear yet if such a system is possible, or how much vision it
might create.

13

Although optimistic predictions see it happen sooner, most researchers say


such a system is at least a decade away.
1. Technologies to restore hearing to the deaf and sight to the blind are
developing parallelly.
2. The basic role of the discussed implants is to stimulate the production of
electrical signals that would travel to the brain via the optic nerve and there
produce some sort of images.
3. The quality of images resulting from these procedures doesnt differ from
normal sight.
4. Scientists predict at least ten more years of developing technologies
before a possible breakthrough in this field.
CASE REPORT: INTRAOCULAR HEMORRHAGE
Supply the missing medical terms from their definitions below:
Patient: A 42-year-old man presented to the emergency ophthalmic room
with a sudden visual failure in his right eye, that happened after a forceful
straining 3 days prior to the attendance. There was no history of systemic or
ophthalmic diseases, direct or surgical trauma.
Ophthalmic examination: Normal left eye with a (1) ____________
_____________ of 6/6 and right eye (1) ___________ ___________ of CCF
(closed counting fingers), normal (2) ___________ ___________, and 4
different-sized (3) ___________ (4) ___________ (5) ___________. The
crucial one was a (6) ___________, well (7) ___________, leveled, domeshaped hemorrhage with a (8) ___________ surface, extending between the
temporal vascular arcades, ...
.
.
.
Laser surgery: Half- an hour later, a significant amount of the hemorrhage had
been removed down into the (9) ___________ cavity, its level reaching down
to the (10) ___________, and the patient recovered a central vision of 6/36
and clearer (11) ___________ ___________ ____________. The next day,
visual acuity improved to 6/12, and the major part of blood was cleared away
from the (12) ___________. Follow up showed... no retinal damage or
(13) ___________ due to the laser treatment, and (14) ___________ was not
required.
1 clarity of seeing
2 front part
3 located in front of the retina
4 located below the substance resembling glass
5 bleedings
6 located in front of the macula
7 limited
8 having a rounded, elevated surface
9 glasslike
10 tiny pit in the center of the yellow spot
11 lower area of vision
14

12 irregular yellowish depression on the retina, area of sharpest vision


13 repeated bleeding
14 surgical extraction of the vitreous body

15

Das könnte Ihnen auch gefallen